Location via proxy:   [ UP ]  
[Report a bug]   [Manage cookies]                

CBM English Book 2022-1

Download as pdf or txt
Download as pdf or txt
You are on page 1of 137

GRAMMAR BASICS

Before you get started, it’s a good idea to review some terms that will be used in this section. Remember,
you will not be tested on the names of things or rules; you will only need to recognize what is correct and
incorrect grammar. But to review, you’ll need to be familiar with the following areas of grammar.

NOUNS
A noun is a word that names something

a person Muhammad, Umar, Jafar, Bilal, mother, friend, coworker, student

a place United states, France, city, state

a thing Cat, dog, pencil, lemonade, glass

an idea Truth, justice, the American way

In the following examples, all the nouns are in italics:

New York City boasts one of the most extensive subways in the world.
Jamshed was able to ride from one end of Chicago to the other in a subway car.
Most of the system is underground, but sometimes the tracks go above ground.

You may notice that some nouns are capitalized. These are called proper nouns. The nouns that are not
capitalized are called common nouns.

PRONOUNS
A pronoun is a word that takes the place of a noun. It stands for a person, place thing, or idea in a sentence.
All the pronouns in the following example are in italics:

Ahmed is an excellent soccer player. He enjoyed the game. It proves to be good exercise and a lot of fun as
well. His mother is thrilled that he plays. She feels it is a great sport.

The list of pronoun is as follows:

Subject Reflexive Object Possessive


I myself me mine
You yourself you your
He himself him his
She herself her hers
It itself it its
Who whom whose
We ourselves us our
They themselves them their
VERBS
A verb is an action word. Do not let the word action fool you, though.

Verbs can express physical action:


I ran through the house.
Danish kicked the ball.

They can express mental action:


Maaz thought he would do well on the exam.
Hasan felt the meal was too expensive.

Or, they can express a state of being:


The test is easy.
You are interesting.

Even though there may be more than one verb in a sentence, try to identify the main verb. This is the one
that identifies the main action or actions in a sentence. The main verb must be conjugated. This means that
it will not be in the form “to _______,” or with an–ing (by itself) at the end. For example:

To run could not be a main verb

Running could not be a main verb (by itself)

Run or runs could be a main verb

Drill 1
Directions: Circle the main verb in each sentence: Answers are given on page at the end of the chapter.

1. The tiny island is surrounded by a spectacular body of water.


2. My pen is running out of ink.
3. I always wanted to go to that show.
4. Did you enjoy the party the other night?
5. My computer program is faulty.
6. The customers in the shop are most important.
7. The Pasco Del Rio runs along the San Antonio River in the heart of the city.
8. This beautiful sword is a wonderful souvenir of an event.
9. You don’t need to read all the information to get the question right.
10. My sister was thrilled to be invited.

MODIFIERS
A modifier is a word or group of words that describes another word in the sentence. Adjectives and adverbs
are types of modifiers.

ADJECTIVES
An adjective is a word or group of words that describes a noun or a pronoun. Adjective can describe a lot of
different things about nouns. They can tell you:

What kind of noun?


The book was red. What kind of book was it?
Red

My father is smart. What kind of father is he?


Smart

Serious students do well. What kind of students?


Serious
ADVERBS
An adverb is a word that describes a verb, an adjective, or another adverb. They answer a different set of
questions in a sentence. Adverbs generally tell you how much or how well, when, where, or how. For
example

Arif has learned to speak English perfectly.


How has he learned to speak? Perfectly.

Ammar really loves to go to the park.


How much does he love to go? Really.

The class started very late.


When did the class start? Late.
How late was the class? Very.

Jamshed performed well on the test.


How did Jamshed perform? Well.

Drill 2
It is very common to use adverbs incorrectly in place of adjectives and vice versa. Just remember, if you are
describing a noun, use an adjective. If you are describing anything else, use an adverb, Many adverbs end in
–ly.

Directions: Decide whether to use the adjective or adverb in each of the following:

1. Muhammad performed good/well on the exam.


2. The car moves quick/quickly.
3. The ground was firm/firmly after the night’s frost.
4. Junaid scored the test accurate/accurately.
5. The winner of the contest has certainly performed beautiful/beautifully.
6. My new car runs so smooth/smoothly.
7. The color of my house is bright/brightly yellow.
8. That old computer is so slow/slowly.
9. After going to the picnic, Bush was real/really tired.
10. That meal was delicious/deliciously.

SUBJECT
The subject of the sentence is the noun or pronoun that performs the action in the sentence. If you are not
sure of the subject, you can find it easily: Simply find the verb and then ask yourself who or what. The
answer will be the subject.

For example:

Jamshed loves to play the game.


Who loves to play the game? Jamshed.
Jamshed is the subject.

The highest scorer on the test was Ahmed.


Who was the highest scorer? Ahmed.
Ahmed is the subject.

The Capital of Massachusetts is Boston.


What is the capital of Massachusetts? Boston.
Boston is the subject.
Drill 3 :Directions: Identify and circle the subject in each of the following sentences:

1. Danish wanted to get a new hat at the store.


2. In addition to being weatherproof, the new camera features a remote control.
3. His eyes peer inquisitively from behind metal-rimmed glasses.
4. The main attraction of the new restaurant is the salad bar.
5. After deciding to go out and play, Umer located his jacket.
6. The arboretum in our town has some beautiful pine trees.
7. Unlike those in the neighboring school, our teachers are quite flexible.
8. My new English class meets three times a week.
9. The bus was so late that we missed school.
10. The county fair is a lot of fun.

Drill 4
Directions: In each of the following sentences try to identify the verb and the subject of the sentence and
the part of speech of each word. Remember, not every part of speech will show up in every sentence.

1. Ahmed wants a really great book. 6. The new mayor decided to get supplies and a
Verb: __________________ new bookcase for her office.
Subject: ________________ Verb: __________________
Adjectives: ______________ Subject: ________________
Adverbs: ________________ Adjectives: ______________
Adverbs: ________________
2. Hamza went to the park with Javed and fed
the pigeons stale bread. 7. After next week Zeeshan will go up to the
Verb: __________________ cabin for the summer, and possibly for part of
Subject: ________________ the fall.
Verb: __________________
Adjectives: ______________
Subject: ________________
Adverbs: ________________
Adjectives: ______________
3. At the local store you can buy a wonderful Adverbs: ________________
present for your brother at a good price.
Verb: __________________ 8. After returning from the store, Ali
Subject: ________________ discovered he had forgotten the most
important item on his list.
Adjectives: ______________
Verb: __________________
Adverbs: ________________
Subject: ________________
4. The people in the house next door are always Adjectives: ______________
going on long vacations. Adverbs: ________________
Verb: __________________
Subject: ________________
Adjectives: ______________
Adverbs: ________________

5. Usman got a ride to school with my sister in


her new car.
Verb: __________________
Subject: ________________
Adjectives: ______________
Adverbs: ________________
Drill 1 Answers

1. is surrounded
2. is running
3. wanted
4. did enjoy
5. is
6. are
7. runs
8. is
9. don’t need
10. was thrilled

Drill 2 Answers

1. well
2. quickly
3. firm
4. accurately
5. beautifully
6. smoothly
7. bright
8. slow
9. really
10. delicious

Drill 3 Answers

1. Danish
2. new camera
3. his eyes
4. salad bar
5. Umer
6. arboretum
7. our teacher
8. class
9. bus, we
10. county fair

Drill 4 Answers

1. Verb: wants. Sub: Ahmed.


Adjectives: great. Adverbs: really.
2. Verb: went, fed. Sub: Hamza. Nouns & Adjec: stale.
3. Verb: can buy. Sub: you. Adjectives: local, wonderful, good.
4. Verb: are going. Sub: people. Adjectives: next door, long. Adverbs: always.
5. Verb: got. Sub: Usman. Adjectives: new.
6. Verb: decided. Sub: mayor, Adjectives: new
7. Verb: will go, Sub: Zeeshan. Adjectives: next. Adverbs: up, possibly.
8. Verb: discovered, had forgotten. Sub: Ali. Adjectives: important Adverbs: most.
Grammar Basics Exercise:
1. The racehorse ran swifter in today’s race than he ran in his practice sessions last week. No error
A B C D E

2. The realtor felt badly about not being able to sell their house, because they were in a big hurry to
A B C
move to their condominium. No error
D E

3. When one leaves his car to be repaired, he assumes that the mechanic will repair the car good. No
A B C D E
error

4. Noman played a real fine game in spite of the fact that the Jets lost by a touchdown which the
A B C D
opposing team scored in the last minute of play. No error.
E

5. The butterfly’s wing made whispering sounds as the creature rose beautiful to the blue sky. No
A B C D E
error

6. It hurt Haris that his friends laughed so cruel about the Halloween costume that he had made.
A B C D
No error
E

7. Dawood and Atif felt badly that they knocked over the computer when they were playing catch in
A B C D
the library. No error
E

8. When the Trojans lost the bowl game, Rizwan felt so badly that he moped during the entire evening.
A B C D
No error.
E

9. Moving quick, the Navy Seal Team came on shore to secure the landing zone for the troops that
A B C D
followed. No error
E

10. Although the new device was the most clever designed bird feeder that Mr. Siraaj had ever
A B C
owned, it could not keep squirrels from stealing the birdseed. No error
D E
Grammar Basics Exercise Answers
1. A
2. B
3. D
4. A
5. D
6. C
7. A
8. C
9. A
10. B
Adjectives and Adverbs Practice Test

1. Choose the correct sentence:


A. After a good night’s rest, one can think clearly.
B. After a good night’s rest, one can think real clearly.
C. After a good night’s rest, one can think clear.

2. Choose the correct sentence:


A. The omission of his name from the list was intentional.
B. The omission of his name from the list was made intentional.
C. The omission of his name from the list was intentionally.

3. Choose the correct sentence:


A. I make an effort to drive by the bakery because everything smells so deliciously.
B. I make an effort to drive by the bakery because everything smells very deliciously.
C. I make an effort to drive by the bakery because everything smells very delicious.

4. Choose the correct sentence:


A. The mayor will certain veto this foolish expenditure.
B. The mayor will sure veto this expenditure.
C. Surely, the mayor will veto this foolish expenditure.

5. Choose the correct sentence:


A. On the senior class field trip, the seniors behaved really bad and disappointed the teachers.
B. On the senior class field trip, the seniors behaved real bad and disappointed the teachers.
C. On the senior class field trip, the seniors behaved really badly and disappointed the teachers.

6. Choose the correct sentence:


A. Missing the train, Joe felt badly about not being at the game.
B. Missing the train, Joe felt bad about not being at the game.
C. Missing the train, Joe felt real bad about not being at the game.

7. Choose the correct sentence:


A. My old car has been running bad since I had it overhauled.
B. My old car has been running well since I had it overhauled.
C. My old car has been running good since I had it overhauled.

8. Choose the correct sentence:


A. Martin sure has had a bad day.
B. Martin certainly has had a bad day.
C. Martin has sure had a bad day.

9. Choose the correct sentence:


A. After his third arrest, his alibi did not sound plausible to the jury.
B. After his third arrest, his alibi did not sound plausibly to the jury.
C. After his third arrest, his alibi did not sound real plausible to the jury.

10. Choose the correct sentence:


A. Mr. Jones speaks boastful about his children.
B. How boastfully Mr. Jones speaks about his children!
C. How boastful Mr. Jones speaks about his children!
Adjectives and Adverbs Practice Test Answer and Explanation

Question 1
A is correct. The adverb clearly is modifying the verb think.

Question 2
A is correct. The adjective intentional is modifying the noun omission.

Question 3
C is correct. The adjective delicious is modifying the sense verb smells.

Question 4
C is correct. Surely is an adverb that modifies the verb veto.

Question 5
C is correct. The adverb badly is modifying the verb behaved, and the adverb really is modifying the adverb
badly.

Question 6
B is correct. The adjective bad is modifying the sense verb felt.

Question 7
B is correct. The adverb well is modifying the verb running.

Question 8
B is correct. The adverb certainly is modifying the verb (has) had.

Question 9
A is correct. The adjective plausible is modifying the sense verb sound.

Question 10
B is correct. The adverb boastfully is modifying the verb speaks.
SUBJECT-VERB AGREEMENT
Rule 1:

Remember that the subject and verb in a sentence must agree in person and number.

The elevator works very well. The elevators work very well.

SUBJECT SEPARATED FROM THE VERB


When solving the grammar test, you must always check the subject and verb to be sure they agree.
However sometimes it is difficult to decide exactly what the subject is, if the subject and verb are
separated.

The boys in the room are studying.

Very often, if the subject and verb are separated they will be separated by a prepositional phrase. The
preposition phrase has no effect on the verb.
subject + [prepositional phrase] + verb

The study of languages is very interesting.

Several theories on this subject have been proposed.

The view of these disciplines varies from time to time.

The danger of forest fires is not to be taken lightly.

The effects of that crime are likely to be devastating.

EXAMPLE 1:
1. When she spoke with the police, she reported her loss; she stated that a large quantity of
A B
clothing and of valuable jewelry were missing. No error
C D E
2. Between the small shops and boutiques of Greenwich Village and the giant department
A B
stores of midtown Manhattan lie the ethnically varied residential neighborhood of
C D
Chelsea. No error
E
Rule 2:
If you see any of these five expressions in the sentence, check out the subject before them. If the
subject is singular before these expressions we will use singular verb and if the subject is plural we
will use plural verb.
The following expressions also have no effect on the verb.
together with along with in addition to
accompanied by as well as

Nasir, along with his manager and some friends, is going to a picnic tonight.
Mr. Robbins, accompanied by his wife and children, is arriving tonight.
NOTE: If the conjunction and is used instead of one of these phrases, the verb would then be plural.
Nasir and his manager are going to a party tonight.

EXAMPLE 2:
The debate coach, together with the members of the winning team, is traveling to Washington for the
awards ceremony.
(A) together with the members of the winning team, is traveling
(B) along with the members of the winning team, they are traveling
(C) along with the members of the winning team, are traveling
(D) together with the members of the winning team
(E) together with the members of the winning team, are to travel

Exercise: 1 Subject-Verb Agreement


Choose the correct, form of the verb in parentheses in the following sentences.
1. Nasim, along with twenty friends (is/are) planning a party.
2. The picture of the soldiers (bring/brings) back many memories.
3. The quality of these recordings (is/are) not very good.
4. If the duties of these officers (isn’t/aren’t) reduced, there will not be enough time to finish the
project.
5. The effects of cigarettes smoking (have/has) been proven to be extremely harmful.
6. The use of credit cards in place of cash (have/has) increased rapidly in recent years.
7. Advertisements on television (is/are) becoming more competitive than ever before.
8. Living expenses in this country, as well as in many others, (is/are) at an all-time high.
9. Mr. Jahangir, accompanied by several members of the committee, (have/has) proposed some
changes of the rules.
10. The levels of intoxication (vary/varies) from subject to subject.

Rule 3 A: WORDS THAT ALWAYS TAKE SINGULAR VERBS AND PRONOUNS

Some words are often confused by students as being plural. The following words must be followed by
singular verbs and pronouns in formal written English.
any + singular noun no+ singular noun some+ singular noun

any body nobody somebody

anyone no one someone

anything nothing something

every +singular noun each + singular noun

Everybody who has not purchased a ticket should be in this line.


Something
every bodywas under the house.
Anybody who has lost his ticket should report to the desk. (note the singular pronoun)
No problem is harder to solve than this one.
everyone either*
Nobody works harder than Umer does.
everything neither*
EXAMPLE 3:
Everyone who attends a concert at the sports arena knows that they will be searched for
A B C
drugs before entering. No error.
D E

Rule 3 B: Either and neither are singular if they are not used with or and nor, therefore singular
verb should be used for them.
If either of you takes a vacation now, we will not be able to finish the work.

Rule 4: EITHER/NEITHER
When either and neither are followed by or and nor, the verb may be singular or plural, depending on
whether the noun following or and nor is singular or plural. If or or nor appears alone, the same rule
applies. Study the following formulas.

neither + noun + nor + plural noun + plural verb

either or
Examples:
Neither Usama nor his friends are going to the beach today.

Either Usama or his friends are going to the beach today.

neither + noun + nor + singular noun + singular verb

either or

Neither Salim nor Shamir is going to the beach today.


Either Salim or Shamir is going to the beach today.
Examples:
Neither Jamil nor Jaffar is going to class today.
Neither Mohsin nor his friends are going to class today.

Nasir or Aslam is bringing the car.

Note: One should look for the noun after or/nor to decide the verb in the sentence, even if the
sentence do not contain either or neither
Neither the boys nor Kamran has seen this book before.
Neither the director nor the secretary wants to leave yet.
EXAMPLE 4:

You nor I are going to agree with the speaker; sometimes, however, it is a good idea to listen to
someone whom one many disagree with.

A) Neither you nor I are going to agree B) Neither of us are going to agree
C) Neither you nor me is going to agree D) Neither I nor you am going to agree
E) Neither you nor I am going to agree
Rule 5: GERUNDS AS SUBJECTS
If a sentence begins with [verb + ing] (gerund), the verb must also be singular.
Dieting is very popular today.
Not studying has caused him many problems.
Washing with a special cream is recommended for scalp infections.
Being cordial is one of his greatest assets.
Writing many letters makes her happy.

Example 5:
Learning about other cultures make people more tolerant. No error
A B C D

Rule 6: COLLECTIVE NOUNS


Also many words indicating a number of people or animals are singular. The following nouns are
usually singular. In some cases they are plural if the sentence indicates that the individual members
are acting separately.
Congress family group committee class

organization team army club crowd

Examplesgovernment
of collective nouns:
jury pair minority public
The committee has met, and it has rejected the proposal.
The family was elated by the news.
The crowd was wild with excitement.
The organization has lost many members this year.
Our team is going to win the game.
The following nouns are used to indicate groups of certain animals. It is not necessary to learn the
noun; however, they mean the same as group and thus are considered singular.
flock of birds, sheep school of fish

herd of cattle pride of lions

The flock of birdspack of dogsoverhead.


is circling
The herd of cattle is breaking away.
A school of fish is being attacked by sharks.

EXAMPLE 6:
The Debate Club host lunchtime debates on current issues ranging from affirmative action to nuclear
proliferation.
A) The Debate Club host lunchtime debates on current issues
B) The Debate Club host lunchtime debates on current issues,
C) The Debate Club hosts lunchtime debates on current issues
D) Lunchtime debates on current issues being hosted by the Debate Club,
E) Lunchtime debates on current issues hosted by the Debate Club,

Rule 7: A NUMBER OF/THE NUMBER OF


a number of + ……………. + plural verb…..

the number of + ………….. + singular verb….


A number of students are going to the class picnic. (a number of many).
The number of days in a week is seven.
A number of the applicants have already been interviewed.
The number of residents who have been questioned on this matter is quite small.
Example 7:
1. The number of people lined up for tickets (was/were) four hundred.
2. A number of suggestions (was/were) made.
3. There (is/are) a number of important announcements in the bulletin.
4. Here (is/are) the number of milk shakes you requested.

Rule 8: NOUNS THAT ARE ALWAYS PLURAL


The following nouns are always considered plural. They cannot be singular. In order to speak of them
as singular, one must say “a pair of ___________.”
scissors shorts pants jeans tongs

trousers eyeglasses pliers tweezers

The pants are in the drawer.


A pair of pants is in the drawer.
The pliers were on the table.
The pair of pliers was on the table.
These scissors are dull.
This pair of scissors is dull.
Example 8:
This pair of scissors (is/are) for cutting paper.
Your clothes (is/are) dirty.

Rule 9: THERE IS / THERE ARE


Remember that with sentences beginning with the existential there, the subject is actually after the
verb.

there is + singular subject….

there was

there has been (or non-count)


there are + plural subject…

there were

there have been


There is a storm approaching.
There have been a number of telephone calls today.
There was an accident last night.
There were too many people at the party.
There has been an increase in the importation of foreign cars.
Exercise 2: Subject – Verb Agreement
Choose the correct form of the verb in the following sentences.
1. Neither Bashir nor Alam (is/are) going to the school today.
2. Anything (is/are) better than going to another friend tonight.
3. Skating (is/are) becoming more popular every day.
4. A number of reporters (was/were) at the conference yesterday.
5. Everybody who (has/have) a fever must go home immediately.
6. Your glasses (was/were) on the bureau last night.
7. There (was/were) some people at the meeting last night.
8. The committee (has/have) already reached a decision.
9. A pair of jeans (was/were) in the washing machine this morning.
10. Each student (has/have) answered the first three question.
11. Either Musharraf or his brother (make/makes) breakfast each morning.
12. After she had perused the material, the secretary decided that everything (was/were) in order.
13. The crowd at the basketball game (was/were) wild with excitement.
14. A pack of wild dogs (has/have) frightened all the ducks away.
15. The jury (is/are) trying to reach a decision.
16. The army (has/have) eliminated this section of the training test.
17. The number of students who have withdrawn from class this quarter (is/are) appalling.
18. There (has/have) been too many interruptions in this class.
PRONOUNS

Rule 10:Ambiguous References


Some references arc ambiguous because the pronoun could refer to one or more antecedents.

The teacher told the student that it was her responsibility to hand out the books.

Who is responsible? The teacher or the student? Because the pronoun her may refer to either
the teacher or the student, the sentence needs to be rewritten:
The teacher told the student that one of her responsibilities, as teacher was to hand out
books.

AMBIGUOUS: Aslam showed his brother Kamran a copy of a photo he had taken.
(Who took the picture?)
CLEAR: Aslam showed a copy of a photo he had taken to his brother Kamran.

AMBIGUOUS: When Danish phoned his father, he wasn't feeling well.


(Who felt ill?)
AMBIGUOUS: Shakil told his father that he was working too hard.
(Who was working too hard?)
CLEAR: Shakil said to his father, "I'm working too hard."

Example 9:
Mr. Bashir told Mr. Aslam that he would have to work all night in order to finish the report. No
A B C D E
error

PRONOUN CHOICE
Rule 11: Use of Subject Pronouns: I, he, she, it, you, we, they, who
There will always be a verb for Subject Pronoun.
1. Elvis asked that (he, him) and (she, her) practice handstands.
Verb: Practice Who Practice? He and She
He and She are subject pronoun
2. Then he and I went home.
Verb: Went Who went?: He and I He and I =subject
3. When we went out for pizza, he and she fought over the check.
Verb: Went Who Went? We
Verb 2: Fought Who Fought? He and She (he and she = subject)
4. The instructors in the course were he and Donald.
Verb: were Who were? he and Donald

EXAMPLE 10:
Two candidates for the U.S. Senate, Buckley and him, made speeches to the group.
A) Two candidates for the U.S. Senate, Buckley and him, made speeches to the group.
B) Two candidates for the U.S. Senate, Buckley and he, made speeches to the group.
C) Buckley and him, two candidates for the U.S. Senate, made speeches to the group.
D) Speeches to the group were made by Buckley and he, two candidates for the U.S.
Senate.
E) Buckley and he made speeches to the group.
Rule 12: Use of Object Pronouns:
Always use object pronoun after preposition and never use object pronoun for a verb.
BETWEEN you and me, TO Shahzeb and her, AMONG us women, AT us, FROM her and him,
WITH me and you.
Example:
Incorrect: You and me are going to work on this project.
Correct: You and I are going to work on this project.
Verb: are Who are? You and I (subject)
Other Examples of Object Pronouns
Tariq invited him to the party.
The waiter gave him and me orange soda.
EXAMPLE 11:
Because of the bomb threat everyone was asked to evacuate the bank but a security
A B C
guard, a fireman, and I. No error
D E
Rule 13: Pronoun in Comparison:

To find the correct pronoun in a comparison, as in "Jahangir runs faster than (him, he)," first
complete the comparison with the verb that would follow naturally. That will tell you which
pronoun to use.
Jahangir runs faster than he runs.
My brother has bigger feet than I do.

Because you would never say "Jahangir runs faster than him runs" or "My brother has
bigger feet than me do," the correct pronouns are she and I:

Jahangir runs faster than he.


My brother has bigger feet than I.
Apply the same principle to comparisons using as.
Kashif is as tough as he is.
He is twice as fast as they are.
The department could use a stronger leader such as she is.
A man such as I am could solve the problem.
Note: In comparison, we will use subject pronoun after than and as.
EXAMPLE 12:
My grandmother leads a more active life than many other retirees who are
A B C
younger than her. No error.
D E
Rule 14: When a pronoun is side by side with a noun (we boys, us women), eliminate the
noun to determine which pronoun to use.
(We, Us) seniors decided to take a day off from school in late May. (By dropping the noun
seniors, you can easily tell which pronoun is correct. Since no one would say "Us decided to take,"
we is clearly the correct choice.)
This award was presented to (we, us) students by the teachers. (Drop the noun students, and
the proper choice of pronoun becomes clear.)
This award was presented to us by the teachers.
Rule 15: Use of possessive pronouns:
Always use possessive pronoun (my, our, your, his, her, their, its,) before a gerund, a noun that
looks like a verb because of its –ing ending.

Examples:

Her asking the question shows that she is alert. (Asking is a gerund.)
Mother was upset about your opening the presents too soon. (Opening is a gerund.)
Their coming home late upset the evening's plans. (Coming is a gerund.)
EXAMPLE 13:
I appreciate you offering to help me with my research project, but the honor system
A B
prevents students from giving and receiving assistance. No error.
C D E
Rule 16: THE PRONOUN ONE AND YOU
If one (meaning a person in general) is used in a sentence, a subsequent pronoun referring to the
same person must also be one or he. If you is used, the subsequent pronoun must also be you. He or
you can be in the possessive, complement, or reflexive case.

one

one’s + noun + verb …

one+ verb… he*

*NOTE: Many times his


it is+ noun
considered more appropriate to use he or she and similar expressions so that
the masculine pronoun is not used exclusively. On the grammar test, however you need not worry
about the problem. If a sentence begins with one, be sure that you or they DOES NOT follow.
If one takes this exam without studying, one is likely to fail.
If one takes this exam without studying, he is likely to fail.

One should always do one’s homework.


One should always do his homework.

you

you + verb … + your + (verb) …

If you take this exam without studying, you are likely to fail.
You should always do your homework.

NOTE: It is NEVER CORRECT to say:

If one takes this time without studying, you are likely to fail.

If one takes this exam without studying, they are likely to fail.
Singular plural
Example 14:
One should never tell your secrets to a gossip if he wishes them to remain secret. No error
A B C D E

WHO/WHOM
Who and whoever are Subject pronouns.
Whom and whomever are object pronouns.

Rule 17: In general, the patterns for who and whoever are:
a) who (whoever) + verb
The woman who sang yesterday has sung for years
WHO + VERB
Who came to the party?
WHO + VERB

Give the money to whoever needs it.


WHOEVER + VERB
I said that whoever had finished could leave.
WHOEVER + VERB
b)whom (whomever) + subject + verb
OBJ. WHOM+S+VERB
The woman whom I met yesterday is a voice teacher.
OBJ.WHOMEVER + S + VERB
Give it to whomever you like

EXAMPLE 15:
The wealthy socialite decided that her fortune would be left to whomever of her relatives
A B
could present her with the best plan for dispensing part of the money to deserving
C D
charities. No error.
E.

Rule 18: Sometimes expressions like the following separate who (whoever) or whom (whomever) from
its own verb or subject and verb:
"l think"' "she said" "we know" “do you know"

He is a student who we believe can do the job.


Give the job to the person who you think is best suited for it.
He is a man whom I feel you can trust.

EXAMPLE 16:
Tell the story to whomever you think should hear it. No error
A B C D E
Rule 19: COMPARISONS
Comparison Signals
like as
unlike as (adj.) as
Comparisons are a form of parallelism that deserves special attention. more than as much as
less than as little as
As the name indicates, comparisons compare two parts of the sentence faster than as fast as
different from the same as
(or occasionally more). in contrast to/with

To spot comparisons, you must first learn certain signal words or


phrases. Once you find a comparison, identify the two parts of the sentence that are being compared
to each other. Finally, ensure that these two parts are truly parallel, both structurally and logically.

The most important comparison signals are Like, Unlike, As, and Than. Whenever you see one of
these four words, stop and find the two items being compared. Other common comparison signals
are shown in the chart above.
Rule 20: Like vs. As
Like and As are two very common comparison signals. You should learn to distinguish between them.
Like is a preposition. This means that LIKE must be followed by nouns, pronouns, or noun
phrases. Never put a clause or a prepositional phrase after like! (Remember, a clause contains a
working verb, one that can be the main verb in a sentence.) You can correctly use like to compare two
nouns.
Consider the following example:
Right: LIKE her brother, Amna aced the test.

Here, like is followed by the noun phrase her brother. The whole phrase Like her brother indicates a
comparison between Amna and her brother (two nouns). Note that like can be followed by gerunds (-
Ing forms used as nouns): LIKE swimming, skiing is great exercise.
On the other hand, as can be either a preposition (appearing with a noun) or a conjunction
(appearing with a clause). You can correctly use as to compare two clauses. Again, however, you
cannot use like to compare clauses.
Wrong: LIKE her brother DID, Amna aced the test.
Right: AS her brother DID, Amna aced the test.
The words her brother did form a clause (did is a working verb). Therefore, you must use as to make
the comparison between the two clauses Amna aced the test and her brother did. Using like to
compare clauses is common in speech but always wrong in writing.

According to the SAT, there is no difference in meaning between Like her brother; Amna aced the test
and As her brother did, Amna aced the test. You can compare Amna and her brother directly, or you
can compare what they did.

Example 17:
1. He speaks (like/as) a native speaker.
2. Nobody plays (like/as) she does.
3. Like he was getting off at the next station, he started to get his things ready. No error
A B C D E
Rule 21: Keeping Comparisons Parallel
Comparisons must be logically parallel. That is, they must compare similar things.

Farhan`s build, LIKE his brother, is broad and muscular.

What two things are being compared? As written, the sentence is comparing Farhan`s build directly
to his brother. This is not a logical comparison: someone`s build is not in the same class of things as
someone`s brother. In order to correct this error, we need to change the comparison.

Right: Farhan`s build, LIKE his brother`s, is broad and muscular.

We do not need to repeat the word build after brother`s-it is implied.

Right: Farhan`s build, LIKE that of his brother, is broad and muscular.

We can also use the word that to stand for build. If the first noun were plural, we would use those
instead: Farhan`s toes, LIKE THOSE of his brother, are short and hairy.

Right: Farhan, LIKE his brother, has a broad and muscular build.

Example 18:
To go skateboarding is no longer as popular as hockey. No error
A B C D

The climate of Ranchi is better than Gaya (incorrect)


The climate of Ranchi is better than that of Gaya (correct)

Incorrect: His drawings are as perfect as his instructor.


(This sentence compares drawings with instructor.)

Correct: His drawings are as perfect as his instructor’s.


(instructor’s = instructor’s drawings)
Incorrect: The salary of a professor is higher than a secretary.
(This sentence compares salary with secretary)
Correct: The salary of a professor is higher than that of a secretary.
(that of = the salary of)
Incorrect: The duties of a policeman are more dangerous than a teacher.
(This sentence compares duties with teacher.)
Correct: The duties of a policeman are more dangerous than those of a teacher.
(those of = the duties of)

Finally, we can change the first term and rephrase the sentence accordingly.

Take a look at a harder example:

Beethoven`s music, which broke a number of established rules with its structure and
melodic form, is considered more revolutionary than Bach.

First, we find the comparison signal: MORE revolutionary THAN… Now we look for the two things
being compared. It is often easier to find the second thing, which follows the comparison signal: More
revolutionary than Bach. So, what is more revolutionary than Bach? The subject of the sentence:
Beethoven`s music. This comparison is not parallel.
We have to be careful, since sometimes we talk about the music of Bach as “Bach” (e.g., I like to listen
to Bach on the radio). However, if the sentence has referred to Beethoven`s music with the word
music, then the sentence should do the same with Bach`s music.

Right: Beethoven`s music, which broke a number of established rules with its structure and
melodic form, is considered MORE revolutionary THAN BACH`S.

Note again that we do not have to repeat the word music, as long as we have written Bach`s.
We could also write that of Bach at the end of the sentence.

Comparisons must be structurally parallel. That is, they must have a similar grammatical
structure.

I like to run through forests more than I enjoy walking through crowds.
Are the objects of comparison grammatically parallel? No, because to run through forests does not
have the same structure as walking through crowds. To run is an infinitive, whereas walking is a
gerund (walking is being used as a noun). To write a concise, parallel sentence, we should simply use
one verb (like) and convert both objects to –Ing forms.
Right: I like running through forests MORE THAN walking through crowds.

Example 19:
The novel Pride and Prejudice by Jane Austen was once more widely read and was more popular in high
A B C
schools in the United States than Charlotte Bronte. No error.
D E

Rule 22: Comparative and Superlative Forms


When comparing two things, use the Comparative Form of an adjective or adverb. When comparing
more than two things, use the Superlative Form of an adjective or adverb.

Regular Forms
Comparative: She is SHORTER than her sister. (Add –er)
Superlative: She is the SHORTEST of her five siblings. (Add –est)
Comparative: You are MORE INTERESTING than he. (Add the word more)
Superlative: You are the MOST INTERESTING person here. (add the word most)
Comparative: You are LESS INTERESTING than she. (Add the word less)
Superlative: You are the LEAST INTERESTING person here. (Add the word least)
For irregular forms such as good/better/best, see the Glossary on “Comparative Forms” and
“Superlative Forms.”
Do not compare an adverb that ends in –ly by changing the ending to –er. This error is common in
speech. Instead, add more.
Wrong: Adnan runs QUICKLY. He runs QUICKER than Javed.
Right: Adnan runs QUICKLY. He runs MORE QUICKLY than Javed.
However, some adverbs that do not end in –ly are made into comparatives by adding –er.
Right: Adnan runs FAST. He runs FASTER than Javed.

Do not use a comparative adjectives unless you have a than in the sentence.
Wrong: With winter coming, I will have HIGHER energy bills.
The sentence implies the comparison than now. On the SAT, however, you must make that
comparison explicit, using the word than.
Wrong: I will have HIGHER bills OVER last year.
Right: I will have HIGHER bills THAN last year.

Always use than with a comparative form.

Example 20:
I like apples better(than/over)papayas.

Rule 23: Double comparative and double superlatives must not be used.
He is more wiser than his brother (incorrect)
He is wiser than his brother (correct)

Rule 24: When comparative degree is used in superlative sense, it is followed by


‘any other’
Umer Gul is better than any bowler (incorrect)
Umer Gul is better than any other bowler (correct)

Example 21:
1. This flower is more beautiful than any flower in this garden. No error
A B C D E

2. This house is bigger than any other house in this street. No error
A B C D E

Rule 25: Pronoun in comparison


To find the correct pronoun in a comparison, as in "Jahangir runs faster than (him, he)," first
complete the comparison with the verb that would follow naturally. That will tell you which
pronoun to use.

Jahangir runs faster than he runs.


My brother has bigger feet than I do.

Because you would never say "Jahangir runs faster than him runs" or "My brother has
bigger feet than me do," the correct pronouns are she and I:

Jahangir runs faster than he.


My brother has bigger feet than I.
Apply the same principle to comparisons using as.

Kashif is as tough as he is.


He is twice as fast as they are.
The department could use a stronger leader such as she is.
A man such as I am could solve the problem.

Example 22:
Bilal admitted that he has less money than I.
(A) Bilal admitted that he has less money than I.
(B) Bilal admitted that he has fewer money than I
(C) Bilal admitted that he has less money than me.
(D) Bilal admitted that he has less money than I
(E) Bilal admitted that he has fewer money than me

Rule 26: ILLOGICAL PARTICIPIAL MODIFIER (DANGLING MODIFIER)

A participial phrase (one containing a [verb + ing] without auxiliaries) can be used to join two
sentences with a common subject. When the two phrases do not share common subject, we call the
participial phrase an illogical participial modifier. Actually, the subject of the participial phrase is
understood rather than explicit. Consider the following sentences.

Incorrect: After jumping out of a boat, the shark bit the man. (We understand
that the actual subject of the verb jumping is the man; therefore,
immediately after the comma, we must mention the man.)

Correct: After jumping out of the boat, the man was bitten by shark.

For clarity, introductory participial phrases must be followed immediately by the noun which is
logically responsible for the action of participle. There is no written subject in the participial phrase
is preceded by the preposition. The following preposition commonly precede participial phrase.
by upon before after while

After preparing the dinner, Michelle will read a book.


By working a ten-hour day for four days, we can have a long weekend.
While reviewing for the test, Marcia realized that she had forgotten to study the use
of participial phrase.

Illogical: Attacked by an angry mob, the gashes in the boy’s throat


were life-threatening.
(Note that attacked means the same as having been attacked.
The actual subject of the verb attacked is the boy; therefore
reference to him must appear immediately after the comma.)

Correct: Attacked by an angry mob, the boy suffered life-threatening


gashes in his throat.

Observe the corrected form of the following illogical participial modifier. Remember that the noun
appearing after the comma must be the logical subject of the participial modifier.

Illogical: Having apprehended the hijackers, they were whisked off to


FBI headquarters by the security guards.
Correct: Having apprehended the hijackers, the security guards
whisked them off to FBI headquarters.
(After the guards had apprehended the hijackers, the
guards whisked…)
Illogical: Before singing the school song, a poem was recited.
Correct: Before singing the school song, the students recited a poem.
(Before the students sang…)
Illogical: Guiding us through the museum, a special explanation was
given by the directors.
Correct: Guiding us through the museum, the director gave us a special
explanation.
(While the director was guiding us)
Exercise 7: Illogical Modifiers
Following the examples given above, correct these illogical participial modifiers. You may have to
reword the main clause and add a subject.

1. Being thoroughly dissatisfied with the picture, it was hidden in the closet.
2. Seeing the advancing army, all valuables were hidden under the stairwell.
3. Plunging into the water, the drowning child was rescued.
4. Criticizing the defendant for this cruel behavior, the sentence was handed out by the judge.
5. After painting the car, it was given to the man’s wife by the man.
6. Being an early rise, it was easy for Edna to adjust to her company’s new summer schedule.
7. After winning the tennis match, the victory made Nancy jump for joy.
8. Having wandered through the mountain passes for days, an abandoned shack where they could
take shelter was discovered by the hikers.
9. Being very protective of its young, all those who approach the nest are attacked by the mother
eagle.
10. Before playing ball, a two-minutes period of silence was observed by the baseball players for their
recently deceased teammate.

EXAMPLE 23:
Lecturing at the university, read the poetry of Allama Iqbal was the advice Professor Arshad
A B C
gave his audience. No error
D E
Rule 27: REDUNDANCY
A sentence in which some information is unnecessarily repeated is called redundant. Given here are
some words combinations that are always redundant, and thus should NEVER be used.
advance forward advance, proceed, and progress all mean “to

proceed forward move in a forward direction”; thus, the word

progress forward forward is not necessary

return back return and revert mean “to go back or to send

revert back back” so back is not necessary

sufficient enough these words are identical; one or the other


should be used

compete together compete means “to take part in a contest


against others”

reason …because these words indicate the same thing; the


pattern is reason … that

join together join means “to bring together,” “to put


together” or “to become a part or member
of,” “to take place among”

repeat again repeat means “to say again” (re-usually


means “again”)

new innovation innovation mean “a new idea”

same identical these words are identical

two twins twins means “two brothers or sisters”

the time when the time and when indicate the same thing;

one or the other should be used

the place where the place and where indicate the same thing;

one or the should be used

SUBJECT AREA REVIEWS


Examples of correct sentences

The army advance after the big battle.


OR
The army moved forward after the big battle.

The peace talks advance.


OR
The peace talks progressed.
We have sufficient money to buy the new dress.
They have enough time to eat a sandwich before going to work.
The teacher proceeded to explain the lesson.
Jaffar and his brother are competing in the running games.
The teacher asked us to join the student who were cleaning the room.
Musab repeated the question slowly so that Jaffar would understand.
Besides the two evening showing, there will also be a matinee.
The reason I want to take that class is that the professor is supposed to be very
eloquent.
This is where I left him.
That was the time I hit a home run.

Exercise 9: Redundancy
Cross out the Redundant in which of the following sentences.

Example:
The carpenter join the two beams together with long nails.
(Together is the redundant word.)

1. After Ali had shown Tahir how to insert a paper once, he repeated the operation again.
2. The twins have the same identical birthmarks on their backs.
3. I think we have sufficient enough information to write the report.
4. When the roads became too slippery, we decided to return back to the cabin and wait for the
storm to subside.
5. The mountain climbers proceeded forward on their long trek up the side of the mountain.
6. Raheel and his brother competed together in the sports gala.
7. I think that we should come up with a new innovation for doing this job.
8. The minister joined the bride and groom together in holy wedlock.
9. My cousins love to play with the two twins from across the street.

Rule 28: PARALLEL STRUCTURE

When information in a sentence is given in the form of a list or series, all components must be
grammatically parallel or equal. There may be only two components or there may be many
components in a list; however, if the first is, for examples, an infinitive, the rest must also be
infinitives. Consider the following correct and incorrect sentences.

Not parallel: Pasha is rich , handsome , and many people like him.
adjective adjective clause
Parallel: Pasha is rich , handsome , and popular.
adjective adjective adjective
Not parallel: Mr. Haris is a lawyer, a politician, and he teaches.
noun noun clause

Parallel: Mr. Haris is a lawyer, a politician, and a teacher.


noun noun noun

(Remember that a clause standing alone would be a complete sentence, meaning it has a subject and
a verb.)

Not parallel: The soldier approached the enemy camp slowly and silent.
adverb adjective
Parallel: The soldier approached the enemy camp slowly and silently.
adverb adverb
Not parallel: He likes to fish , swim , and surfing .
infinitive simple form [verb+ing]
Parallel: He likes to fish, to swim, and to surf.
infinitive infinitive infinitive
OR
He likes fishing , swimming, and surfing.
[verb+ing] [verb+ing] [verb+ing]
Not parallel: When teenagers finish high school, they have several choices
going to college, getting a job, or the army.
verb + noun verb + noun noun
Parallel: When teenagers finish high school, they have several choices:
going to college, getting a job, or joining the army.
verb + noun verb + noun verb + noun
Not parallel: Tasneem entered the room, sat down, and is opening his book.
past past present progressive
Parallel: Tasneem entered the room, sat down, and opened his books.
past past past

Note: If the sentence indicates that the different clauses definitely happened or will happen at
different times, then this rule does not need to be followed. For example:

He is a senior, studies everyday, and will graduate a semester early.


present present future

Exercise 10 : Parallel Structure


Change the following sentences so that they are parallel, if needed.

1. The puppy stood slowly, wagged its tail, blinked its eyes, and barked.
2. Ecologists are trying to preserve our environment for future generation by protecting the ozone
layer, purifying the air, and replanting the trees that have been cut down.
3. The chief of police demanded from his assistants an orderly investigation, a well-written report,
and hard work.
4. Mutahir is a scholar, an athlete and an artistic.
5. Slowly and with care, the museum director removed the Ming vase from the shelf and placed it on
the display pedestal.
6. The farmer plows the fields, plants the seeds, and harvests the crop.
7. Quaid-e-Azam was a good leader and was well educated, hard-working, and always told the truth.
8. Children love playing in the mud, running through puddles, and getting very dirty.
9. Collecting stamps, playing chess, and mounting beautiful butterflies are Derrick’s hobbies.
10. Despite America’s affluence, many people are without jobs, on welfare, and have a lot of debts.

Rule 29: FREQUENTLY MISUSED WORDS AND PHRASES


and/or
This phrase, found most often in legal writing, makes even the most charming words sound like a
manual for vacuum-cleaner maintenance. "Empty and/or clean your machine after every use,"
says the instruction booklet. Such a phrase has no business in an essay.

data
Data is a synonym for facts and statistics and is always plural, although it is increasingly
used as a singular noun.
The data show that few drivers observe posted speed limits any more.
A single fact is a datum, but the word is rarely used. Rather, "a piece of data" would do just fine.
between, among
Use between to refer to anything split into two or divided by two, like the Milky Way that Barbara
split between George and herself. Use among for a division by more than two. For example, the
seven famished kids divided the Doritos among themselves. Also:
Among the three of us, we should figure out the answer.
We should probably choose between Katie's and Sam's answers.

different from, different than


Republicans are different from (not different than) Democrats because one political party differs
from another. Since the phrase to differ than is not English, different than is not standard usage. Use
different from.

Colloquial: Rich people are different than you and me


Standard: Rich people are different from you and me

due to, owing to


These phrases are often used interchangeably, but in standard usage, owing to means "because of,"
as in "Owing to the strike, the milk was not delivered." Due to means the "result of," as in "That the
milk was finally' delivered was due to the contract settlement." Owing to may begin a sentence,
but due to may not.

lose, loose
One letter, the o, makes a difference. Don't confuse lose with loose, which rhymes with goose. Loose
things include rules, shoelaces, screws, and escaped killers "on the loose." Among other things, you
can lose in Monopoly, lose your way, and lose your mind. Be careful with loose change in your
pocket, or you may lose it.

affect, effect
Affect is a verb, effect is a noun, except in a rare instance. Spelling affect (verb) when you mean effect
(noun) will affect your reputation as an accurate speller. The effect on the SAT II may be a lower
score. Then again, your score may not be appreciably affected.
The rare case: when effect means to bring to pass or to accomplish, as in "Study hard to effect a change
in, your spelling performance."

all right
All right is two words; it's never one. It's not like already and altogether, which can be either one
word or two, depending on your meaning. Remember that and you'll do all right.

already
Like altogether, already is one word unless used in the sense of "The dinner is all ready to eat." Already
as a single word is an adverb referring to a prior, present, or future time, as in "Enough, already."

altogether
Altogether is one word, except when referring to a group, as in, "The family was all together at
Thanksgiving." Otherwise, altogether (as on.- word) means thoroughly or completely, as in "Spelling
is altogether too complicated."

Because (used after reason)


Nonstandard: the reason for her back pain is because she has poor pasture.
(Use that instead of because)
Standard: the reason for her back pain is that she has poor posture.
Being as/being that
Nonstandard: Being that hurricanes are common, no one builds houses close to water.
(being that and being as should not be used in place of because)
Standard: because hurricanes are common, no one builds houses close to the water.

Former/latter
Nonstandard: After visiting Colby, Bates and Bowdoin, Joey chose the latter.
(both latter and former refer to two)
Standard: After visiting Colby, Bates and Bowdoin, Joey chose Bowdoin.
After visiting Colby, Bates and Bowdoin, Joey chose the latter.
After visiting Colby, Bates and Bowdoin, Joey chose the former.

Hisself/himself
Nonstandard: At ten months, he could walk by hisself.
Standard: At ten months, he could walk by himself.

Nonstandard: Bodybuilders like to look at them-self in the mirror.


Standard: Bodybuilders like to look at themselves in the mirror.

Reason is because…
Colloquial: The reason for the delay is because of mechanical trouble.
Standard: The reason for the delay is mechanical trouble.

Used to:
Nonstandard: Brian use to arrive late to class almost every day.
(The phrase use to is never correct. Use used to)
Standard: Brian used to arrive late to class almost every day.

who's, whose
Like every contraction, who's is a marriage of two words, in this case who and is, as in " Who's going
to spell who's correctly from now on?" Answer: You! Whose, on the other hand, is a type of
pronoun indicating possession, as in "You're a person whose spelling is getting better.

doesn't
Doesn't combines does and not. The apostrophe takes the place of the o in not. That's the only
difference between spelling does not and doesn't. Therefore, it doesn't make sense to write doesn’t,
as many people do.

its, it's
Its is a marriage of it and is. The apostrophe stands for the missing letter. That is the only way
its correct. Its (without the apostrophe) indicates possession, as in "The cat chased its tail."
You won't go far writing her’s or his’s, so don't write it unless, you mean it is.

than, then
Use than rather than then when making a comparison (Phil eats faster than Fido.) Then is a time
word, like the word when. Both contain the letter e. Remember this tale: Phil eats faster than his dog.
First Phil finishes, then Fido. The end.

good, well
Good is an adjective. Like any adjective it can be used before a noun, as in good apple, good grief, and
good night. That's easy.
Good sometimes causes bad trouble when it's used after a verb. Good should not be used after
most verbs, so avoid talks good, drives good, writes good, and so on.
Good, however, may be used after some verbs (called linking verbs), such as looks, smells, sounds,
feels, and all forms of to be. So it's perfectly correct to say, "That idea sounds good." If you're not sure
whether a verb is a linking verb, substitute a form of the verb to be in its place. If the sentence
keeps its basic meaning, the verb might well be a linking verb. For example,
The apple tastes good. = The apple is good.
(Replacing tastes with is, preserves the basic meaning. Therefore, tastes must be a linking verb.)
Other common linking verbs: appear, stay, remain, grow, and become.
Where good is not correct, use well-as in talks well, drives well, and writes well.

kind of, sort of, type of


The phrase kind of is informal except when it is used literally, as in
Backgammon is a kind of board game. (Not a kind of a board game)
Few knew that Harold was that kind of man. (Not that kind of a man)
Kind of is used with singular nouns, while kinds of is used with plural, as in
That kind of animal lives in the jungle. Those kinds of animals live in the jungle.
It's not proper to mix singular and plural words as in
Those kind of animals live in the jungle.
Everything noted about kind of also applies to sort of and tube of

Lie, Lay:
Lie cannot take an object. Lie tells what a person or thing does for himself or by Itself. The
principal parts of lie are:
lie (simple form] Jaffar lies on his bed for a few minutes after lunch every day..
lay (past form) Mani lay on the sofa all yesterday afternoon.
lain (past participle) The watch had lain, unnoticed. for several days before I found it.
lying (present participle) A man was lying injured in the street after the accident.

Lay must have an object. Lay tells "what a person or thing does for someone or something else. i.e
principal o, its '.f lay are
lay (simple form) You should lay the tiles very evenly.

Rise/Raise
Rise cannot take an object. Rise tells what someone or something does for himself or by itself. The
principal parts of rise are:
rise (simple form) The temperature rises sharply in the afternoon.
rose (past form) The sun rose at seven yesterday.
risen (past participle) Prices have risen a great deal lately.
rising (present participle) The baby's temperature is rising by the hour.

Raise must have an object. Raise tells what someone or something does for someone or
something else. The principal parts of raise are:
OBJ.
raise (simple form) Please raise the window a little.
OBJ
raised (past form) The Browns raised their children.
OBJ.
raised (past participle) The store has raised its prices.
raising (present participle) The new book is raising many
OBJ.
interesting questions.
Advice/Advise
Advise is .a verb.
VERB
The doctor advised her to quit smoking.
Advice is a noun.
NOUN
He pave me some good advice.

Differ From/Differ With


To differ from is "to be dissimilar." Children differ physically from adults.”
To differ With is "to disagree with."
I differ with you on this issue. {I disagree with you.)

Emigrate, Immigrate:
To emigrate means "to leave one country to live in another." In the early part of this century many
people emigrated from Europe. They went to live in the United States.

To immigrate means "to move to a new country."


In the early part of this century many people from Europe immigrated to the United States.

Farther, Further:
Farther means "to or at a more distant point in space." (actual distance)
We have to drive a few miles farther.
Further means "to or at a more distant point in time, degree, or quantity." (figurative distance)
Let us consider this problem further. (time)
We should do further research on this matter. (quantity)
Be careful riot to excite the children further. (degree)

Rule 29: M O D I F I E R S - F E W , L I T T L E , MUCH AND M ANY


Few, fewer, and fewest, as well as many, are followed by plural count nouns.

PL. CN

There are few students from Japan in our English class.

PL, CN
This year we received fewer replies to our ad than ever before.

PL. CN
Jahangir has the fewest chapters left to read of anyone ii the class.

PL. CN
There are many reasons to study hard for that test.

Little, less, and least, as well as much, are followed by non-count nouns.

NCN
He gave me a little advice about choosing a school.
NCN
Susan has less money than I.
NCN
He did the least amount of work of anyone in the class.
NCN
There is not much time to finish this job completely.
NOTES:
a. In general, plural count nouns can be recognized by the -s plural form. However, do not forget
that the following words are plural: people, men, women, children.
b. The following are examples of non-count nouns: fruit, homework, bread, money, furniture, and
time. Do not add -s to these
words.
c. The word news looks plural, but it is a non-count noun. Example: Little news is coming from that
country.
ERROR EXAMPLES:

WRONG: There are much books on the shelf.


RIGHT: There are many books on the shelf,

WRONG: There is not many industry in that town.


RIGHT: There is nut much industry in that town.

WRONG: He had few winter clothing when he arrived.


RIGHT: He had little winter clothing when he arrived.

WRONG: You need a little dollars to buy this book.


RIGHT. You need a few dollars to buy this book.

WRONG: Lloyd scored the least points in the baseball game.


RIGHT: Lloyd scored the fewest points in the basketball game.

Rule 30: FACTUAL CONDITIONS – Possible Results


Remember that although a past verb is used , the opinion is for future time. In order of most possible
to least possible, use the following modals: would, could, might.

Would
Could Verb
If S V (past) S Might word
If We Found Her We Would Write Her
address
If We Found We Could Write Her
Her
If We Found address We Might Write Her

Her
address
OR

Would
Could
S Might Verb word If S V (past)
We Would Write Her If We Found Her address

We Could Write Her If We Found Her address

We Might Write Her If We Found Her address


Avoid using would and a verb word instead of a past tense verb in an "if" clause.
EXAMPLES
INCORRECT: If Jim's family meet Karen, I am sure that they would like her.
CORRECT: If Jim's family met Karen, I am sure that they would like her.
or
I am sure that that would like her if Jim's family met Karen.
INCORRECT: If you made your bed in the morning, your room looks better when you got back in
the afternoon.
CORRECT: If you made your bed in the morning, your room would look better when you got back it the
afternoon.
or
Your room would look better when you got back in the afternoon, if you made your bed in the
morning.

INCORRECT: If Judy didn't drink so much coffee, she wouldn't have been so nervous.
CORRECT: If Judy didn’t drink so much coffee, she wouldn't be so nervous
or
Judy wouldn’t be so nervous if she didn't drink so much coffee.
INCORRECT: If you would go to bed earlier, you wouldn't be so sleepy in the morning.
CORRECT: If you went to bed earlier, you wouldn't be so sleepy in the morning.
or
You wouldn’t be so sleepy in the morning if you went to bed earlier.

INCORRECT: If she would eat fewer sweets, she would lose weight.
CORRECT: If she ate fewer sweets, she would lose weight.
or
She would lose weight if ate fewer sweets.

EXAMPLE 24:
Q.1: Choose the correct answer.
If Americans ate fewer foods with sugar and salt, their general health _______better.
(A) be (B) will be (C) is (D) would be

Q.2: Choose the incorrect word or phrase and correct it.


If drivers obeyed the speed limit, fewer accidents occur.
(A) (B) (C) (D)

Rule 31: FACTUAL CONDITIONALS-Probable Changes In Past Results


Remember that the speaker or writer is expressing an opinion about the results of the past
under different conditions or circumstances. In order of the most to the least probable, use
the following modals: would, could, might.

Would have
Could have
If S Had Participle S Should have Participle
If We Had Found Her We Would have Written Her
address
If We Had Found We Could have Written Her
Her
If We Had Found address We Should have Written Her

Her
address
Avoid using would have and a participle instead of had and a participle in the clause beginning
with if. Avoid using have as a participle.
EXAMPLES

INCORRECT: If we had the money, we would have bought a new stereo system.

CORRECT: if we had had the money, we would have bought a new stereo system.
or
We would have bought a new stereo system if we had had the money.

INCORRECT: If the neighbors hadn't quieted down, I would have have to call the police.
CORRECT: If the neighbors hadn't quieted dawn, I would have had to call the police.
or
I would have had to call the police if the neighbors hadn't quieted down.

INCORRECT: If his mother let her, Anis would have stayed longer.
CORRECT: Anis would have stayed longer if his mother had let her.
or
If her mother had let his, Anis would have stayed longer.

INCORRECT: If we would have known that he had planned to arrive today, we could have met her at
the bus station.
CORRECT: If we had known that he had planned to arrive today, we could have met her at the bus
station.
Or

We could have met her at the bus station if we had known that she had planned to arrive today.

INCORRECT: If I had more time, I would have checked my paper again.


CORRECT: If I had had more time, I would have checked my paper again. or
I would have checked my paper again if I had had more time.

EXAMPLE 25:
Q.1: Choose the correct answer.
According to some historians, if Napoleon had not invaded Russia, he _____________ the
rest of Europe.
(A) had conquered
(B) would conquer
(C) would have conquered
(D) conquered

Q.2: Choose the incorrect word or phrase and correct it.


If dinosaurs would have continued roaming the earth, man would have evolved quite differently.
(A) (B) (C) (D)

EXAMPLE 26:
If I would have know more about the person whom I was writing to, I would have
A B C
written a better answer. No error.
D E
Rule 32: CONTRARY-TO-FACT CONDITIONALS-Impossible Results Were
If S Were
If The party Were On Friday, we
could go

Avoid changing were to agree with the subject in contrary-to-fact statements.


EXAMPLES:
INCORRECT: If Barbara was really my friend, she would call me once in a while.
CORRECT: If Barbara were really my friend, she would call me once in a while.
(Barbara is not my friend.)
or
She would call me once in a while if Barbara were really my friend. (Barbara is not my friend.)

INCORRECT: If the meat was a little more done, this would be an excellent meal.
CORRECT: If the meat were a little more done, this would be an excellent meal. (The meat is not
done.)
or

This would be an excellent meal if the meat were a little more done. (The meat is not done.)

INCORRECT: If my daughter is here, I would be very happy.


CORRECT: If my_ daughter were here, I would be very happy. (My daughter is not here.)
or
I would be very happy if my daughter were here. (My daughter is not here.)

INCORRECT: This apartment be perfect if it were a little larger.


CORRECT: This apartment would be perfect if it were a little larger.
(The apartment is not larger.)

or
If it were a little larger, this apartment would be perfect. (The apartment is not larger.)

EXAMPLE 27:
Q.1: Choose the correct answer.
If humans were totally deprived of sleep, they hallucinations, anxiety, coma, and
eventually, death.
A) would experience
B) experience
C) would have experienced
D) had experienced

Q.2: Choose the incorrect word or phrase and correct it.


If we were to consider all of the different kinds of motion in discussing the movement of an
(A)
object, it i s very confusing, because even an object at rest is moving as the earth turns.
(B) (C) (D)
MISCELLANEOUS:
1. “Whether – that” can’t be used together. Either use whether or that.

i) He couldn’t decide whether that he might stay at hostel or in a private room.

2. “Had –Had” used together are usually correct.

i) If the engineer had-had competent assistants to help him finish the project he would
have completed it.

3. “Effect” means Result “Affect” means Influence.

i) I have found that a mild solution is more affective (effective) than the commercial
preparation available in drug stores in the treatment of this ailment.

4. “Than” mean introducing second member of comparison.


“Then” means at that time next or after word.

i) I have decided to give more importance to my studies then (than) other activities.
ii) It is better to go home then (than) to wait here.

7. Specialized in is correct and interested in is correct.


8. In spite of” is correct whereas “in spite” is wrong.

9. Double negatives can’t be used together like.


Seldom - never
Can’t - hardly
Has n’t - neither

10. All ready- means everybody or everything ready.


Already – means previously.
They were all ready to write when the teacher arrived.
They had already begun writing when the teacher arrived.
11. All together—means everybody or everything together.
Altogether—means completely.
The boy and girls sang all together. (correct)
This was altogether strange for a person of his type. (correct)
12. Beside –means close to.
Besides –refers to something that has been added.
He lived beside the stream. (correct)
He found wild flowers and weeds besides. (correct)
13. Principal—means chief or main (as an adjective):
a leader (as a noun)
Principle—means a fundamental truth or belief.
The teacher taught us the principals (principle) of algebra thoroughly.
His principal supporters came from among the peasants. (correct)
The principal of the school asked for cooperation from the staff. (correct)
Humility was the guiding principle of Buddha’s life (correct))
NOTE: Principal may also mean a sum placed at interest.
Part of his monthly payment was applied as interest on the principal.
15. For Non-livings we use “which” whereas for livings, we use “who”.
16. With superior we use “to” and not “than”
This computer is superior than (to) the old one.
Answers and Explanations of Examples:
Example 1: Q.1: D (change were to was) ; Q.2: Example 18: C
C (change lie to lies) (comparison should be
Example 2: A (sentence is correct) parallel)
Example 3: C (pronoun should be singular) Example 19: D
Example 4: E (comparison should be
Example 5: B (change make to makes) parallel)
Example 6: C Example 20: than
Example 7: Q.1: was ; Q.2: were ; Q.3: are ; Q.4: Example 21: Q.1: D
is (change any to any
Example 8: Q.1: is ; Q.2: are other) ; Q.2 : E
Example 9: B (ambiguous pronoun) Example 22: B
Example 10: B Example 23: A
Example 11: D (change I to me) (modifier should be
Example 12: D (change her to she) followed with a
Example 13: A (change you to your) subject)
Example 14: B (change your to his) EXAMPLE 24: Q.1: D ; Q.2: D (change occur to
Example 15: B (change whomever to would occur)
whoever) EXAMPLE 25: Q.1: C ; Q.2: A (change would
Example 16: B (change whomever to have to had)
whoever) EXAMPLE 26: A (change would have to had)
Example 17: Q.1: like ; EXAMPLE 27: Q.1: A ; Q.2: B (change is to
Q.2: as ; Q.3 A (change would be)
like to as)
Subject Verb Agreement
Directions: Underline the italicized verbs in the sentences that contain errors in subject-
verb agreement.
1. According to a noted meteorologist, there is various explanations for the accelerating rate of
global warming.
2. In this critically acclaimed film, there is a well-developed plot and an excellent cast of characters.
3. Through the locks of the Panama Canal passes more than fifty ships each day.
4. There are a number of state legislatures currently debating strict environmental laws.
5. If there is competing proposals, your idea may not be acted upon until next week.
6. There is at least five types of climbing rose and a unique variety of small fir in the Botanical
Gardens.
7. Despite numerous professed sightings, there is still no conclusive evidence of extraterrestrial
beings.
8. Multipurpose vehicles, which can be very useful on rough terrain, (is/are) now banned in many
states.
9. The level of chemicals and other air pollutants (is/are) now monitored in many offices.
10. The fundamental hitting skills of Aslam Khan (goes/go) largely unnoticed by the average fan.
11. A community as diverse as Karachi (attracts/attract) immigrants from many countries.
12. One-way tickets for domestic travel (is/are) often more expensive than round-trip fares.
13. So-called “bullet trains” from Tokyo to Osaka (completes/complete) the 300-mile trip in about
two hours.
14. Donations to the church-sponsored orphanage (is/are) up by 50 percent over last year.
15. Einstein’s theory of relativity (ranks/rank) with the most developed hypotheses involving space
and time.
16. The fishing industry, along with railroad safety issues, (is/are) of great concern to the state
assembly.
17. Either the manager or one of his coaches usually (removes/remove) a pitcher from the mound.
18. Both the word scuba and the word radar (is/are) acronyms.
19. Auto exhaust, in addition to industrial pollution, (is/are) a cause of smog in southern Punjab.
20. A group of students who had gathered outside the Union office (is/are) agitating violently.

Identify the error.

21. Bicycling, as well as walking and jogging, reduce one’s dependence on


A B C D
motorized transportation. No error
E
22. The recent establishment of “Crime Busters,” officially sanctioned neighborhood block-
A B
watching groups, have dramatically improved relations between citizens and police.
C D
No error.
E
23. The effort to create appropriate theatrical effects often result in settings that cannot be
A B
effective without an imaginative lighting crew. No error.
C D E
24. Every one of the shops in the town were closed on Thursday because of the ten-inch
A B C
rainfall that had fallen during the day. No error.
D E
25. The Zoning Improvement Plan, better known as zip codes, enable postal clerks
A B
to speed the routing of an ever-increasing volume of mail.
C D
26. The high protein content of various strains of alfalfa plants, along with the

characteristically long root system that enables them to survive long droughts,
A B
make them particularly valuable in arid countries.
C D
27. Furquanabad, Oudh, the capital of the state, are not only the largest city in Oudh but
A B C
also a typical metropolitan area, often used in market research.
D E
28. There is about 600 schools in the United States that use the Montessori method to
A B C D
encourage individual initiative.
E
29. Everyone who has traveled across the United States by car, train, or bus are
A B C
surprised to see such a large expanse of territory with such variation among the
D
life-styles of the people.

30. The governor’s aides are convinced that the announcement of the investigation, coming just days before
A B
the filing deadline, were calculated to discourage the governor from running for reelection. No error
C D E
31. Many professional athletes are motivated by either personal pride and love of their
A B
sport, but some seem interested only in money. No error
C D E
32. Also supported by the commission was the proposed health clinics and the proposed
A B
center to distribute information on job-training opportunities. No error
C D E
33. Neither of the scout leaders know how to trap wild animals or how to prepare them for hunting.
A B C D
34. Nearly all the editors of the magazine agree that of the two articles to be published, Fujimara’s
A B C
is the more exciting. No error
D E
35. The record left by fossils, the ancient remains of plants and animals, provide scientists with
A B
their primary source of information about prehistoric life. No error
C D E
36. The jury took offense at the prosecutor’s mocking tone but could deny neither the accuracy
A B C
of the charges or the seriousness of the crime. No error
D E
37. Available through the school’s guidance office is a job directory and a list of job referral
A
centers that provide information for students in need of employment. No error
B C D E
38. Studying the language and the culture of a foreign country is highly recommended to the
A B
tourist who expect to learn from his or her vacation abroad. No error
C D E
39. One of the most popular and attractive new cars were available for so little money that
people thought something was wrong.
(A) were available for so little money that people thought something was wrong.
(B) was available for so little money that people thought something were wrong.
(C) was available for so little money that people thought something has gone wrong.
(D) was available for so little money that people thought something was wrong.
(E) was selling for very little money, so people thought of it as something was wrong.

40. In scenarios reminiscent of the old science fiction movie “Fantastic Voyage”, medical
researchers hope exploring the body with miniature robots sent into the bloodstream.
(A) hope exploring
(B) hope to explore
(C) hope it can explore
(D) have hopes to explore
(E) are having hopes to explore
Answers Exercise 1: Answers exercise 2:
1. is 1. is
2. brings 2. is
3. is 3. is
4. aren’t 4. were
5. have 5. has
6. has 6. were
7. are 7. were
8. are 8. has
9. has 9. was
10. vary 10. has
11. makes
12. was
13. was
14. has
15. is
16. has
17. is
18. have

Answers (sub-verb agreement) subject requiring a singular verb,


reduces.
1. are 22. C: The subject of the sentence is
2. are establishment, which is singular; so the
3. pass main verb should be has, not have.
4. correct 23. B: The subject of this sentence is the
5. are singular noun effort. The main verb should
6. are therefore be results, rather than the plural
7. correct result.
8. are 24. A:Every one is singular. Were closed is
9. is plural: It should be corrected to was closed.
10. go 25. B (enables)
11. attracts 26. C (makes)
12. are 27. A (is)
13. complete 28. A (There are)
14. are 29. C is (is)
15. ranks 30. C (change were to was)
16. is 31. B (change and with nor)
17. removes 32. A (were)
18. are 33. C (nor)
19. is 34. E
20. is 35. A (provides)
21. B:Reduce is a plural form of the verb, to 36. D (nor)
reduce. But the subject of this verb is 37. A (are)
bicycling, a gerund, and a gerund is 38. C (expects)
always singular. The phrase between 39. D
commas, as well as walking and jogging, 40. B
may seem to make the subject plural.
But a phrase following a subject, set off
by commas, and introduced by a
compound preposition like as well as, or
in addition to, is not treated formation
PRONOUN EXERCISE:
DIRECTIONS: Put “C” if the sentence is correct. Put “I” if there is an error with the pronoun.

1. When one has many problems, he should try to solve them one at a time.
2. Mariam could never understand him wanting to be a nurse.
3. I often think back to the time when mutual friends introduced Paul and I.
4. Kamran noticed many people who had been waiting hours to buy their tickets.
5. When the children realized that they were by theirselves in the dark, they became really frightened.
6. In the course of life one should always remember their old friends.
7. Do you remember the teacher that his daughter became a doctor?
8. For them of you who wish to know more about journalism, we recommend that you order a book
from the following list.
9. Neither my aunt nor my cousins were able to explain their behavior.
10. Ahmed was surprised to realize that it was us, his old school friends, calling him from Paris.
11. The dean asked all the students, including Basheer and I to show our visitor every possible courtesy.
12. Give the refunds to those who have filled out the correct form.
13. They say that English can be a very difficult language for one to learn in his later years.
14. Modern society, including conservatives, liberals, hippies, and blacks, has many problems that they
must solve.
15. Hashim met the player who you admire so much.
16. Huzefa called to his old friend Umer as he walked across the campus.
17. In the paper it says it is going to rain today.
18. Did you ever see a man as tall as he?
19. Neither of the girls remembered to give I her notebook.
20. I am worried about your having to review so much material.

21- Each of the students in the accounting class has to type their own research paper this semester.
A B C D

22- Mr. Pasha used to think of hisself as the only president of the company.
A B C D

23-I certainly appreciate him telling us about the delay in delivering the materials because we had planned to
A B C D
begin work tomorrow.

24-The explanation that our instructor gave us was different than the one yours gave you.
A B C D

25-All the scouts got theirselves ready for the long camping trip by spending their weekends living in the open.
A B C D
26-The worker who I see in the subway every afternoon seems tired and dejected. No error
A B C D E
27-Mr. Shariq’s decision to retire came as a shock to all who respected his ability. No error
A B C D E

28-With the exception of Farhan and I, everyone in the class finished the assignment before the bell rang.
A) Farhan and I , everyone in the class finished
B) Farhan and me , everyone in the class finished
C) Farhan and me , everyone in the class had finished
D) Farhan and I , everyone in the class had finished
E) Farhan and me everyone in the class finished

29-The animals who were chosen to represent the Democratic and Republican parties, the donkey and the
A B
elephant, were created by the renowned cartoonist Tahir Naqsh. No error
C D E

30-I should like you and he to supply the necessary data for the annual statement that must be prepared in
A B C
advance of the spring meeting. No error
D E
Answer sheet of pronoun error
1- C
2- I (his wanting)
3- I (Paul and me)
4- C
5- I (by themselves)
6- I (one’s old friend)
7- I (teacher whose daughter)
8- I (For those of you)
9- C
10- I (it was we)
11- I (including Betty and me)
12- C
13- C
14- I (it must solve)
15- I (whom you admire)
16- I (As Bob walked across the campus, he called to his old friend Nasir.)
17- I (The paper says)
18- C
19- I (to give me)
20- C
21- C (should be his. Each is singular and must be followed by a singular verb and pronoun.)
22- B (should be himself. Hisself is not a word.)
23- B (should be his. Possessive forms must be used before gerund.)
24- C (should be from. Always use different from.)
25- B (should be themselves. The form theirselves does not exist.)
26- A (Error in pronoun case. The pronoun should be whom because it is the object of the verb see. )
27- E (Sentence is correct).
28- C (This corrects the two errors in this sentence the error in pronoun case (me for I) and the error in tense (had
finished for finished).
29- A (Misuse of relative pronoun. Change who (refers to people) to that (refers to things)).
30- B (Error in pronoun case. Change he to him.)
Grammar Practice Tests

To develop your ability to handle the questions on the Test of Standard Written English, do the following four
exercises. These exercises require you to spot errors in usage, grammar, diction, and style.
After completing each exercise, check your answers with the answer keys at the end of Exercise D.
Then, you know what to do: read those answer explanations!

EXERCISE A

1. If I was the principal I would declare Allama Iqbal’s birthday a school holiday and hold a
A B C
special assembly in this honor. No error
D E

2. Every one of the candidates for the position is submitting an application and three letters of
A B
recommendation attesting their fitness for the job. No error
C D E

3. In correcting your essay for class, be sure to watch out for misspellings, sentence fragments,
A B
dangling modifiers, and how you punctuate each sentence. No error
C D E

4. When the kitchen was remodeled last summer, neither the butcher block counter nor the
A B C
wood floor were refinished. No error.
D E

5. The headmaster suspended Jane for a week because she had cut several consecutive classes
A B C
and had insulted her senior teacher. No error
D E

6. Do you realize the affect this news will have on Jane when she reads about it? No error
A B C D E

7. The reason why I was late is because the subway train broke down in the tunnel and we had
A B C
to wait for more than an hour. No error
D E

8. I felt real sorry for her, and, therefore, I let her go with only a warning. No error
A B C D E

9. I hope you will vote for whoever is the best candidate for the position. No error
A B C D E

10. John practiced intensively and as a result is now better than any member of his item.
A B C D
No error
E
11. If Richard would have felt any pity for his opponent, he would have ended the bout sooner
A B C D
than he did. No error
E
12. Complaining about the lack of police protection in the neighborhood was the minister of the
A B C
church and several parents of children attending the local school. No error
D E

13. By the time the fire engines arrived at the scene, the Janitor extinguished the blaze and the
A B C
tenants were returning to their homes. No error
D E

14. This purse is the most loveliest of all the gifts I received for my birthday. No error
A B C D E

15. Since the doctor’s prescribing the incorrect medication led to the patent’s untimely death.
A B C D
No error
E
EXERCISE B

1. Mary is jealous of people like you and I and shows her resentment in many different and
A B C
subtle ways. No error.
D E

2. The Mayor of San Francisco, along with many other dignitaries are going to attend a festive
A B C D
dinner in honor of the queen’s visit. No error.
E

3. Irritated by the criticism leveled at him in the press, the commissioner decided to resign
A B C
from the post which he had held for three year. No error.
D E

4. The other members of the committee are not at fault; it is I who is responsible for the failure
A B C D
of our enterprise. No error.
E

5. I shop at the supermarket because it is cheaper and having a greater variety of foods than the
A B C D
local grocery store. No error.
E

6. As the spectators ran for shelter from the sudden thunderstorm which was drenching the
A B C D
stadium. No error.
E

7. Since Jones has made more spectacular plays than any other player on the field, he should
A B C D
be given the Most Valuable Player award. No error.
E

8. Mr. Baxter praised his employee and said that he would have reacted in a similar manner if
A B
he was faced with a similar situation. No error.
C D E
9. If a rudimentary knowledge of word processing, stenography, and clerical systems are not
A B C
enough for today’s secretary, then what must she know? No error
D E

10. Although the campers had brought an ample supply of freeze-dried food with them, they
A B
preferred to eat the fish they catched in the nearby stream. No error
C D E

11. If I had known your address, I would of written to you or phoned you or even visited you.
A B C D
No error.
E

12. In this modern age, many people find talking on the phone to their friends more satisfying
A B C
than to write. No error.
D E

13. One of the many factors which are influencing my decision is the element of cost. No error.
A B C D E

14. I do not mind John’s telling that story to my friends; I resent him relating it to the people
A B C
who dislike me. No error
D E

15. Neither-the pupils nor the teacher is happy about the cancellation of the annual festive
A B C D
Christmas part. No error.
E

EXERCISE C

1. As part of the campaign to raise money for their camping trip, the Boy Scouts in our
A B
community collected aluminum cans and brung them to the redemption center. No error.
C D E

2. I was irritated by the special relationship between you and him; I thought you were
A B C D
playing favorites. No error.
E

3. Now that you are using this skin lotion your complexion has become clearer and you look
A B C
more lovelier. No error.
D E
4. The trip the Joneses are planning includes golfing in Bermuda, fishing in the Gulf Stream,
A B
and to snorkel in the waters off Nassau. No error.
C D E
5. In his famous soliloquy, Hamlet thinks of death as a means of being rid from all worldly
A B C D
cares. No error.
E

6. Not one of the other contestants have the vitality that Frances possesses; I am confident that
A B C
she will win the contest. No error.
D E

7. That a medal would be given to whomever had the best attendance record for the school year
A B
was announced by Mrs. Mary Fitzpatrick, principal of the school. No error.
C D E

8. Our team of debaters have proved itself the best in the division, having won all its matches
A B C D
this year. No error.
E

9. I had difficulty deciding between Carter and Reagan, but I finally voted for the latter.
A B C D
No error.
E
10. While Nero fiddled and Rome burned as we have so often been told. No error
A B C D E

11. All that time, it was her whom I held responsible for the calamity which had occurred to
A B C D
our family. No error.
E

12. John was living in his apartment house for two years before he met his next-door
A B C D
neighbor. No error.
E
13. Every pupil in the class except her and Julius has handed in the required book report and
A B C
is ready for the class discussion. No error.
D E

14. Because your mother needs you to go to the store, you had ought to go home before
A B C
going to baseball practice. No error.
D E

15. I have prepared for this examination for more than two weeks, however I still feel
A B C
nervous about this test. No error.
D E
EXERCISE D

Direction: In each of the following sentences, some part or all of the sentence is underlined. Below each
sentence you will find five ways of phrasing the underlined part. Select the answer that produces the most
effective sentence, one that is clear and exact, without awkwardness or ambiguity, and blacken the corresponding
space on your answer sheet. In choosing answers, follow the requirements of standard written English. Choose the
answer that best expresses the meaning of the original sentence.

Answer (A) is always the same as the underlined part. Choose answer (A) if you think the original sentence needs
no revision.

1. Although it was obvious that James felt badly about the affect of his impulsive words, we decided that further
punishment was in order.
(a) felt badly about the affect
(b) felt badly about the effect
(c) seemed sorry about the affect
(d) felt bad about the effect.
(e) felt bad about the affect.

2. Beethoven wrote nine symphonies; and they are still being played by orchestras all over the world.
(a) symphonies; and they are still
(b) symphonies which are still
(c) symphonies; And they are still
(d) symphonies, and they are still
(e) symphonies; which are still

3. Attempting to fill out the application ,the ink smeared and he had to rewrite the entire form.
(a) The ink smeared and he had to rewrite the entire form
(b) he smeared the ink and had to rewrite the entire form
(c) the ink smeared and the entire form had to be rewritten
(d) he smeared the ink and the entire form had to be rewritten
(e) the ink smeared so he had to rewrite the entire form

4. If the actors would not have required so much rehearsal time, we could have finished this film on schedule.
(a) If the actors would not have required so much rehearsal time
(b) If so much rehearsal time would not have been needed
(c) Had the actors not needed so much rehearsal time
(d) If the actors did not require so much rehearsal time
(e) Since the actors required so much rehearsal time

5. Having rang the bell, the class came to order.


(a) Having rang the bell, the class came to order
(b) The bell having rang, the class came to order
(c) Ringing the bell, the class came to order
(d) With the bell having rang the class came to order
(e) The bell having rung, the class came to order.

6. Stating that he couldn’t eat hardly anything, John asked permission to leave the table.
(a) he couldn’t eat hardly anything
(b) hardly anything could be eaten
(c) he could eat hardly anything
(d) he couldn’t eat hardly nothing
(e) he couldn’t eat scarcely anything
7. Walking on this slippery surface, one must use care to avoid accidents.
(a) one must use care to avoid accidents
(b) care must be used to avoid accidents
(c) accidents can be avoided by using care
(d) you must avoid accidents by using care
(e) requires care to avoid accidents

8. One gets more of a thrill out of watching a boxing match at the arena than out of seeing it on television
because you can feel the excitement, of the crowd.
(a) because you can feel the excitement of the crowd
(b) since you can feel the crowd’s excitement
(c) as you can sense the crowd’s excitement
(d) because he will feel the excitement of the crowd
(e) since he can feel the excitement of the crowd

9. We expected the plane to be late, due to the announcement that heavy fogs had delayed its takeoff.
(a) We expected the plane to be late, due to
(b) We expected the plane will be late because of
(c) We expected the plane would be late due to
(d) We anticipated that the plane would be late, due to
(e) We expected the plane to be late because of

10. The packages have been wrapped and are all ready to be mailed as soon as the courier arrives.
(a) Have been wrapped and are all ready to be mailed
(b) Are wrapped and are already to be mailed
(c) Have been wrapped and are already for mailing
(d) Are being wrapped and will be already for mailing.
(e) Are being wrapped and will already to mail

11. Inflation to some a source of increased profits, to others on fixed incomes a source of diminishing wealth.
(a) Inflation to some a source of increased profits, to others on fixed incomes
(b) A source of increased profits to some; to others inflation on fixed incomes
(c) Although some find inflation a source of increased profits, to others on fixed incomes, it is
(d) To some, inflation is a source of increased profits; to others on fixed income they are
(e) Although inflation is to some a source of increased profits, to others on fixed incomes

12. Since the polls have just closed, it is too early to predict who the winner will be.
(a) Since the polls have just closed, it is too early to predict who the winner will be
(b) Due to the fact that the polls have just closed, it is too early to make predictions about the winner.
(c) The polls have just closed, it is too early to predict the winner
(d) Because the polls have just closed, it is too early to predict whom the winner will be
(e) The polls having just closed, it is too early to predict whom the winner will be

13. First you mix the batter and then the cake is baked.
(a) First you mix the batter and then the cake is jacked
(b) First you mix the batter and then you bake the cake
(c) The first step is to mix the batter and then the cake is baked
(d) First the batter is mixed and then you bake the cake
(e) Mix the batter first before the cake is backed

14. We couldn’t of finished without you’re help.


(a) We couldn’t of finished without you’re help
(b) We could not have finished without your help.
(c) We could not of finished without your helping
(d) We couldn’t have finish without you’re help
(e) We cannot finish without your helping
15. I have never and never will ask for special treatment.
(a) I have never and never will ask for special treatment
(b) I have never asked and never will ask for special treatment
(c) Special treatment is what I never asked and never will
(d) I have never asked four special treatments. I never will
(e) For special treatment I have never and never will ask

Answers to Grammar Practice Exercises


EXERCISE A

1. A 4. D 7.B 10.D 13. C


2. C 5. E 8.A 11. A 14. A
3. D 6. B 9.E 12.C 15. A

EXERCISE B

1. B 4. C 7.E 10.D 13. E


2. A 5. B 8.C 11.B 14. C
3. E 6. A 9.C 12.D 15. E

EXERCISE C

1. D 4. C 7.A 10.A 13. E


2. E 5. C 8.A 11. A 14. C
3. D 6. B 9.E 12.A 15. C

EXERCISE D

1. D 4. C 7.A 10.A 13.B


2. B 5. E 8.E 11.C 14.B
3. B 6. C 9.E 12.A 15.B
Answer Explanations
EXERCISE A

1. (A) Error in mood A condition contrary to fact statement require the subjunctive mood. Change was to
were
2. (C) Error in agreement between pronoun and antecedent. Change their to his or her.
3. (D) Error in parallel structure. Change how you punctuate to the punctuation of.
4. (D) Error in agreement. In a neither . . . nor construction the verb agrees with the noun or pronoun which
precedes it. Change were refinished to was refinished.
5. (E) There is no error in this sentence.
6. (B) Error in diction. Change affect to effect.
7. (B) Grammarians dislike the expression the reason is because. Change because to that.
8. (A) Misuse of adjective for adverb. Change real to really.
9. (E) There is no error in this sentence.
10. (D) Error in comparison. Change any member to any other member.
11. (A) Error in mood. The subjunctive mood is needed in a condition contrary to fact statement. Change
would have-felt to had felt.
12. (C) Error in agreement between subject and verb. In this sentence we have a compound subject-minister
and paresis. Change was to were.
13. (C) Error in sequence of tenses. Change extinguished to had extinguished.
14. (A) Error in comparison. Change most loveliest to either most lovely or loveliest.
15. (A) The entire group of words is a sentence fragment. The deletion of the conjunction since will make this
a good sentence.

EXERCISE B

1. (B) Error in case. Like is a preposition and should be followed by a pronoun in the objective case. Change I
to me.
2. (A) Error in agreement between subject and verb. In the sentence which appears in the exercise, the
subject (mayor) is singular and the verb (are going) is plural. Since are is not underlined, we cannot
change it to is. We must, therefore, make the subject plural. By changing along with to and, we get a
compound subject which is plural.
3. (E) There is no error in this sentence.
4. (C) Error in agreement between pronoun and verb. Who in this sentence is a first person singular pronoun
because its antecedent is I. the verb should also be first person singular. Change is to am.
5. (B) Lack of parallel structure. Change having to has.
6. (A) This is a sentence fragment. Remove As.
7. (E) There is no error in this sentence.
8. (C) Error in mood. A condition contrary to fact statement should be in the subjunctive mood. Change was
faced to were faced.
9. (C) Error in agreement between subject (knowledge) and verb (are). Change are to is.
10. (D) Improper conjugation .change catched to caught.
11. (B) Error in diction. Change would of known to would have known.
12. (D) Error in parallel structure. Change to write to writing.
13. (E) There is no error in this sentence.
14. (C) Error in case. The pronoun preceding a gerund (relating) should be in the possessive case. Change him
to his.
15. (E) There is no error in this sentence.
EXERCISE C

1. (D) Error in conjugation. The past tense of bring is brought.


2. (E) There is no error in this sentence.
3. (D) Error in comparison of adjectives. Change lovelier to lovely.
4. (C) Error in parallel structure. Change to snorkel to snorkeling.
5. (C) Error in idiom. Change from to of.
6. (B) Error in agreement. Change have to has.
7. (A) Error in case. The subject of had should be in the nominative case. Change whomever to whoever.
8. (A) A collective noun can be regarded as either singular or plural. However, the use of the words itself and
its in this sentence indicates that the author regards team as singular. Change have proved to has proved.
9. (E) There is no error in this sentence.
10. (A) The sentence fragment can be corrected by the removal of while.
11. (A) Error in case. The predicate complement of a copulative verb (was) is in the nominative case. Change
her to she.
12. (A) Error in tense. Change was living to had been living.
13. (E) There is no error in this sentence.
14. (C) Error in diction. Change had ought to ought
15. (C) This is a run-on sentence. Change weeks, however to weeks; however.

EXERCISE D

1. (D) Copulative verbs like feel should be followed by an adjective. Affect should not be used as a noun. Both
errors are corrected in choice D.
2. (B) Choice B corrects the errors in punctuation found in the other choices.
3. (B) choices A, C, and E contain a dangling participle. Choice D has an unnecessary shift from the active
voice to the passive voice.
4. (C) Choices A, B, and D do not use the subjunctive mood to indicate a condition contrary to fact statement.
Choice E changes the meaning of the sentence.
5. (E) Choice A and C have a dangling participle; choices B and D contain an error in conjugation (rang
instead of rung).
6. (C) The double negative found in choices A, D, and E is corrected in choice C. Choice B changes the
meaning of the sentence.
7. (A) Choices B and C create a dangling participle. Choice D changes the meaning of the sentence. The
comma makes choice E incorrect.
8. (E) The shift from the third person pronoun to the second person (one to you) to choices A, B, and C is
corrected in choice E. Choice D changes the meaning of the sentence.
9. (E) Choices A, C, and D suffer from the misuse of due to. Choice B contains an error in tense.
10. (A) Already is misused in choice, B, C, D, and E.
11. (C) Choice A, B, and E result in sentence fragments. Choice D contains an error in agreement.
12. (A) Choice B misuses due to. Choice C is a run-on sentence. Choices D and E contain an error in case
(whom instead of who)
13. (B) Choices A, C, D and E contain an unnecessary shift from the active voice to the passive vice.
14. (B) Choice A, C, D, and E contain either or both of the errors in diction found in this question: couldn’t of
instead of couldn’t have and you’re instead of your.
15. (B) Choice A, C, D, and E suffer from the ellipsis (omission) of an important word.
ADVANCE GRAMMAR PRACTICE TEST NO. 1
50 QUESTIONS – 30 MINS
1. The man, as well as his three brothers, were attending his father’s funeral. No. error.
A B C D E
2. Neither Harry nor his friends was able to attend the first meeting of the club. No error.
A B C D E
3. All of the doctors who were at the meeting was concerned about the issue of malpractice. No error.
A B C D E
4. I know that others besides Fred has been irritated by the click of the door. No error
A B C D E
5. Hoping to achieve excellence, the football team practice every day. No error.
A B C D E
6. Us gardeners are never given as much as we deserve. No error.
A B C D E
7. Alex and me want to borrow my mother’s car. No error
A B C D E
8. He was told by his father to give the horse it’s oats. No error.
A B C D E
9. The band of robbers objected to him taking most of the gold. No error.
A B C D E
10. The person who would have to take out the garbage turned out to be I. No error
A B C D E
11. If Jack was at the party, we would be having a much better time. No error.
A B C D E
12. Walking down the quiet street, the traffic light turned red. No error.
A B C D E
13. He told me that Edwin is the kinder of the three. No error.
A B C D E
14. The boy sitting at that desk is smarter than any student in the class. No error.
A B C D E
15. In the summer, I always enjoy swimming and to jog. No error.
A B C D E
16. When Gus finally arrived at the store, the owner sells him a watch. No error.
A B C D E
17. Mrs. Pinkerton went to her present doctor for the last three years. No error.
A B C D E
18. The weatherman says that he agrees with the proposal the station made him. No error.
A B C D E
19. Their teacher says that he is as impertinent or more impertinent than his brother. No error.
A B C D E
20. After skiing down the hill at a steep angle, the snow became very wet. No error.
A B C D E
21. If I was you, I’d go to the game. No error.
A B C D E
22. Although both roads lead to New York City, take the longest because it is smoother. No error.
A B C D E
23. Walking down the road, my father saw both Barbara and she. No error.
A B C D E
24. While reading the directions with his friend’s lighter, the car rolled off the cliff. No error.
A B C D E
25. The adults did not wish to go with you and I. No error.
A B C D E
26. The other team had five more hits than us. No error.
A B C D E
27. The appraiser told me that fifty dollar were too much to pay for the ring. No error
A B C D E
28. Every one of the dozen eggs that he had purchased were broken on the bumpy ride home. No error.
A B C D E
29. It is I who is to be appointed acting chairman. No error.
A B C D E
30. There is several practical ways to go from our house to theirs. No error.
A B C D E
31. The old man cried that he had got nothing for his years of hard labor.
(a) he had got nothing (b)’’he had got nothing (c) ‘’he had gotten nothing
(d) he had gotten nothing (e) he had got no thing
32. He gave the toy to one of the boys who have been playing on the nursery floor.
(a) who have (b) who has (c) whom has (d) whom have (e) which have

33. Because the track was muddy, neither of the horses ran it’s best race.
(a) it’s best race (b) their best race (c) its best race
(d) their best races (e) its better race

34. Each of the candidates shakes the hand of whomever he meets.


(a) shakes the hand of whomever he meets (b) shake the hand of whomever he meets
(c) shakes the hand of whoever he meets (d) shakes the hand of whomever they meet
(e) shaked the hand of whomever he meets

35. Between you and I, Harvey’s an idiot.


(a) between you and I, Harvey’s. (b) between you and me, Harvey’s (c) between you and I, Harvey’s.
(d) among you and I, Harvey’s (e) among you and me, Harvey’s.

36. To who is the envelope addressed?


(a) To who is the envelope (b) To who is the envelop (c) To whom is the envelope
(d) To whom is the envelop (e) To whom are the envelope

37. He told me that his father never had liked Martin and she.
(a) his father never had like Martin and she. (b) “his father never had like Martin and she.”
(c) his father had never liked Martin and her.. (d) ‘’his father had never liked Martin and she.
(e) ‘’his father had never liked Martin and her.:

38. If my brothers friend was smarter, he would be easier to be with.


(a) brothers friend was smarter, he. (b) brother’s friend was smarter, he.
(c) brother’s friend were smarter, he. (d) brother’s friend were smarter, he.
(e) if my brothers friend were smarter, he

39. Realizing the pressure of the situation, a home run was hit by the batter.
(a) a home run was hit by the batter. (b) a home run had to be hit by the batter.
(c) a home run should be hit by the batter. (d) the batter hit a home run.
(e) a home run was to be hit by the batter.

40. To use this tool properly, this cap must be screwed on correctly.
(a) this cap must be screwed on correctly (b) this cap must be screwed on correct.
(c) this cap is to be screwed on correctly. (d) these cap must be screwed on correctly.
(e) you must screw on this cap correct.
41. He might have been smarter than any other student in the country
(a) might have been smarter than any other. (b) might have been smarter than any other.
(c) might have been smarter than any. (d) might have been more smarter than any other.
(e) might have been smarter than any other.

42. He told us that he neither enjoys boating or fishing.


(a) neither enjoys boating or fishing. (b) enjoys neither boating or fishing.
(c) neither enjoys boating nor fishing. (d) enjoys neither boating nor fishing.
(e) neither enjoys to boat or fishing.

43. When I arrived at the meeting, you are already there.


(a) you are already there. (b) you are all ready there. (c) you were already there.
d) you were all ready there. (e) you was already there.

44. Being as you are my brother, I will help you.


(a) Being as you are my brother, I. (b) Being as you are mine brother, I. (c) Being as you are my brother I.
(d) Since you are my brother, I. (e) Since you is my brother, I.

45. Will every one please sign his name.


(a) every one please sign his. (b) every one please sign their. (c) every one please sign his.
d) every one please signs his. (e) everyone please sign their.

46. Herman and myself were asked to clean up the room and to remove the garbage.
(a) myself were asked to clean up the room and to remove the garbage.
(b) myself was asked to clean up the room and to remove the garbage.
(c) I am asked to clean up the room and to remove the garbage.
(d) I were asked to clean up the room and to remove the garbage.
(e) myself were asked to clean up the room and that we should remove the garbage.

47. That was the sort of a look that strikes fear into men’s hearts
(a) sort of a look that strikes fear into men’s (b) sort of look which strikes fear into men’s
(c) sort of a look that strikes fear into men’s (d) sort of look who strikes fear into men’s
(e) sort of a look who strikes fear into men’s

48. His teacher told him, ‘’You has come a long way, but you still have a long ways to go.’’
(a) has come a long ways, but you still have a long ways to go”
(b) have come a long ways, but you still have a long ways to go.”
(c) has come a long ways, but you still have a long ways to go.”
(d) have come a long way, but you still have a long ways to go.”
(e) have come a long way but you still have a long ways to go.”

49. Tom doesn’t want to go; therefore, lets stay at home.


(a) go; therefore, let’s stay at home. (b) go, therefore, lets stay at home.
(c) go: therefore, let’s stay at home. (d) go, therefore, let’s stay at home.
(e) go;. therefore, let’s stay at home.

50. Jogging down the street, he ran right between the two ladies.
(a) right between the two. (b) write between the two.
(c) right among the two. (d) write among the two.
(e) right between the too.
ADVANCE GRAMMAR PRACTICE TEST 2.
50 QUESTIONS – 30 MINS
1. A bag of potato chips were always present when the family dined on hot dogs or hamburgers. No error.
A B C D E
2. The boss would like to see anyone who are here. No error.
A B C D E
3. He gave the shovel to one of the men who was struggling to uproot the big oak tree. No error.
A B C D E
4. Henry or you has been appointed to serve as the disinterested judge in this case. No error.
A B C D E
5. One must tell yourself not to lose sight of the line in the center of the road. No error.
A B C D E
6. Twenty-five feet are the entire length of the smiths lawn. No error.
A B C D E
7. Alex and me want to borrow my mother’s car. No error.
A B C D E
8. Carry this package to whoever you happen to see there. No error.
A B C D E
9. Marty always wanted to buy a house of his own. No error.
A B C D E
10. Because of the beat, Mrs. Maaz decided against him doing all of the work. No error.
A B C D E
11. Between you and I, Hector should never have been allowed to go alone. No error.
A B C D E
12. Looking at each of us, Sam asked for who this joke was intended. No error.
A B C D E
13. If he was my friend, I would be very happy. No error.
A B C D E
14. Realizing that it was too late, the car was turned around. No error.
A B C D E
15. To make sure that no robbers could enter the house, the doors were locked and bolted. No error.
A B C D E
16. This painting is, in my opinion, more lovelier than the other one. No error.
A B C D E
17. The drummer played more than any other person in the band. No error
A B C D E
18. This man was the most unique person that I have ever met. No error.
A B C D E
19. Farhan decided that he either wants to go now or to wait until next week. No error.
A B C D E
20. The leader of the vast army performed this task eagerly and with speed. No error.
A B C D E
21. As soon as Jerry drove to the hospital, his wife has a baby. No error.
A B C D E
22. Chris’ family stayed in their present house in the country every summer for six years. No error.
A B C D E
23. Even if we had asked, sally would not have come here anyways. No error.
A B C D E
24. Miles hadn’t hardly enough time to finish his work. No error.
A B C D E
25. She could best describe Rick’s dwelling as a sort of a shack. No error.
A B C D E

26. I like him more than any other friend.


(a) him more than any other. (b) him more than any
(c) he more than any (d) him more than any other
(e) him more then any.

27. The robber, as well as his three cohorts, are going to be put on trial.
(a) his three cohorts, are. (b) his 3 cohorts, are.
(c) his 3 cohorts, is. (d) its three cohorts, is.
(e) his three cohorts, is.

28. Three is several deserts on the menu.


(a) There is several deserts. (b) There is several desserts
(c) There are several deserts. (d) There are several desserts.
(e) There was several deserts.

29. Us doctors must be honest; furthermore, we must set an example.


(a) Us doctors must be honest; furthermore, we. (b) We doctor must be honest; furthermore, we.
(c) Us doctors must be honest, furthermore, we. (d) Us doctors must be honest: furthermore, we.
(e) We doctors must be honest, furthermore, we.

30. If he were here, you’re mother would feel much better.


(a) were here, you’re. (b) was here, you’re.
(c) would be here, you’re. (d) were here, your.
(e) was here, you’re.

31. I do not ride horses as much as him.


(a) horses as much as him. (b) horses as much as he.
(c) hoarses as much as he. (d) hoarses as much as him.
(e) horses as much as his.

32. If I would have been at home, the robbers could not have entered.
(a) would have been at home, the robbers (b) would be at home, the robbers.
(c) have been at home, the robbers.. (d) had been at home, the robbers.
(e) had been at home, the robbers.

33. To be able to graduate, you must first complete your senior year.
(a) you must first complete your senior year. (b) you must first complete you’re senior year.
(c) you senior year must first be completed. (d) you’re senior year must first be completed.
(e) you must first complete you’re senior year.

34. Our British guest though that our house was much more lovelier than his.
(a) British guest thought that our house was much more lovelier than.
(b) British guest thought that our house was much lovelier than.
(c) british guest thought that our house was much more lovely then.
(d) british guest though that our house was much more lovelier than.
(e) british guest though that our house was much lovelier than.
35. Horace Greeley uttered the words with eloquence and clearly.
(a) uttered the words with eloquence and clearly.
(b) utter the words with eloquence and clearly.
(c) uttered the words with eloquence and with clearly.
(d) uttered the words eloquently and with clarity.
(e) uttered the words with eloquence and clearness.

Directions: In each of the following examples, there are five variations of the same sentence. Select the best
version and blacken the corresponding space on your answer sheet.

36. (A) My mother pointed to the gift on the table which she had bought.
(B) My mother is pointed to the gift on the table which she had bought.
(C) My mother pointed to the gift she had bought, which was on the table.
(D) My mother pointed to the gift in the table which she had bought.

37. (A) ‘’I know,’’ she cried, ‘’that Abe is braver than any man’’!
(B) ‘’I know,’’ she cried, ‘’that Abe is more braver than any man’’!
(C) ‘’I know,’’ she cried, ‘’that Abe is more braver than any other man!’’
(D) ‘’I know,’’ she cried, ‘’that Abe is braver than any man!’’
(E) ‘’I know,’’ she cried, ‘’that Abe is braver than any other man!’’

38. (A) Alton B. Parker, who was a great man, addressed the crowd with candor and wittily.
(B) Alton B. Parker, whom was a great man, addressed the crowd with candor and wittily.
(C) Alton B. Parker, who were a great man, addressed the crowd with candor and with wit.
(D) Alton B. Parker, who was a great man, addressed the crowd with candor and with wit.
(E) Alton B. Parker, whom was a great man addressed the crowd with candor and with wit.

39. (A) The man said that he has went to his present doctor for two years.
(B) The man said that he has went to his present doctor for too years.
(C) The man said that he went to his present doctor for two years.
(D) The man said that he has gone to his present doctor for two years.
(E) The man said that he has gone to his present doctor for too years.

40. (A) They decided to buy a car, and one was soon chosen.
(B) They decided to buy a car, and one was soon chosen.
(C) They decided to buy a car, and one was soon chosen.
(D) They decided to buy a car and soon chose one.
(E) They decided and buy a car, and one was soon chosen.

41. (A) Walking to work, uncle Dick found a box of money.


(B) Walking to work, uncle Dick founded a box of money.
(C) Walking to work, a box of money was found by uncle Dick.
(D) Walking to work, a box of money was found by uncle Dick.
(E) Walking to work, uncle Dick found a box of money.

42. (A) If he was here, he would settle this dispute among the two of us.
(B) If he was here, he could settle this dispute between the two of us.
(C) If he were here, he could settle this dispute among the two of us
(D) If he was here, he could settle this dispute between the two of us
(E) If he was here, he could settle this dispute between the too of us.
43. (A) Elaine said that she had told everyone except you and I.
(B) Elaine said that she has told everyone except you and I.
(C) Elaine said that she had told everyone except you and I.
(D) Elaine said that she had told everyone except you and me.
(E) Elaine said that she had told everyone except you myself.

44. (A) Bill admitted that he has less money than I.


(B) Bill admitted that he has fewer money than I
(C) Bill admitted that he has less money than me.
(D) Bill admitted that he has less money than I
(E) Bill admitted that he has fewer money than me

45. (A) How come you used to could do your own cooking?
(B) How come you used to be able to do your own cooking?
(C) How come you used to be able to do you’re own cooking?
(D) How come you used to could do you’re own cooking?
(E) How come you used to be able to do you’re own cooking?

46. (A) They cut down the tree because of it’s swaying.
(B) They cut down the tree because of it swaying.
(C) They cut down the tree because of its swaying.
(D) They cut down the tree due to it’s swaying.
(E) They cut down the tree due to it swaying.

47. (A) Each of the players are going to accept their awards.
(B) Each of the players is going to accept their awards.
(C) Each of the players are going to accept his awards.
(D) Each of the players are going to accept they’re awards.
(E) Each of the players are going to accept his award.

48. (A) Neither of the boys was going to find their money.
(B) Neither of the boys was going to find his money.
(C) Neither of the boys was going to find their money.
(D) Neither of the boys was going to find found their money.
(E) Neither of the boys was going to find his money.

49. (A) The Russians, they are coming.


(B) The Russians they are coming.
(C) The Russians, they are coming.
(D) The Russians, are coming.
(E) The Russians, is coming.

50. (A) Clarence or you has to find the man who escaped.
(B) Clarence or you have to find the man who escaped.
(C) Clarence or you has to find the man whom escaped.
(D) Clarence or you has to find the man that escaped.
(E) Clarence or you have to find the man whom escaped.
ADVANCE GRAMMAR PRACTICE TEST 3
50 QUESTIONS – 30 MINS
1. A group of prisoners have just been driven to the jail, which is in the center of town. No error.
A B C D E
2. Either Felix or I are going to drive to the airport to meet my mother’s friend. No error.
AB C D E
3. The clothes that he wore was always too small. No error.
A B C D E
4. Will you make this effort for Mary and she? No error.
A B C D E
5. They thought that the man who stole the car was him. No error.
A B C D E
6. The host gave the glass of wine to Mr. Jones, whom quickly drank it. No error.
A B C D E
7. The witness claimed that some of the bullets had pierced the front door of his sporting goods store.
A B C D
No error.
E
8. We could see that the group of soldiers was upset when the strangers took they’re seats. No error.
A B C D E
9. No one but David or we could drive the company’s truck. No error.
A B C D E
10. Seeing the enemy of our family, my feelings turned from happy to sad. No error.
A B C D E
11. The young campers are more calmer now that the storm has passed. No error.
A B C D E
12. Alice is as abrasive or more abrasive than her friend. No error.
A B C D E
13. Dick told his mother that he neither enjoys running nor playing tennis. No error.
A B C D E
14. My uncle’s friend likes to play golf and sailing. No error.
A B C D E
15. When Mr. Smith finally approached the house, the young woman walks right by him. No error.
A B C D E
16. His son, who was here last Christmas, attended his present collage for two years. No error.
A B C D E
17. The leader assured his coops that everything would be alright. No error.
A B C D E
18. Because his employees quit, Mr. Jones will have to do all of the work himself. No error.
A B C D E
19. Be sure and turn off the light before you leave the house. No error.
A B C D E
20. The foreman of the construction workers lives in an suburb of New York. No error.
A B C D E
21. Were very happy to see that your family could come. No error.
A B C D E
22. Barry said that he would like to come, but he must spend the day in New Jersey. No error.
A B C D E
23. My teacher asked, “What is the answer to this problem” ?No error.
A B C D E
24. The teacher told John’s mother that he had always been good in English. No error.
A B C D E
25. Hank told me that he hadn’t heard from cousin Moe for three years. No error.
A B C D E
26. The man selling fruit told us that a bag of peaches were still a good buy.
(a) told us that a bag of peaches were. (b) told us that a bag of peaches was.
(c) told us that a bag of peaches were. (d) told us that a bag of peaches was.
(e) told us concerning that a bag of peaches were.

27. The group of men, they is waiting in the rain.


(a) The group of men, they is waiting (b) The group of men, they are waiting
(c) The group of men is waiting (d) The group of men, it is waiting
(e) The group of men, is waiting

28. You must always think that you are right.


(a) always think that you are right. (b) all ways think that you are right.
(c) always think that one is right. (d) always think that you are write.
(e) all ways think that one is right.

29. They approved of the Company’s giving all it’s employees a long vacation.
(a) company’s giving all it’s. (b) company’s giving all its.
(c) company giving all it’s. (d) company giving all its.
(e) company’s giving all its.

30. The terrible accident did not injure me as much as he.


(a) injure me as much as he. (b) injured me as much as he.
(c) injure I as much as he. (d) injure I as much as him.
(e) injure me as much as him.

31. If Pete was an athlete, he would have made more money.


(a) If Pete was an athlete, he. (b) If Pete had been an athlete, he.
(c) If Pete were a athlete, he. (d) If Pete were an athlete, he.
(e) If Pete would be an athlete, he.

32. Walking swiftly through the town, a straight coarse was kept by the man.
(a) the straight coarse was kept by the man (b) the straight coarse was kept by the man.
(c) the man kept a straight coarse. (d) the man kept a straight coarse.
(e) a Straight Course must be kept.

33. He tried and be better than any player.


(a) tried and be better than any. (b) tried to be better than any.
(c) tried to be better than any. (d) tried to be better than any.
(e) tried and be better than any other.

34. His mother says that he must shower, dress, and to leave.
(a) must shower, dress, and to leave. (b) has got to shower, dress, and to leave.
(c) must shower, dress, and to leave. (d) has got to shower, to dress, and leave.
(e) must shower, dress, and leave.

35. They lived in their present house for the past ten years.
(a) lived in their present house for the past. (b) lived in their present house for the passed.
(c) did live in their present house for the past. (d) have live in their present house for the passed.
(e) have lived in their present house for the past.
Directions: In each of the following examples, there are five variations of the same sentence. Select the best
version and blacken the corresponding space on your answer sheet.

36. (A) He saw the bird in the sky that was almost extinct.
(B) He saw the bird in the sky that were almost extinct.
(C) He saw the bird in the sky that had almost been extinct.
(D) In the sky, he saw the bird that was almost extinct.
(E) In sky, he saw the bird that was all most extinct.

37. (A) He either enjoys speaking in public nor to teach.


(B) He neither enjoys speaking in public nor teaching.
(C) He either enjoys speaking in public or teaching.
(D) He either enjoys speaking in public nor to teach.
(E) He either enjoys speaking in public nor to teaching.

38. (A) The man whom you just met is rowdy, boisterous, and sometimes lets his temper go.
(B) The man who you just met is rowdy, boisterous, and sometimes lets his temper go.
(C) The man that you just met is rowdy, boisterous, and sometimes lets his temper go.
(D) The man who you just met is rowdy, boisterous, and temperamental.
(E) The man whom you just met is rowdy, boisterous, and bad-temper.

39. (A) A team can’t trade a player who played for his present team for five years.
(B) A team can’t trade a player whom played for his present team for five years.
(C) A team can’t trade a player who has played for his present team for five years.
(D) A team can’t trade a player who has played for his present team for five years.
(E) A team can’t trade a player who played for his present team for five years.

40. (A) When you are in Hong Kong, one must eat at the Peninsula Hotel.
(B) When you are in Hong Kong, one should eat at the Peninsula Hotel.
(C) When you are in Hong Kong, you must eat at the Peninsula Hotel.
(D) When you are in Hong Kong, one could eat at the Peninsula Hotel.
(E) When you are in Hong Kong, one ought to eat at the Peninsula Hotel.

41. (A) Observing the dog in the road, the bus stopped immediately.
(B) While observing the dog in the road, the bus stopped immediately.
(C) Observing the dog in the road, the driver immediately stopped the bus.
(D) Observing the dog in the road, the bus was stopped immediately.
(E) After observing the dog in the road, the bus has stopped immediately.

42. (A) If I had the money, I would have bought the 24-carat gold ring.
(B) If I have the money, I would have bought the 24-carat gold ring.
(C) If I had the money, I would have bought the 24-carrot gold ring.
(D) If I had had the money, I would have bought the 24-carrot gold ring.
(E) If I had had the money, I would have bought the 24-carat gold ring.

43. (A) Deliver this check to whomever is behind the clerks desk.
(B) Deliver this check to whomever is behind the clerks desk.
(C) You deliver this check to whomever is behind the clerks desk.
(D) Deliver this check to whoever is behind the clerks desk.
(E) Deliver this check to who ever is behind the clerks desk.
44. (A) The horse always kicked over its bucket of water.
(B) The hoarse always kicked over its bucket of water.
(C) The horse always kicked over it’s bucket of water.
(D) The horse all ways kicked over it’s bucket of water.
(E) The hoarse always kicked over it’s bucket of water.

45. (A) He asked, “How do you know that the culprit is him?”
(B) He asked, “How do you know that the culprit is he?”
(C) He asked, “How do you know that the culprit is him?”
(D) He asked, “How do you know that the culprit was he?”
(E) He asked, “How do you know that the culprit was he?”

46. (A) ‘’Are any of the workers still in the room?’’ the boss asked.
(B) ‘’Is any of the workers still in the room?’’ the boss asked.
(C) ‘’was any of the workers still in the room?’’ the boss asked.
(D) ‘’Are any of the workers still in the room?’’ the boss asked.
(E) ‘’Are any of the workers still in the room?’’ the boss asked.

47. (A) Harvey or I are coming to your house.


(B) Harvey or I is coming to your house.
(C) Harvey or I am coming to your house.
(D) Harvey or I are coming to you’re house.
(E) Harvey or I am coming to you’re house.

48. (A) There is many men who would like to be in your shoes.
(B) There is many men whom would like to be in your shoes.
(C) There are many men whom would like to be in your shoes.
(D) There are many men who would like to be in your shoes.
(E) There was many men who would like to be in your shoes.

49. (A) He told us that not only the farmer but also me were going to be put in jail.
(B) He told us that not only the farmer but also I were going to be put in jail.
(C) He told us that not only the farmer but also me am going to be put in jail.
(D) He told us that not only the farmer but also I was going to be put in jail.
(E) He told us that not only the farmer but also me were going to be put in jail.

50. (A) Carl, as well as his two brothers, was invited to the picnic.
(B) Carl, as well as his two brothers, were invited to the picnic.
(C) Carl, as well as his too brothers, was invited to the picnic.
(D) Carl, as well as his two brothers, are invited to the picnic.
(E) Carl, as well as his too brothers, were invited to the picnic.
ADVANCE GRAMMAR PRACTICE TEST 4.
50 QUESTIONS – 30 MINS
1. Some of the men who was present grumbled when the game was postponed. No error.
A B C D E
2. The major drawback are the people who wish to block the plan before it is put into practice. No error.
A B C D E
3. The boys knew that the worker who was there was him. No error.
A B C D E
4. Given this parcel to whomever is at the appointed place. No error.
A B C D E
5. The accident did not injure the driver as much as they. No error.
A B C D E
6. If Sally would have been there, the youngsters would not have been so sad. No error.
A B C D E
7. Upon arriving at the stadium, the ticket windows were mobbed with people. No error.
A B C D E
8. Mr. Jackson is the best salesman of the two whom we have seen. No error.
A B C D E
9. This movie is as outrageous as or more outrageous than the one we saw last night. No error.
A B C D E
10. The frantic wife cried. ‘’Do you only like to sleep and reading the newspaper ? ‘’ No error.
A B C D E
11. Robert was always kind, considerate, and liked to think of others. No error.
A B C D E
12. When the runner reached second base, the batter already struck out. No error.
A B C D E
13. Mrs. Muzaffar will not be able to buy another ring like that one any where. No error.
A B C D E
14. The consensus of opinion among the board of directors was to sell the company. No error.
A B C D E
15. Because of your incorrect directions, the driver may of made the wrong turn. No error.
A B C D E
16. I know that I last saw his car somewhere in the vicinity of that clothing store. No error.
A B C D E
17. We would not make the decision between the five of us. No error.
A B C D E
18. Norman is taller then any other boy in the class. No error.
A B C D E
19. The lawyer sighted several examples of this type of case to prove his point. No error.
A B C D E
20. Do you mean to imply that he was present at the scene of the crime? No error.
A B C D E
21. The young salesman showed great talent , consequently , he was given a raise in salary. No error.
A B C D E
22. ‘ ‘ That boy ‘ ‘ , shouted my father, ‘ ‘ will never grow up!’ ‘ No error.
A B C D E
23. While he was in Hong Kong. Mr. Smith found that many Chinese live there. No error.
A B C D E
24. The Brazilian immigrant was startled by the size of New York City. No error.
A B C D E
25. They always enjoyed Uncle Marvin’s company. No error
A B C D E
26. I disremember whether or not I was at the game. No error.
A B C D E
27. Bob and myself went to the most exciting play on Broadway. No error.
A B C D E
28. Danish should of known not to remain in Kafiristan for too long. No error.
A B C D E
29. My father always let’s Arty repair the television set. No error.
A B C D E
30. Walking through the destroyed town, the civil engineer surveyed the affect of the storm. No error.
A B C D E
31. Both the city council and the mayor is unsure of the solution.
(a) council and the mayor is (b) council and the mayor is (c) council and the mayor are
(d) council and the mayor are (e) council and the mayor is

32. My father says that everyone in the army must get their hair cut short.
(a) everyone in the army must get their. (b) every one in the army must get their.
(c) every-one in the army must get his. (d) everyone in the army must get his.
(e) everyone in the army might get their.

33. Dr. Bingley and his family takes fewer vacations than us.
(a) fewer vacations than us.
(b) less vacations than us.
(c) fewer vacations than we.
(d) less vacations than we.
(e) fewer vacations than we.

34. Although that opera star had many fans, I never liked him singing.
(a) fans, I never liked him singing.
(b) fans I never liked him singing.
(c) fans; I never liked his singing.
(d) fans I never liked his singing.
(e) fans, I never liked his singing.

35. Walking away, the guests left the host with he.
(a) the guests left the host with he.
(b) the host was left by the guests with him.
(c) the guests left the host with him.
(d) the guests leaved the host with him.
(e) the guests they the host with him.

36. If he had been taller, he would have been a great basketball player.
(a) If he had been taller, he.
(b) If he would have been taller, he.
(c) If he had been tallest, he.
(d) If he could have been taller, he.
(e) If he was just taller, he.

37. To make sure that they do not break, the dishes must be handled carefully.
(a) the dishes must be handled carefully.
(b) the dishes must be handled with care.
(c) the dishes should be handled carefully.
(d) you must handled the dishes carefully.
(e) the dishes must handle carefully.
38. When I approached the old man, he greets me respectfully.
(a) he greets me respectfully.
(b) he greeted me respectfully.
(c) he greets me respectively.
(d) he greeting me respectively.
(e) he greeted me respectively.

39. Jumping over the hurdles, the runner may of twisted his ankle.
(a) the runner may of twisted his ankle.
(b) the runner may have twisted his ankle.
(c) his ankle may of been twisted.
(d) his ankle may have been twisted.
(e) the runner may of twisted its ankle.

40. The adventurer told us that he used to be able to go anywhere.


(a) us that he used to be able to go anywhere.
(b) we that he used to be able to go anywhere.
(c) us that he used to could go anywhere.
(d) ourselves that he used to be able to go anywhere.
(e) us that he used to be able to go any where.

Directions: In each of the following examples, there are five variations of the same sentence. Select the best
version and blacken the corresponding space on your answer sheet.

41. (A) They found out that the mission had been a success by the next day.
(B) They found out that the mission was a success by the next day.
(C) They are finding out that the mission had been a success by the next day.
(D) They found out that the mission by the next day was a success.
(E) By the next day, they found out that the mission was a success.

42. (A) The commander of the men says that they must march or that they will die.
(B) The commander of the men says that they must march or die.
(C) The commander of the men say that they must march or that they will die.
(D) The commander of the men say that they must march or die.
(E) The commander of the men says that they must march or that they would die.

43. (A) My former instructer always said that the work was hard.
(B) My formal instructor always said that the work was hard.
(C) My former instructor all ways said that the work was hard.
(D) My former instructor always said that the work is hard.
(E) My former instructor always said that the work is hard.

44. (A) When the police arrived at the scene of the crime, the robbers had already escaped.
(B) When the police arrived at the scene of the crime, the robbers already escaped.
(C) When the police arrived at the scene of the crime, the robbers al ready escaped.
(D) When the police arrived at the scene of the crime, the robbers have already escaped.
(E) When the police arrived at the scene of the crime, the robbers had already escaped.

45. (A) Reaching into the hat, a group of rabbits was pulled out by the magician.
(B) Reaching into the hat, a group of rabbits were pulled out by the magician.
(C) Reaching into the hat, a group of rabbits are pulled out by the magician.
(D) Reaching into the hat, a group of rabbits was pulled out buy the magician.
(E) Reaching into the hat, the magician pulled out a group of rabbits.
46. (A) If I were your father, I would make you behave.
(B) If I was your father, I would make you behave.
(C) If I were you’re father, I would make you behave.
(D) If I was you’re father, I would make you behave.
(E) If I am your father, I would make you behave.

47. (A) This basket of peaches is to be given to the man whom is across the hall.
(B) This basket of peaches are to be given to the man whom is across the hall.
(C) This basket of peaches are to be given to the man who is across the hall.
(D) This basket of peaches is to be given to the man who are across the hall.
(E) This basket of peaches is to be given to the man who is across the hall.

48. (A) William dislikes they’re continuous smoking.


(B) William dislikes their continuous smoking.
(C) William dislikes they’re continual smoking.
(D) William dislikes their continual smoking.
(E) William dislikes them continual smoking.

49. (A) Some of the girls are two concerned with her clothes.
(B) Some of the girls is two concerned with her clothes.
(C) Some of the girls are too concerned with her clothes.
(D) Some of the girls is too concerned with her clothes.
(E) Some of the girls are too concerned with their clothes.

50. (A) Half of their house were destroyed.


(B) Half of they’re house were destroyed.
(C) Half of their house are destroyed.
(D) Half out of their house were destroyed.
(E) Half of their house was destroyed.
ADVANCE GRAMMAR PRACTICE TEST 5
50 QUESTIONS – 30 MINS
1. When the speech stopped, every one of the guests were caught in an awkward position. No error.
A B C D E
2. Harvey told me that ham and eggs are his favorite breakfast. No error.
A B C D E
3. I think everyone should do this very easy task by themselves. No error.
A B C D E
4. Larry’s friend, whom is on vacation, often plays tennis at the school. No error.
A B C D E
5. Did Douglas say that he knew whom that man was? No error
A B C D E
6. The interviewer at the university rejected both my brother and he. No error
A B C D E
7. Sue said to me that she wished that Greg was here. No error.
A B C D E
8. Upon entering my mother’s room, the lost glove was soon found on the floor. No error.
A B C D E
9. To safeguard against natural disasters, the building that was just completed was built with steel and
A B C D
concrete. No error.
E
10. Their mother told me that John is the smarter of the three brothers. No error.
A B C D E
11. This fish is more big than any other fish that I have seen. No error.
A B C D E
12. The soldiers who just arrived were told to shower, to put on new clothes, and that they would report in the
A B C D
morning. No error.
E
13. Living life with gusto, the old man told us that he still liked to walk, to read, and to play golf. No error.
A B C D E
14. My father always believed that man is a greedy animal. No error.
A B C D E
15. When the guests left, the host already fell asleep. No error.
A B C D E
16. After tasting the flavors of ice cream, Mabel said that she thought that they were both alike. No error.
A B C D E
17. If you had been here, we could of left long ago. No error.
A B C D E
18. I know that Uncle Ralph would not have liked that kind of a house. No error.
A B C D E
19. My teacher says that we may refer back to our notes. No error.
A B C D E
20. My sister told me that he is a extremely ugly man. No error.
A B C D E
21. Who’s book is that on my brother’s desk? No error.
A B C D E
22. The coach was happy because the infielder made less errors today. No error.
A B C D E
23. After I had been given the two choices for the evening meal, beans or steak. I opted for the later. No error.
A B C D E
24. In the passed, the man who lives in that house has gone on some interesting trips. No error.
A B C D E
25. The children watched respectfully while their father poured the wine. No error.
A B C D E
26. I would love to attend your party. However , I have a prior appointment. No error.
A B C D E
27. Howard asked, “when did that man come to town”?No error.
A B C D E
28. The French ambassador was unable to attend the special banquet. No error.
A B C D E
29. Bob said that his Brother is attending college in Boston. No error.
A B C D E
30. Speaking with an impeccable accent, the English visitor told us of the days when Britain ruled the seas.
A B C D
No error.
E
31. If he would have known that either Edward or 1 was going to come, things would have turned out
differently.
(a) If he would have known that either Edward or I was.
(b) If he would know that either Edward or I was.
(c) If he had known that either Edward or I were.
(d) If he had known that either Edward or I was.
(e) If he had known that either Edward or I was.

32. The policeman says that you must make sure that one knows how to both swim and run.
(a) you must make sure that one knows
(b) you must make sure that you know
(c) you must make sure that you knows
(d) you must make sure that one know
(e) you must make sure that one no

33. All of them appreciated the compliment except he.


(a) them appreciated the compliment except he.
(b) they appreciated the compliment except he.
(c) them appreciated the compliment except he.
(d) them appreciated the compliment except him.
(e) them appreciated the compliment except him.
34. Mrs. Ross’ daughter is the most prettiest girl in the class.
(a) Mrs. Ross’ daughter is the most prettiest.
(b) Mrs. Rosses daughter is the most prettiest.
(c) Mrs. Ross’ daughter is the prettiest.
(d) Mrs. Ross’ daughter is the most prettier.
(e) Mrs. Rosses’ daughter is the most prettiest.
35. Jim exclaimed, “I would not have helped you anyways!”
(a) exclaimed, “I would not have helped you anyways!”
(b) exclaimed, “I would not have helped you anyway!”
(c) exclaimed, “I would not have helped you anyway!”
(d) exclaimed, “I would not have helped you anyways!”
(e) exclaimed, “I would not have helped you anyways!”
36. The tornado ripped through the central part of town, toppling small buildings, uprooting trees, and power
lines were snapped.
(a) Power lines were snapped
(b) Power lines snapping
(c) Snapping power lines
(d) Snapped power lines
(e) Power lines snapped

Directions: In each of the following examples, there are five variations of the same sentence. Select the best
version and blacken the corresponding space on your answer sheet.

37. (A) My father always liked to say, ‘’You’re a better man than I am, Gunga Din!’’
(B) My father always liked to say, “Your a better man than I am, Gunga Din!”
(C) My father all ways liked to say, “You’re a better man than I am, Gunga Din!”
(D) My father always liked to say, “You’re a better man then I am, Gunga Din!”
(E) My father always liked to say, “You’re a better man than I am, Gunga Din!”

38. (A) Driving very slowly, the storm almost blew our car off the road.
(B) Driving very slowly, my father struggled to keep the car on the road.
(C) Driving very slowly, the storm almost blew our car off the road.
(D) Driving very slowly, our car was almost blew off the road.
(E) Driving very slowly, the storm all most blew our car off of the road.

39. (A) He wanted to go with Lisa and she, but he was unable to.
(B) He wanted to go with Lisa and she, but he was unable to.
(C) He wanted to go with Lisa and her, but he was unable to.
(D) He wanted to go with Lisa and herself, but he was unable to.
(E) He wanted to go with Lisa and her; but he was unable to.

40. (A) If I was you, I’d be opposed to his going alone.


(B) If I am you, I’d be opposed to his going alone.
(C) If I were you, I’d be opposed to him going alone.
(D) If I was you, I’d be opposed to him going alone.
(E) If I were you, I’d be opposed to his going alone.

41. (A) Mr. Baabar Knew that everybody could do it for theirselves.
(B) Mr. Baabar Knew that everybody could do it for theirselves.
(C) Mr. Baabar Knew that everybody could do it for himself.
(D) Mr. Baabar Knew that everybody could do it for themselves.
(E) Mr. Baabar Knew that everybody could do it for itself.

42. (A) The mechanic promised my mother and she that he would fix our new car two months ago.
(B) Two months ago the mechanic promised my mother and her that he would fix our new car.
(C) Two months ago the mechanic promised my mother and she that he would fix our new car.
(D) The mechanic promised my mother and she two months ago that he would fix our new car.
(E) The mechanic promised my mother and she that he would fix two months ago our new car.

43. (A) All that were on the floor was one piece of the machine.
(B) All that was on the floor were one piece of the machine.
(C) All that are on the floor is one piece of the machine.
(D) All that were on the floor were one piece of the machine.
(E) All that was on the floor was one piece of the machine.
44. (A) Here rest the bodies of men who gave his life for his country.
(B) Here rests the bodies of men who gave his life for his country.
(C) Here rest the bodies of men who gave their lives for their country.
(D) Here rest the bodies of men who gave their lives for his country.
(E) Here rest the bodies of men whom gave his life for his country.

45. (A) Neither Adam or I would want to go to your party.


(B) Neither Adam or me would want to go to your party.
(C) Neither Adam nor me would want to go to your party.
(D) Neither Adam or I would want to go to you’re party.
(E) Neither Adam nor I would want to go to your party.

46. (A) Talking to whomever was present, the old man told all his stories.
(B) Talking to whomever was present, many stories were told by the old man.
(C) Talking to whoever was present, the old man told all his stories.
(D) Talking to whoever was present, many stories were told by the old man.
(E) While talking to whomever was present, the old man told all his stories.

47. (A) Everyone wants to live somewheres.


(B) Everyone wants to live somewhere.
(C) Every one wants to live somewheres.
(D) Everyone wants to live somewhere.
(E) Every one wants to live somewhere.

48. (A) A bucket of lobsters were bought for supper any way.
(B) A bucket of lobsters are to be bought for supper anyway.
(C) A bucket of lobsters were bought for supper anyways.
(D) A bucket of lobsters was bought for supper anyway.
(E) A bucket of lobsters are going to be bought for supper anyway.

49. (A) A box of ice cream can’t hardly feed this crowd.
(B) A box of ice cream can’t hardly feed these crowd.
(C) A box of ice cream can hardly feed this crowd.
(D) A box of ice cream can hardly feed these crowd.
(E) A box of ice cream are hardly able to feed these crowd.

50. (A) Talking in the shower, his voice could be heard next door.
(B) While talking in the shower, his voice could be heard next door.
(C) Talking in the shower, the boy could be heard next door.
(D) Talking in the shower, his voice could was heard next door.
(E) Talking in the shower, his voice was able to be heard next door.
ANSWERS & EXPLANATIONS (TEST 1 TO 5)
Test 1 26. D (correct form is we)
27. B (correct form is was)
1. C (correct form is was attending)
28. C (correct form is was)
2. B (correct form is were)
29. C (correct form is am)
3. C (correct form is were)
30. A (correct form is are)
4. D (correct form is have)
31. D (had got nothing is common in British
5. E (Team can be considered either singular or
writing but considered poor in American
plural, so practice is not incorrect)
usage)
6. A (correct form is we)
32. A
7. A (correct form is I)
33. C
8. D (correct form is its)
34. A
9. C (correct form is his)
35. B
10. D (correct form is me)
36. C
11. A (correct form is were)
37. C
12. A (dangling modifier)
38. D
13. D (dangling modifier)
39. D
14. D (correct form is any other)
40. A
15. D (correct form is jogging)
41. A
16. D (correct form is sold)
42. D
17. B (correct form is has gone)
43. C
18. D (correct form is to)
44. D
19. B (correct form is as impertinent as)
45. C
20. A (dangling modifier)
46. D
21. B (correct form is were)
47. B
22. C (correct form is longer)
48. D
23. D (correct form is her)
49. E
24. A (dangling modifier)
50. A
25. D (correct form is me)

Test 2 19. B (correlatives must be placed before parallel


terms)
1. A (correct form is was )
20. D (lack of parallel structure; speedily is better)
2. D (correct form is is)
21. D (correct form is had)
3. B (correct form is were)
22. B (correct form is has stayed)
4. A (correct form is have)
23. D (correct form is anyway)
5. A (correct form is oneself)
24. A (correct form is had)
6. B (correct form is is)
25. D (a is not necessary)
7. A (correct form is I)
26. A
8. B (correct form is whomever)
27. E
9. E
28. D
10. C (correct form is his)
29. B
11. B (correct form is me)
30. D
12. C (correct form is whom)
31. B
13. A (correct form is were)
32. E
14. A (dangling modifier)
33. A
15. E
34. B
16. C (correct form is lovelier)
35. C (lack of parallel structure in other choices)
17. E
36. C
18. B (correct form is most unusual)
37. E
38. D 44. A
39. D 45. B
40. D (preferable to keep subject the same 46. C
throughout the sentence) 47. E
41. E 48. B
42. B 49. D
43. D 50. B

Test 3 25. C (correct form is cousin)


26. B
1. A (group can be considered either singular or
27. C
plural, so have is not incorrect)
28. A
2. B (correct form is am)
29. B
3. C (correct form is were)
30. E
4. D (correct form is her)
31. B
5. D (correct form is he)
32. D
6. C (correct form is who)
33. D
7. E
34. E
8. D (correct form is their)
35. E
9. B (correct form is us)
36. D
10. A (dangling modifier)
37. E
11. B (more is not necessary)
38. E
12. A (correct form is as abrasive as)
39. C
13. B (correct form is enjoys neither)
40. C
14. D (correct form is to sail, for parallel structure)
41. C
15. C (correct form is walked)
42. E
16. D (correct form is has attended)
43. D
17. D(correct form is all right)
44. A
18. D (correct form is himself)
45. B
19. B (correct form is to)
46. A
20. C (correct form is a)
47. C
21. A (correct form is we’re)
48. D
22. E
49. D
23. D (correct form is ? ‘ ‘)
50. A
24. D (correct form is English)

Test 4 14. A (of opinion is not necessary)


15. C (correct form is may have)
1. B (correct form is were)
16. C (correct form is somewhere)
2. A (correct form is is)
17. C (correct form is among)
3. D (correct form is he)
18. C (correct form is than)
4. C (correct form is whoever)
19. A (correct form is cited)
5. D (correct form is them)
20. E
6. A (correct form is had been)
21. C (correct form is :)
7. A(dangling modifier)
22. B (correct form is , ‘‘)
8. B (correct form is better)
23. C (correct form is Chinese)
9. E
24. E
10. C (correct form is to read)
25. E
11. C (correct form is thoughtful)
26. A (correct form is forget)
12. D (correct form is had struck)
27. A (correct form is I)
13. D (correct form is anywhere)
28. A (correct form is should have) 40. A
29. C (correct form is lets) 41. E
30. D (correct form is effect) 42. B
31. D 43. E
32. D 44. A (choice (E) is incorrect because an
33. E introductory subordinate clause should be set
34. E (an introductory subordinate clause should off from the main clause by a comma)
be set off from the main clause by a comma) 45. E
35. C 46. A
36. A 47. E
37. D 48. D
38. B 49. E
39. B 50. E

Test 5 27. D (correct form is ?’’)


28. A (correct form is French)
1. C (correct form is was)
29. B (correct form is brother)
2. B (the preferable form is is)
30. E
3. D (correct form is himself)
31. D
4. B (correct form is who)
32. B
5. C (correct form is who)
33. D
6. D (correct form is him)
34. C
7. D (correct form is were)
35. B
8. A (dangling modifier)
36. C
9. A (dangling modifier)
37. A
10. C (correct form is smartest)
38. B
11. B (correct form is bigger)
39. C (choice (E)is awkward because of the
12. D (correct form is to report)
semicolon, which should usually not be used
13. E
before but)
14. E
40. E
15. D (correct form is had fallen)
41. C
16. D (both is not necessary)
42. B
17. D (correct form is could have)
43. E
18. D (a is not necessary)
44. C
19. C (back is not necessary)
45. E
20. C (correct form is an)
46. C
21. A (correct form is whose)
47. D
22. D (correct form is fewer)
48. D
23. D (correct form is latter)
49. C
24. D (correct form is past
50. C
25. E
26. C (correct form is :)
MOST COMMON ERRORS-01

1. People of Karachi is of high concern for the higher authority regarding employment issues. No Error
A B C D E

2. The business community of Pakistan attract the foreign investors for establishing new joint ventures.
A B C D
No Error
E

3. Neither Jones nor his friends is interested in doing summer courses. No Error
A B C D E

4. The city of Karachi, along with the other urban cities are facing severe power shortage. No Error
A B C D E

5. Neither of you are going to play baseball as it is raining outside. No Error


A B C D E

6. Jane together with her other three friends are watching live telecast of the baseball match. No Error
A B C D E

7. Everything that lies under the large sea rock are a huge secret for the marine excavators. No Error
A B C D E

8. Each of us have to climb the tree for getting the ball to continue the game. No Error
A B C D E

9. Every student will submit their admit cards with the administration for the registration. No Error
A B C D E

10. The student who you like will get the award this year. No Error
A B C D E

11. Who ever likes to eat blue berry cake can have it at 2$ each. No Error
A B C D E

12. The pair of jeans that I bought was not comfortable enough to meet my expectations. No Error
A B C D E

13. A group of seven managers are dealing with the present issues of the company. No Error.
A B C D E

14. After the war had been fought, the tank was driven through the camps by the soldiers. No Error.
A B C D E
15. After comparison it was found that Aslam’s car performed better then that of Sadiq’s. No Error
A B C D E

16. The color you specified is different then the color Jack told us. No Error
A B C D E

17. Grueber’s intellect totally differs from the one acquired by Alan. No Error
A B C D E
18. The competition between four of us will enhance our skills. No Error
A B C D E

19. There should be a discussion among both of us for better decision-making. No Error
A B C D E

20. Despite of increasing illiteracy among the poor, there is least provision for education in the up coming
A B C D
budget. No Error
E

21. Due to the increasing global warming, cold regions are becoming hotter every year. No Error.
A B C D E

22. Now that you are using a skin cleanser, your complexion has become more clearer. No Error.
A B C D E

23. This is the most best result that you have ever achieved in Chemistry. No Error
A B C D E

24. The reason I left preparation for the examinations was because I was suffering from severe fever for
A B C D

two weeks. No Error


E

25. The people who have transformed from Christianity to Islam are the people which were erudite and
A B C D

experienced. No Error
E

26. Employees with less personal problems are likely to be more productive. No Error
A B C D E

27. The reason I will not be going to Mexico this year is because I will use up all my travel money in
A B C D

attending an important meeting in Singapore. No Error


E

28. If I would have studied hard yesterday then I might have scored better marks. No Error.
A B C D E

29. If you were to work at least four hours a day on the project, we would complete it in a shorter time, and
A B C
with less problems. No Error.
D E

30. My uncle, who was on vacation, along with my two cousins and I, went fishing down by the river.
A B C D
No error.
E
31. Everyone who visits Singapore is impressed by its cleanliness, which is mainly a result of rigorous
A B C
implementation of their strict laws. No Error.
D E

32. I will not object to his delivering the lecture as long as he is told not to make personal attacks on his
A B C D
critics. No Error
E
33. I prefer Marlene to any hairdresser I have visited in the past because she has such a good
A B C

understanding of her clients' needs. No Error.


D E

34. The archivist had not only a deep interest but a clear understanding of the historical documents in the
A B C D
museum. No Error.
E

35. Was the woman who you think you saw leaving the building wearing a nurse's uniform? No Error
A B C D E

36. The academy members waged a relentless war against my supervisor and I because our research
A B C
seemed to contradict the findings of the head of the institution. No Error.
D E

37. The most important skill I have learned in my two years of senior high school was
A B
to direct the course of my own studies and career selection .No Error.
C D E

38. Scientific advances over the last fifty years have led to revolutionary changes in health, agriculture and
A
communication, and generally enhancing socio-economic development and the quality of our lives.
B C D
No Error.
E

39. I would like to thank whoever it was that wrote that piece of music: it has given me so much pleasure.
A) I would like to thank whoever it was that wrote that piece of music:
B) I would like to thank whomever it was that has written that piece of music:
C) I would like to thank whomever it might be that wrote that piece of music:
D) Whoever it was that wrote that piece of music, I would like to thank because
E) I would like to thank whoever it was that wrote that piece of music,

40. If the gardener would sow the seeds in the greenhouse rather than the garden, he might get a better
display of flowers.
A) If the gardener would sow the seeds in the greenhouse rather than the garden
B) If the gardener sowed the seeds in the greenhouse rather than the garden
C) If the gardener would sow the seeds in the greenhouse rather than in the garden
D) If the gardener were to sow the seeds in the greenhouse rather than in the garden
E) If the gardener would sow the seeds in the greenhouse instead of the garden
41. After working for two hours, the essay started to take shape, and he began to hope that he might finish
before the deadline.
A) After working for two hours, the essay started to take shape, and he began to hope
B) When the essay started to take shape after he had worked on it for two hours, he began to hope
C) When the essay started to take shape after he had worked on it for two hours, he begun to hope
D) When the essay started to take shape after working on it for two hours, he began to hope
E) After working for two hours, the student's essay started to take shape, and he began to hope

42. After you have written a definition in your vocabulary notebook, add a few ways to
A B
use the word or a sentence to illustrate it's meaning. No Error.
C D E

43. The general reported that, as a result of financial constraints, less soldiers would be
A B C
sent on peace-keeping missions this year. No Error.
D E

44. Mack’s height is greater than John. No Error


A B C D E

45. If one works harder you get more benefit. No Error


A B C D E

46. Its you who has become the most exasperating student for the whole class. No Error
A B C D E

47. Much people who have taken retirement earlier in their life are taking higher pensions. No Error
A B C D E

48. After the inflation had gone up, the people raised the issue of increasing poverty. No Error.
A B C D E

49. The scientists of past had proven that the earth revolved around the sun. No Error
A B C D E

50. The growing interest of Henry in learning new skills has led him to learn skim
A B
reading, formal writing and to develop presentations. No Error
C D E
MOST COMMON ERRORS-03

1. The news of the loss suffered by our troops were much worse than we had expected.
A B C D

2. Just outside the ruins are a castle surrounded by high walls and stately trees.
A B C D

3. The president of the college, together with the deans, are planning a conference
A B C
for the purpose of laying down certain regulations.
D

4. Only about one in twelve of the young men and women in this country receive a college degree.
A B C D

5. Neither of the boys who have been helping us know when this game will be over.
A B C D

6. Among the forest of skyscrapers stand the 108-story-tall Empire Building


A B C
which was constructed at the cost of hundreds of millions of dollars.
D

7. The child was almost in tears as he told his mother that everyone but Everest and he had been permitted
A B C D
to go.

8. If one does not have respect for himself, you cannot expect others to respect him.
A B C D

9. Mrs. Harvey spoke harshly to both Walter and I because she thought we had torn her favorite book.
A B C D

10. Because Sam and Michelle had done all of the work their selves, they were unwilling to give
A B C
the results to Joan.
D

11. I know that the book was returned; Mrs. Brown, the librarian, took it from myself when I entered.
A B C D

12. Those TV sets are all too expensive for we to buy at this time, but perhaps we will return later.
A B C D

13. The only teachers who were required to attend the meeting were George, Betty, Jill and me.
A B C D

14. Every doctor must know both medical theory and technique so that he can apply it in helping his
A B C D
patients.

15. People are more likely to buy a newspaper if his attention is caught by something sensational.
A B C D
16. In order to get married in this state, one must present a medical report along with your identification.
A B C D
17. The findings of Gregor Mendel were first published in 1866, but scientists paid little attention
A B
to it for about fifty years.
C D

18. Fish are unable to sleep, but sometimes a fish in an aquarium will lie on their side and appear
A B C
to be completely unaware of everything around it.
D

19. Compared to prices a few years ago, prices of food and clothing today have increased
A
to such a degree that some people can’t afford to buy it.
B C D

20. The tartar chief controls a thousand troops, all of which must obey his orders in both war and peace.
A B C D

21. It was me at the door when you were taking a bath.


A B C D

22. Louise is the more capable of all the girls who are from the South.
A B C D

23. According to recent surveys, the CBS news programs have bigger audience than NBC.
A B C D

24. The Joneses have visited Hawaii and Alaska, and they assure me that they like Alaska the best.
A B C D

25. Kathy was definitely a faster swimmer than any one on her team and appeared headed for the
A B C D
state championship.

26. Some ingenious literary detectives suggest that Shakespeare’s plays were, in fact, written by
A B
Bacon, of Oxford or Derby or anyone else who preferred to remain unknown as a playwright.
C D

27. Rebuilding that worn-out carburetor will be as expensive, if not more expensive than, buying a new one.
A B C D

28. He was standing quietly when presently a young woman, who had been combing her hair and
A B C
watched him, approached and asked him for directions.
D

29. Here they used to sit in the shade through a long lazy summer day to talk over village gossip or
A B C
telling endless sleepy stories about nothing.
D

30. He is one of those persons who is never satisfied.


A B C D

31. Sitting up late last night, Tom not only read the assignment but also many poems
A B C
by one of his favorite poets.
D

32. Every day the watchman would lock the doors, turning on the spot light and walk around the building.
A B C D

33. Jim spent his vacation going to Mexico, flying to Hawaii, and then he visited Canada.
A B C D

34. Being in poor health, we advised John to stay in bed and not to go with us.
A B C D

35. Though the meeting was about to end and everybody was in a hurry to leave Richard insisted
A B C
to ask several questions.
D

36. Having finished high school, the village education board appointed him to a teaching post in the primary
A B C D
school.

37. After searching for evidence in the house, the police concluded that the thief must have come
A B C
in through the window and stole the silver while the family was asleep.
D

38. Having finished his term paper before the deadline, it was delivered to the professor before the class.
A B C D

39. She has died for ten years, but she is still remembered by all who knew her.
A B C D

40. Biologists are predicting that they are able to alter genes and control heredity.
A B C D

41. This is the second time you break a cup; you broke one yesterday, I never break anything.
A B C D

42. The pilot that was killed in the crash maintains an outstanding record of safety.
A B C D

43. The cost of groceries rose so rapidly during the past several years that lower-income
A B
families can scarcely buy what they need.
C D

44. Many centuries prior to the Norman invasion, the Romans have conquered Britain, and
A B
remained there for almost four centuries.
C D
45. What a pity we didn’t know about the show! We should like to see it!
A B C D

46. Have you read in the Columbia Spectator that Jeff’s leg was broken while playing football?
A B C D

47. All experienced hunters know that wild animals became particularly restless before a storm comes.
A B C D

48. As automation increases, the problems of unemployment will become progressively more
A B C
serious unless more men and women would be given the training necessary for white—collar positions.
D

49. We change our programs the moment we have any further news of the situation.
A B C D

50. The principle reason for the great amount of pollution is that the government does not take
A B C
the lead in passing strong legislations against it.
D

51. The curtain had hardly raised when a shot startled the audience, a scream came from offstage
A B C
and the manager rushed out to say the opera would not go on that night.
D

52. This table is not sturdy enough to support a television, and that one probably isn’t neither.
A B C D

53. Having been hurt in the traffic accident, he lay in bed for two weeks, to wait for his wound to heal
A B C D

54. She approached me timidly from the farther end of the room, and trembling slightly, sat down besides me.
A B C D

55. Of the three options presented, the latter seemed the best.
A B C D

56. No one would have attended the lecture if you told the truth about the guest speaker.
A B C D

57. If certain drugs were taken together, they can cause a person to become extremely ill.
A B C D

58. Despite of all the snow in the valley, the cabin remained warm and comfortable.
A B C D

59. I should be most happy to go down with you if I had not been busy then.
A B C D

60. Cliff’s and Al’s car broke down again, but luckily enough they knew how to fix it.
A B C D
61. What do you think we shall do if it will rain on the day fixed for the sports meet?
A B C D

62. Pal did so well in his speech today that he should have practiced it many times this past week.
A B C D

63. If I brought with me a camera when the accident happened yesterday, I might have taken a picture of
A B C
myself.
D

64. If a crisis was to occur, those unfamiliar with the procedures would not know how to handle the situation.
A B C D

MOST COMMON ERRORS-04

1. My grandmother's legacy is substantial, especially if the value of the rare stamps are taken into
A B C D
Consideration. No error.
E

2. In such areas as sports, ranking of individual performance is relatively well accepted since the parameters
A B C
on which the rating are based are generally objective. No error.
D E

3. The series of letters that Margaret wrote to her father contains a valuable commentary on the prevailing
A B
social conditions and attitudes that lead to her leaving home at such a young age. No error.
C D E

4. Neither of the artists we have commissioned to execute the mural are able to start work before September.
A B C D
No error.
E

5. The new law is too stringent; it will be neither respected or obeyed. No error.
A B C D E

6. Either of the solutions you have proposed are acceptable to the union, whose members are willing to
A B C D
compromise. No error.
E

7. A number of trainees who take this course every year find that their knowledge of mathematics is
A B C D
inadequate. No error.
E

8. Neither of the answers provided in the memorandum address my concerns about the validity of the
A B C D
procedure. No error.
E
9. Shoes of those kind are bad for the feet low heels are better. No error.
A B C D E

10. The rhododendron, which ornaments so many English gardens, is not native to Europe. No error.
A B C D E

11. The union insisted on an increase in their members’ starting pay, and threatened to call a strike if the
A B C
company refused to meet the demand. No error.
D E

12. My uncle, who was on vacation, along with my two cousins and I, went fishing down by the river. No error.
A B C D E

13. When Russell Wallace and Darwin independently proposed similar theories, Darwin had already
A B
accumulated extensive evidence with which to support his ideas. No error.
C D E
14. For a successful career as a beautician, one must be prepared to dissemble: you must not tell your client
A B C
the unvarnished truth about his or her appearance. No error.
D E

15. Everyone who visits Singapore is impressed by its cleanliness, which is mainly a result of rigorous
A B C
implementation of their strict laws. No error.
D E

16. I will not object to his delivering the lecture as long as he is told not to make personal attacks on his critics.
A B C D
No error.
E

17. Mark scored poorly on the test, which is not surprising since he did not prepare adequately. No error.
A B C D E

18. Was the woman who you think you saw leaving the building wearing a nurse's uniform? No error.
A B C D E

19. The presence of strong feeling, the cause of which is not fully understood, always has the effect of making
A B
we human beings uneasy. No error.
C D E

20. The academy members waged a relentless war against my supervisor and I because our research seemed to
A B C D
contradict the findings of the head of the institution. No error.
E

21. You seldom see anyone in Europe spitting in public nowadays, mainly because they had a successful
A B C
campaign against this health hazard in the post war years. No error.
D E
22. It is an old criticism of the medical profession that they have considered the symptoms and causes of
A B C
disease without sufficient reference to the causes of health. No error.
D E

23. The tribesmen made offerings to placate the gods, whom, they believed, were angry with them. No error.
A B C D E

24. After you have written a definition in your vocabulary notebook, add a few ways to use the word or a
A B C
sentence to illustrate it's meaning. No error.
D E

25. Television viewers claim that the number of scenes depicting alcohol consumption have increased
A B C
dramatically over the last decade. No error.
D E

26. Neither Bradley, nor more recent critics who have written on Shakespeare's tragedies, has been able to
A B C
give a convincing explanation for the timing of events in Othello. No error.
D E

27. The reason I will not be going to Mexico this year is because I will use up all my travel money attending an
A B C D
important meeting in Singapore. No error.
E

28. The teacher sat down besides the frightened child and tried to reassure him that the monster was merely
A B C D
imaginary. No error.
E

29. If you were to work at least four hours a day on the project, we would complete it in a shorter time, and
A B C
with less problems. No error
D E

30. Employees with less personal problems are likely to be more productive. No error.
A B C D E

31. Illiteracy is an enormous problem, it affects millions of people worldwide, and is an impediment to social
A B C D
progress. No error.
E

32. I prefer Marlene to any hairdresser I have visited in the past because she has such a good understanding of
A B C
her clients' needs. No error.
D E

33. The general reported that, as a result of financial constraints, less soldiers would be sent on peace-keeping
A B C D
missions this year. No error.
E
34. The ornate pillars and life-size statues that the magnate has chosen to adorn his swimming pool are like
A B C
a Greek temple. No error.
D E

35. The three richest men in America have assets worth more than the combined assets of the sixty poorest
A B C
countries of the world. No error.
D E

36. The new library is undoubtedly well stocked and functional, but no one can say that its atmosphere is
A B C
anything like the old one. No error.
D E

37. We have no choice but to appoint Mary: she is the best of the two candidates, and there is no prospect of
A B C D
finding more applicants. No error.
E

38. Determination of the long-term effects of aerosols on the upper atmosphere is currently one of the
A B C
more challenging problems in climate research. No error.
D E

39. Scientific advances over the last fifty years have led to revolutionary changes in health, agriculture and
A
communication, and generally enhancing socio-economic development and the quality of our lives.
B C D
No error.
E

40. Katz claimed that reading classic novels is more illuminating than to read autobiographies written by their
A B C D
authors. No error.
E

41. Suzanne recounted her improbable tale with enthusiasm and in a convincing manner. No error.
A B C D E

42. Safety precautions and emergency exits, matters of great concern for builders of commercial
A
establishments, are often overlooked when designing a new home. No error.
B C D E

43. Waiting for the results of the final examination, the student's nerves were on edge; she could not sleep
A B C
properly or eat normally. No error.
D E

44. He was not merely expected to contribute funds to the project, but to work as hard as the other patrons.
A B C D
No error.
E

45. If you are sure that you are in the right, you would not mind an independent examination of the case.
A B C D
No error.
E

46. You would have to choose her, if you are looking for the best athlete to represent the school. No error.
A B C D E

47. If you were willing to ask for directions, instead of doggedly driving on, we might get to our destination
A B C
sooner. No error.
D E

48. I do not wish to make a formal complaint, but I would have been better pleased if you gave the award to
A B C
the person who best deserved it. No error.
D E

49. Fifty percent of the people alive today have never made a phone call, but thirty percent still have no
A B C
electricity connections to their homes. No error
D E

50. All the trapped miners began to lose hope, it had been twenty four hours since the tunnel collapsed.
A B C D
No error.
E

51. She wondered whether the city had changed a lot since she had left to go to university. No error.
A BC D E

52. The station was a hive of bustling activity, the arrival of the train was the most important event of the day
A B C
in that remote place. No error.
D E

53. The farmer should not have been so careless as to leave the door of the house unbolted when he had gone
A B C D
to bed. No error.
E

54. While he thinks the phenomenon is the result of enzyme action, I believe it is caused by a shortage of a
A B C D
neurotransmitter. No error.
E

55. I have nearly written all the new tests for inclusion in the revised edition of my book, and hope to finish
A B C
the work within a week. No error.
D E

56. The students have been practicing for the concert since three weeks, and in that time have improved
A B C D
considerably. No error.
E

57. My grandmother sees remarkable well considering that she has endured four operations on her eyes and
A B
suffered from vitamin deficiency during her childhood. No error.
C D E

58. From ancient times, sculpture had been considered the prerogative of men, and even now, in some parts of
A B C
the world, women sculptors face hostility and suspicion. No error.
D E

59. Sonia Gandhi stepped aside just as she had secured the Prime Minister's position, and, in the process,
A B C
asserting her moral superiority. No error.
D E

60. The most important skill I had learned in my two years of senior high school was to direct the course of my
A B C D
own studies. No error.
E

61. The racing champion had made no mention in his speech of the mechanics on whom his success had
A B C D
depended. No error.
E

62. The unfortunate accident that caused the explosion was extensively reported in all the local newspapers
A B C
and national television. No error
D E

63. I agree that a knowledge of Latin is helpful to build a good English vocabulary, but I do not think I have
A B C
the capacity to study the subject at the moment. No error.
D E

64. Many people genuinely want to be fitter, but few have the tenacity for sticking to a suitable regime of diet
A B C D
and exercise. No error.
E

65. The engineer, who is renowned for his ingenuity, has designed a very unique cooling system for our new
A B C
plant in Spain. No error.
D E

66. The manager tried hard to effect a change in company policy, but the owner, who steadfastly refused to
A B
compromise, overruled him on every point. No error.
C D E
67. Neither of my brothers do anything to make life better for our parents who are both suffering from
A B C D
arthritis. No error.
E

68. Cynthia argued vehemently with her mother over her refusal to attend the school concert. No error.
A B C D E

69. As he held open the door for her, she could not ignore the look on his face, a look that aggravated her self-
A B C
consciousness as they proceeded along the street. No error.
D E

70. With skill and surprising gentleness the fireman managed to lower the injured cat down from the top of
A B C D
the tree. No error.
E

71. Professor Chandra's pioneering work on rainwater harvesting and recharging of groundwater in drought
A B
prone areas have been drawing media attention. No error.
C D E

72. According to Hume, it is not logic and reasoning that determine our actions, but emotion. No error.
A B C D E

73. It will be hard to soothe your mother now that you have so aggravated her by refusing to take her
A B
eminently sensible advice. No error.
C D E

74. The company president has taken steps to ensure that she can handle the pressure and anxiety associated
A B C
with the job, including joining a yoga class and enlisting the support of a network of friends. No error.
D E
75. My father saw how much Uncle Tom was enjoying his early retirement, and so he decided to do the same.
A B C D
No error.
E
M.C.E 01 MOST COMMON ERRORS 02 MOST COMMON ERRORS 03
S S S S S S S S
ANS ANS ANS ANS ANS ANS ANS ANS
#. #. #. #. #. #. #. #.
1 B 26 A 1 B 31 B 61 C 1 C 31 A 61 A
2 B 27 C 2 B 32 C 62 C 2 D 32 B 62 D
3 B 28 B 3 C 33 D 63 A 3 E 33 C 63 B
4 C 29 D 4 C 34 A 64 A 4 B 34 D 64 C
5 B 30 C 5 C 35 D 5 D 35 E 65 C
6 D 31 D 6 B 36 A 6 B 36 D 66 E
7 C 32 E 7 D 37 D 7 E 37 C 67 A
8 B 33 B 8 B 38 A 8 B 38 D 68 C
9 B 34 C 9 B 39 A 9 A 39 C 69 E
10 B 35 A 10 C 40 B 10 E 40 C 70 C
11 E 36 B 11 C 41 B 11 B 41 D 71 C
12 E 37 D 12 C 42 A 12 C 42 D 72 E
13 B 38 C 13 D 43 A 13 E 43 A 73 C
14 B 39 A 14 C 44 B 14 B 44 A 74 D
15 C 40 D 15 A 45 C 15 D 45 C 75 C
16 C 41 B 16 D 46 C 16 E 46 A
17 B 42 D 17 C 47 B 17 B 47 E
18 B 43 C 18 C 48 D 18 A 48 C
19 B 44 C 19 D 49 A 19 C 49 B
20 A 45 C 20 B 50 B 20 B 50 C
21 A 46 A 21 A 51 B 21 C 51 B
22 D 47 A 22 A 52 D 22 A 52 B
23 B 48 C 23 D 53 B 23 B 53 D
24 C 49 C 24 D 54 D 24 D 54 A
25 D 50 D 25 B 55 C 25 C 55 A
26 56 C 26 C 56 B
27 B 57 C 27 C 57 A
28 D 58 A 28 B 58 B
29 D 59 A 29 D 59 D
30 D 60 C 30 A 60 B
Sentence Completion Exercises
Select the word or set of words that best completes each of the following sentences.

EXERCISE A.
15 Question – 10 minutes.

1. Those people who are interested in today are concerned about the effect of toxic wastes on
our environment.
(a) sociology (b) evolution (c) entomology (d) ecology (e) geology

2. Promotion in that company was based all too frequently on favoritism rather than on .
(a) nepotism (b) relationships (c) seniority (d) ability (e) loyalty

3. My were when I heard his explanation; I was convinced that he was telling the
truth.
(a) suspicions - confirmed (b) companions - startled
(c) misgivings – aroused (d) fears - distracted (e) doubts – dispelled

4. Although she was normally a individual, she attacked the heckler who had been interrupting
her speech.
(a) perceptive (b) pusillanimous (c) peaceful (d) choleric (e) boastful

5. The doctors have ordered additional tests because the patient has not as rapidly as had been
expected.
(a) reiterated (b) recuperated (c) deteriorated (d) qualified (e) behaved

6. The owners of the spa advertised that their were especially for the
arthritic.
(a) springs – toxic (b) waters – deleterious
(c) mountains – beneficial (d) facilities – inaccessible (e) waters – healthful

7. Somber news from the flood-stricken area does not justify the attitude which you are
displaying.
(a) lugubrious (b) sanguinary (c) belligerent (d) optimistic (e) gloomy

8. The trite and situations in many television programs alienate many viewers.
(a) predictable (b) novel (c) unpredictable (d) strange (e) gruesome

9. Ever since Pearl Harbor, we realized that we have to be on the alert to prevent a sneak
attack from any adversary.
(a) constantly (b) frequently (c) occasionally (d) obviously (e) manifestly

10. We find it hard to believe that there is no between the resignation of the company
treasurer and the missing found.
(a) connection (b) competition (c) understanding (d) cohesion (e) condescension

11. Voting in local, state, and national elections is the of every American citizen.
(a) imposition (b) emotion (c) respect (d) limit (e) duty

12. Since you disagree with John’s arguments in this dispute, your of his position is ___
.
(a) support – hypocritical (b) approval – understandable
(c) endorsement – logical (d) disapproval – unexpected
(e) denunciation – surprising
13. Peter has a bad habit of making ____ remarks which cause us to forget the gist of what he is
saying.
(a) awkward (b) pertinent (c) digressive (d) telling (e) tentative

14. Few in the audience waited for the ; they had found the performance amateurish and
boring.
(a) curtain (b) epilogue (c) overture (d) playbill (e) dialog

15. Here in America, we have a speech that is neither American, Oxford English, nor English
but a of all three.
(a) motley – conflagration (b) hybrid – combination
(c) nasal – mutilation (d) mangled – conglomeration
(e) feigned – masquerade

EXERCISE B
15 Question – 10 minutes.

1. We were annoyed by her reply for we had been led to expect definite assurances of her
approval.
(a) acerbic (b) noncommittal (c) vehement (d) caustic (e) articulate

2. Because of anticipated the captain had the “Fasten Seat Belts” sign on during the
entire flight.
(a) sabotage (b) difficulties (c) activity (d) turbulence (e) piracy
3. Intolerance among groups is deplorable and usually is based on lack of
information about the cultural groups involved.
(a) ethnic (b) aesthetic (c) belligerent (d) cohesive (e) alien

4. In such weather, you must dress properly in order to avoid catching a cold or the flu.
(a) incompatible (b) variegated (c) salubrious (d) inclement (e) balmy

5. John Ferguson was widely ___ for his contributions to scientific research and was __ for the
Nobel Prize.
(a) castigated – proposed (b) renowned – lauded
(c) known – condemned (d) lauded – ignored
(e) acclaimed – nominated

6. It was a great victory, but its ___ effect will not be _________ until we learn exactly how badly
the enemy fared.
(a) immediate – accomplished (b) long – time - felt
(c) real – appreciated (d) finite – apprehended (e) lasting – implemented

7. The child was not in his response; he showed his openly.


(a) candid – anger (b) inhibited – emotions (c) quick – eagerness
(d) hesitant – indecision (e) slow – hesitation

8. Janet was liked by everyone she met because of her manner and attractive appearance.
(a) universal (b) fortuitous (c) auspicious (d) insipid (e) winsome

9. Because your solution to this problem is so , I must oppose its adoption by this
committee.
(a) practical (b) enigmatic (c) diabolical (d) gratuitous
(e)superannuated
10. John was awarded the Medal of Honor in recognition of his in the very fierce battle.
(a) conformity (b) attitude (c) accuracy (d) garrulity (e) fortitude

11. She should be for her ruthless attack on the reputation of her hapless opponent.
(a) censured (b) extolled (c) admired (d) affronted (e) maligned

12. I have no in this matter: I am forced to follow the guidelines set forth in this
manual.
(a) grudge (b) qualifications (c) prudence (d) wisdom (e) latitude

13. In the face of family crises that would have undone a lesser woman, Eleanor Roosevelt
maintaining an air of composure and dignity.
(a) fled – unseemly (b) cowered – spurious (c) persevered – unruffled
(d) triumphed – false (e) collapsed – hard – won

14. If you come to the conference with such attitude, we cannot expect to reach a
agreement.
(a) a subservient – agreement (b) an indolent – passive
(c) an unwonted – hypothetical (d) an obdurate – harmonious
(e) a complicated – conclusive.

15. She is so hypocritical that she feelings she does not possess but feels she
should ______.
(a) depicts – allow (b) manifests – demonstrate (c) desires – portray
(d) simulates – display (e) despises – abhor

EXERCISE C
15 Questions – 10 minutes.

1. The _____ qualities of this salve will provide you with temporary relief from the pain which you
now suffer.
(a) obscure (b) analgesic (c) soporific (d) caustic (e) esthetic

2. We were amazed at the of the culprits; they were not at all embarrassed by their arrest.
(a) arrest (b) brazenness (c) indignation (d) humility (e) impotence

3. Because she was a writer, she was able to complete four novels by the end of the
year.
(a) perfunctory (b) voracious (c) controversial (d) prolific (e) veritable

4. You term paper is too wordy; try to make it more .


(a) verbose (b) succulent (c) superficial (d) conciliatory (e) succinct

5. Polls indicate that many prospective voters in the next presidential election are about the
outcome; they do not seem to care who wins.
(a) enthusiastic (b) inadequate (c) antagonistic (d) apathetic (e) suspicious

6. The man was advised by his physician to go on a rigorous diet to lose weight.
(a) chronic (b) corpulent (c) cadaverous (d) frightened (e) emaciated
7. I object to this contract because it is ; it stipulates my responsibilities but not yours.
(a) unilateral (b) indifferent (c) questionable (d) inevitable (e) universal

8. Aristotle maintained that tragedy created a by purging the soul of base concepts.
(a) vacuum (b) compromise (c) paradox (d) problem (e) catharsis

9. Before embarking on his first trip to Europe, John discussed his with his friends
and his travel agent; he left little to chance.
(a) itinerary (b) meanderings (c) challenge (d) escapade (e) desires
10. I am seeking an solution to this dispute, one which will be fair and acceptable to both
sides.
(a) obvious (b) equivocal (c) ambivalent (d) equitable (e) infamous

11. Mary hoped to get the job not only because of the salary but also because of the health plan and other
which went with it.
(a) perquisites (b) inconveniences (c) detractions (d) details (e) prerequisites

12. Propaganda may influence the sophisticated and urbane as well as the more _________
members of the community.
(a) knowledgeable (b) worldly (c) gullible (d) philanthropic (e) blasé’

13. The sales tax has been called by many; it presents a greater hardship to the
members of the state.
(a) progressive – disadvantaged (b) retrospective – affluent (c) regressive –
poorer
(d) fair – wealthier (e) liberal – marginal
14. Although I do not wish to ___ your activities, I must insist that you this assignment
before you start anything else.
(a) enlarge – cancel (b) prolong – finish
(c) compromise – accept (d) circumscribe – complete
(e) impute – decline
15. It is no uncommon occurrence in the world of theater, to encounter extreme
prosperity one week and financial the next.
(a) dramatic – solvency (b) chancy – ruin
(c) uncertain – betterment (d) celebrated – recovery

EXERCISE D
15 Question – 10 minutes.

1. Even though Frank’s work is often immature, his latest effort is not ; it is an
intelligent report.
(a) theoretical (b) sophomoric (c) redundant (d) atypical (e) pedantic

2. Stop resorting to ; make a direct charge or drop the matter.


(a) effrontery (b) subterfuge (c) slander (d) criticism (e) innuendo

3. Since all the financial news is so gloomy, I do not understand how you can have such
view about the future of the stock.
(a) a skeptical (b) an apprehensive (c) a roseate (d) a melancholy
(e) a negative
4. Such act cannot be condoned; we must find the culprits and punish them.
(a) an altruistic (b) a sporadic (c) an impromptu (d) a heinous (e) a fortuitous

5. The dinner has been arranged to honor the of Mr. Livingstone whose acts of
philanthropy have helped many needy individuals.
(a) diversity (b) gregariousness (c) tenacity (d) altruism (e) prodigality

6. Many in the audience were yawning because the speeches were, to say the least, .
(a) enthralling (b) soporific (c) maudlin (d) salacious (e) lachrymose

7. He displayed his nature when he contributed only 50 cents at a time while


everyone else was donating at least five dollars.
(a) penurious (b) sordid (c) philanthropic (d) specious (e) lethargic

8. I am afraid that this border raid is the of more serious attacks.


(a) harbinger (b) reply (c) antithesis (d) successor

9. During the middle of the eighteenth century, the style in furniture and
architecture, marked by scroll work and excessive decoration, flourished.
(a) gothic (b) functional (c) rococo (d) medieval (e) artistic
10. The orator delivered a speech that was by the more
sophisticated in the audience who were not swayed by the pompous language.
(a) patriotic – cheered (b) bombastic – ridiculed
(c) sententious – rejected (d) eulogistic – derided (e) fiery – ignored
11. Pacifists argued that our attitude toward our former allies could only result in a
more strained relationship.
(a) truculent (b) harmonious (c) placid (d) uncomfortable (e) subservient

12. The science of was largely influenced by Audubon and his magnificent paintings of American
birds.
(a) Virology (b) ornithology (c) endocrinology (d) photography

13. I have no in bringing this matter before this group: I am only interested in seeing that
justice prevails.
(a) preference (b) obligation (c) animus (d) predilection (e) justification

14. Like the best fiction, Mark Helprin’s Winter’s Tale is part dream, part mad
invention, and all of it hauntingly beautiful.
(a) satiric (b) prosaic (c) naturalistic (d) documentary (e) fantastic

15. They jury was convinced that some of the witnesses were guilty of since of
conflicting testimony had been heard.
(a) subornation – a refutation (b) insubordination – a quantity
(c) falsification – a absence (d) perjury – a plethora
(e) lying – a minimum
ANSWERS TO SENTENCE COMPLETION EXERCISES

SENTENCE COMPLETION EXERCISE A

1. D 4. C 7. D 10. A 13. C
2. D 5. B 8. A 11. E 14. B
3. E 6. E 9. A 12. A 15. B

SENTENCE COMPLETION EXERCISE B

1. B 4. D 7. B 10. E 13. C
2. D 5. E 8. E 11. A 14. D
3. A 6. C 9. C 12. E 15. D

SENTENCE COMPLETION EXERCISE C

1. B 4. E 7. A 10. D 13. C
2. B 5. D 8. E 11. A 14. D
3. D 6. B 9. A 12. C 15. B

SENTENCE COMPLETION EXERCISE D

1. B 4. D 7. A 10. B 13. C
2. E 5. D 8. A 11. A 14. E
3. C 6. B 9. C 12. B 15. D
ANSWER EXPLANATIONS

SENTENCE COMPLETION EXERCISE A.


1. (D) Ecology is the study of the environment and its interrelationship with organisms; therefore, the
ecologists would be fearful about the effect that toxic material would have on the environment.
2. (D) A contrast to promotion by favoritism is promotion on the basis of ability.
3. (E) If the listener believed the speaker, any doubts that he might have had must have been dispelled
(made to disappear).
4. (C) The use of the word although implies that a contrast is being made. A peaceful person would not
normally attack someone.
5. (B) Recuperate means recover health.
6. (E) A spa is a mineral spring. Its waters are thought to be healthful (conductive to health).Note the
difference in meaning between healthful and healthy (having good health).
7. (D) Bad news never justifies an optimistic attitude.
8. (A) the key to this question is and. We need a synonym for trite. This is provided by predictable.
9. (A) constantly means always, without variation.
10. (A) connection indicates a causal relation; the implication is that the treasurer’s resignation and the
missing funds are related.
11. (E) Every citizen must fulfill the obligation, or duty, of choosing the leaders of the country.
12. (A) If a person supports a position with which he disagrees his behavior may be described as
hypocritical.
13. (C) Digressive means wandering away. Such remarks can make a listener forget the topic.
14. (B) An epilogue is a speech delivered at the end of a play.
15. (B) Hybrid means derived from mixed sources.

SENTENCE COMPLETION EXERCISE B.


1. (B) Anyone expecting definite assurances would be irritated by a noncommittal (not committing
oneself to a definite course of action) reply.
2. (D) Turbulence (agitated or disturbed air condition) would cause the captain to ask the passengers to
fasten their seat belts.
3. (A) Ethnic means pertaining to a race or nationality possessing common cultural qualities.
4. (D) Inclement means severe or stormy.
5. (E) Only a person acclaimed (praised) for his work would be nominated (named) for a prize.
6. (C) Choice C is the best of the lot. Appreciate as used in this sentence means become fully aware of.
7. (B) Inhibited means holding back.
8. (E) Winsome means charmingly engaging.
9. (C) A diabolical (fiendish) solution would be opposed.
10. (E) Fortitude means courage.
11. (A) A ruthless (merciless) attack on a hapless (unfortunate) victim should be blamed or censured.
12. (E) Latitude means freedom of action.
13. (C) Roosevelt’s strength enabled her to persevere (undisturbed) dignity.
14. (D) Obdurate means stubborn and unyielding.
15. (D) Simulate means assume the appearance of. A hypocrite pretends to be different from what he
actually is.

SENTENCE COMPLETION EXERCISE C.


1. (B) Analgesic means causing relief from pain.
2. (B) If the culprits display no sense of shame or embarrassment they are guilty of brazenness.
3. (D) Prolific (producing many works) best describes this writer.
4. (E) Succinct means compact, free of unnecessary verbiage.
5. (D) Apathetic means showing little feeling, emotion, or interest. If a voted is apathetic, he does not
care who wins.
6. (B) A corpulent (stout, overweight) person would be advised to lose weight.
7. (A) Unilateral means one-sided.
8. (E) Purging the soul of base concepts is called catharsis.
9. (A) Itinerary means a detailed plan of a journey.
10. (D) equitable means just and fair.
11. (A) A perquisite is a benefit or privilege received in addition to a salary.
12. (C) In this sentence, the use of as well indicates that we are looking for a contrast to the words
sophisticated and urbane. This contrast is provided only by gullible. Propaganda will fool the gullible
(easily deceived).
13. (C) A tax is called regressive when it taxes unfairly those least able to pay. The power members of
society pay a larger proportion of the income than the weather under tax.
14. (D) Circumscribe means limit the scope of.
15. (B) Someone in a financially or chancy profession would be likely to face extremes of prosperity and
ruin.

SENTENCE COMPLETION EXERCISE D.


1. (B) Sophomoric means immature.
2. (E) An innuendo is an indirect hint.
3. (C) roseate means optimistic or promising.
4. (D) Heinous means reprehensible, wicked.
5. (D) Altruism means unselfishness. Prodigality (choice E) means extravagance.
6. (B) Soporific means causing sleep.
7. (A) Penurious means stingy.
8. (A) Harbinger conveys the meaning of forerunner or introducer.
9. (C) The architectural style marked by excessive decoration is called the rococo style.
10. (B) A pompous speech may be described as bombastic. Such a speech would be ridiculed (laughed at)
by those not influenced by pomposity.
11. (A) a truculent (aggressively hostile) attitude will result in a strained (tense) relationship.
12. (B) the study of birds is called ornithology.
13. (C) Animus (hostile feeling) is the only choice that fits this sentence.
14. (E) A novel that is part-dream and part*mad invention is best described as a work of fantasy.
15. (D) Plethora means an overabundant amount. If conflicting testimony exists in large amounts, there
is always a possibility that some of the witnesses were guilty of perjury (lying under oath).
Reading Comprehension 1 Reading Time: 5 Min

“New Answers to an Old Question: Who Got Here First?”

For most of the 20th century, the solution to the mystery of the original Americans--
where did they come from, when, and how?--seemed as clear as the geography of the
Bering Strait, the climate of the last ice age, and the ubiquity of finely wrought stone
hunting weapons known as Clovis points.

According to the ruling theory, bands of big-game hunters trekked out of Siberia sometime
before 11,500 years ago. They crossed into Alaska when the floor of the Bering Strait, drained
dry by the accumulation of water in a frozen world's massive glaciers, was a land bridge
between continents, and found themselves in a trackless continent, the New World when it
was truly new.

The hunters, so the story went, moved south through a corridor between glaciers and soon
flourished on the Great Plains and in the Southwest of what is now the United States, their
presence widely marked by distinctive stone projectile points first discovered near the town of
Clovis, New Mexico. In less than 1,000 years, these Clovis people and their distinctive stone
points made it all the way to the tip of South America. They were presumably the founding
population of today's American Indians.

Now a growing body of intriguing evidence is telling a much different story. From Alaska to
Brazil and southern Chile, artifacts and skeletons are forcing archaeologists to abandon Clovis
orthodoxy and come to terms with a more complex picture of earliest American settlement.
People may have arrived thousands to tens of thousands of years sooner, in many waves of
migration and by a number of routes. Their ancestry may not have been only Asian. Some of
the migrations may have originated in Australia or Europe.
Answering Time: 5, Min
1. What is the main idea of this passage?
A) Hunters from Siberia crossed the Bering Strait 11,500 years ago.
B) The Clovis people may not have been the first to arrive.
C) Clovis points were first found in New Mexico.
D) During the last ice age, the Bering Strait was dry land.

2. In this passage, trekked means


A) Traveled
B) Swam
C) Sailed
D) Hunted

3. In this passage, presumably means


A) Likely
B) Certainly
C) Easily
D) Complexly

4. The passage states that the Clovis people came to North America from
A) Australia
B) Chile
C) Siberia
D) New Mexico

5. The Clovis people are named after the place where


A) they first camped in North America
B) their tents and burials were first found
C) they crossed into North America
D) their stone points were first found

6. Scientists now believe that Native Americans originally came from


A) Siberia in a single migration about 11,500 years ago
B) all parts of North and South America
C) Europe only
D) many places, including Siberia, Europe, and Australia

7. The author's purpose in writing this passage was to


A) give information
B) provide vivid descriptions
C) tell an interesting story
D) Entertain

8. The Clovis people are best known for the type of


A) clothes they wore
B) stone points they made
C) animals they hunted
D) homes they built
Reading Comprehension 2 Reading Time: 5 Min

“Public Speaking”

What does it mean to say that we live in a world of persuasion? It means that we live among
competing interests. Your roommate's need to study for an exam may take priority over pizza.
Your instructor may have good reasons not to change your grade. And the object of your
romantic interest may, alas, have other options.

In such a world, persuasion is the art of getting others to give fair and favorable consideration
to our point of view. When we persuade, we want to influence how others believe and behave.
We may not always prevail--other points of view may be more persuasive, depending on the
listener, the situation, and the merits of the case. But when we practice the art of persuasion,
we try to ensure that our position receives the attention it deserves.

Some people, however, object to the very idea of persuasion. They may regard it as an
unwelcome intrusion into their lives or as a manipulation or domination. In contrast, we believe
that persuasion is inevitable--to live is to persuade. Persuasion may be ethical or unethical,
selfless or selfish, inspiring or degrading. Persuaders may enlighten our minds or prey on our
vulnerability. Ethical persuasion, however, calls on sound reasoning and is sensitive to the
feelings and needs of listeners. Such persuasion can help us apply the wisdom of the past to
the decisions we now must make. Therefore, an essential part of education is learning to resist
the one kind of persuasion and to encourage and practice the other.

Beyond its personal importance to us, persuasion is essential to society. The right to persuade
and be persuaded is the bedrock of the American political system, guaranteed by the First
Amendment to the Constitution.
Answering Time: 5, Min
1. What is the main idea of this passage?
A) We live among competing interests
B) When we persuade, we want to influence others
C) Persuasion is both good and bad, and it is all around us
D) Persuasion is essential to society and is built into our government

2. In this passage to prevail means to


A) Win
B) Persuade
C) Speak
D) Run

3. In this passage, inevitable means


A) Important
B) Unavoidable
C) Winning
D) Unethical

4. The passage states that some people object to persuasion because they think it is
A) dangerous to the Constitution
B) difficult to do well
C) an unwelcome intrusion
D) never successful

5. According to the passage, persuasion means to


A) get people to agree with you and do what you want
B) exercise power over other people
C) become a politician
D) get other people to consider your point of view

6. What is the main idea of the third paragraph of this passage?


A) To live is to persuade
B) We may not always prevail
C) We live among competing interests
D) Persuasion can be ethical or unethical

7. The author's purpose in writing this passage is to


A) tell an interesting story
B) persuade the reader that a point of view is correct
C) entertain the reader with many interesting examples
D) appeal to the reader’s senses.
Reading Comprehension 3 Reading Time: 5 Min

“Media Today”

Our image of what goes on in a newspaper firm is probably shaped by popular movies, T.V
shows, and books. Movies such as The Front Page (1931), All the President's Men (1976),
Absence of Malice (1981), and The Paper (1994) portray newspaper work in different ways,
with a greater or lesser emphasis on glory and heroics. What they and TV versions of
newspapering such as Lou Grant and Ink have in common, though, is that they portray the
creation of hard news stories as almost the sole activity of a paper. Not only is this version of
newspaper reality wrong, but for many newspapers a substantial amount of what they publish
doesn't even get created by that company!

What actually goes on differs between dailies and weeklies, and between newspapers with
large circulations and those with small ones. We can, however, generalize about the basic
approach to creating content. The newspaper's publisher is in charge of the entire company's
operation, which includes financial issues (getting advertising, increasing circulation), printing
issues, and editorial issues. Editorial in this case has two meanings. In a narrow sense it
means the creation of opinion pieces by the firm's editorial writers. More broadly, it means all
non-advertising matter in the paper.
Answering Time: 5, Min
1. The main idea of the first paragraph of this passage is that

A) a newspaper’s publisher is in charge of all operations


B) the word editorial has two meanings
C) media portrayals of newspapers are wrong
D) many movies have been made about newspapers

2. What is the organizational pattern of the second paragraph of this passage?

A) chronological order
B) explanation
C) spatial order
D) cause/effect

3. What is the author’s purpose in writing this passage?

A) to persuade
B) to inform
C) to describe
D) to entertain

4. After reading this passage, you can conclude that media portrayals of newspapers

A) distort reality in order to entertain


B) emphasize the role of the publisher
C) accurately portray the way newspapers are produced
D) define the term editorial incorrectly

5. Which statement from the passage is the author's opinion?

A) Movies and TV programs portray the creation of hard news as a newspaper's sole
activity.
B) What actually goes on differs between dailies and weeklies.
C) The newspaper's publisher is in charge of the entire operation.
D) Our image of what goes on in a newspaper firm is shaped by popular movies, TV
shows, and books.
Reading Comprehension 4 Reading Time: 5 Min

It was the poets of the south, the troubadours, who popularized the concept of courtly love,
revolutionary in its day. Drawing on ideas from Plato and from Arab writers, and influenced by
the growing popularity of the cult of the Virgin Mary, these [twelfth-century] poets composed
their lyric poetry and rather complex songs in the mellifluous langue d'oc and accompanied
them with the music of rebec and viol, fidel and bow, pipe and tabor (tambourine). They deified
women, according them superiority over men, and laid down codes of courtesy, chivalry, and
gentlemanly conduct. Thus were born ideals of honour and courtship that in the centuries to
come would permeate European literature and culture to such a degree that their influence is
still with us today.

Under the rules of courtly love, the mistress, who is an idealised figure, remains unattainable to
her humble, worshipping suitor, who must render her homage and prove his devotion and
loyalty over a period of time before his love is even acknowledged. In this aristocratic game--
for such it was--the woman always had the upper hand and set the pace and tone of the
relationship.

The ideals of courtly love were at breathtaking variance with contemporary notions of courtship
and marriage, and there were many. . .who regarded these newfangled ideas as subversive
and pernicious. . . . The age of the troubadours ended in the early thirteenth century.
Answering Time: 5, Min
1. You can tell from the passage that a rebec is a kind of

A) noble lady
B) style of courtship
C) poem
D) musical instrument

2. How can the author's tone in this passage best be described?

A) sympathetic
B) disapproving
C) angry
D) carefully neutral

3. After reading the passage, you can say that the ideals of courtly love are important
because they

A) still influence literature and culture today


B) were at variance with society's ideas about marriage
C) seemed evil to many people at the time
D) were based on a variety of influences

4. According to the passage, an important influence on the troubadours came from

A) French writers
B) Spanish writers
C) Arab writers
D) Italian writers

5. What is the organizational pattern of the following sentence from the passage?

"The ideals of courtly love were at breathtaking variance with contemporary notions of
courtship and marriage, and there were many. who regarded these newfangled ideas as
subversive and pernicious."

A) Comparison/contrast
B) chronological order
C) division/classification
D) spatial/place order
Reading Comprehension 5 Reading Time: 5 Min

What a strange place this is! I have now lived in this country for three months, and it seems
more and more odd all the time. The people seem friendly, especially the other students, but I
find little to talk about with them, even if my poor English were not a barrier. They want to talk
about nothing but music, TV, and dating. After class, they all go dancing or to the movies, or
they get together to play video games! No one wants to form study groups or practice speaking
English. What a very frivolous society this seems to be!

And yet, how rich everyone is! They all have TVs, stereos, refrigerators, and cars--things that
no one in my country could even dream of affording, even after a lifetime of hard work. How
can they afford these things if they do not take their studies seriously? It is a continuing puzzle
to me.

All my life, I heard that if our whole family worked hard, someday we would be able to afford to
go to America. Then, we thought, when we got here we would work even harder to learn
English and go to school so that we could get good jobs. Surely, we thought, in America
everyone works equally hard! We finally managed to get here--and what a terrible struggle it
was--and nothing is the way we expected it to be. I am so very, very puzzled.
Answering Time: 5, Min
1. The author's tone in this passage can best be described as

A) neutral
B) approving
C) sympathetic
D) disapproving

2. In this passage, frivolous means

A) rich
B) silly
C) serious
D) puzzling

3. The organizational pattern of this passage is

A) chronological order
B) example/illustration
C) cause/effect
D) definition

4. The main idea of this passage is that

A) America is a strange country


B) Americans work too hard
C) America is a rich country
D) Americans enjoy both work and school

5. What type of bias does the author of this passage demonstrate?

A) Political discrimination
B) Racial prejudice
C) Ethnic difference
D) Class distinction
Reading Comprehension 6 Reading Time: 5 Min

Before, whenever we had wealth, we started discussing poverty. Why not now? Why is the
current politics of wealth and poverty seemingly about wealth alone? Eight years ago, when
Bill Clinton first ran for president, the Dow Jones average was under 3,500, yearly federal
budget deficits were projected at hundreds of billions of dollars forever and beyond, and no
one talked about the "permanent boom" or the "new economy." Yet in that more straitened
time, Clinton made much of the importance of "not leaving a single person behind." It is
possible that similar "compassionate" rhetoric might yet play a role in the general election.

But it is striking how much less talk there is about the poor than there was eight years ago,
when the country was economically uncertain, or in previous eras, when the country felt flush.
Even last summer, when Clinton spent several days on a remarkable, Bobby Kennedy-like
pilgrimage through impoverished areas from Indian reservations in South Dakota to ghetto
neighborhoods in East St. Louis, the administration decided to refer to the effort not as a
poverty tour but as a "new markets initiative."

What is happening is partly a logical, policy-driven reaction. Poverty really is lower than it has
been in decades, especially for minority groups. The most attractive solution to it--a growing
economy--is being applied. The people who have been totally left out of this boom often have
medical, mental or other problems for which no one has an immediate solution. "The economy
has sucked in anyone who has any preparation, any ability to cope with modern life," says
Franklin D. Raines, the former director of the Office of Management and Budget who is now
head of Fannie Mae. When he and other people who specialize in the issue talk about
solutions, they talk analytically and long-term: education, development of work skills, shifts in
the labor market, adjustments in welfare reform.

But I think there is another force that has made this a rich era with barely visible poor people. It
is the unusual social and imaginative separation between prosperous America and those still
left out. . . . It's simple invisibility, because of increasing geographic, occupational, and social
barriers that block one group from the other's view.
Answering Time: 5, Min
1. In this passage, the word straitened means
A) difficult
B) wealthy
C) past
D) distant

2. In this passage, the word flush means


A) poor
B) angry
C) rich
D) hard working

3. The main idea of the passage is that


A) The country is enjoying economic growth.
B) The poor are benefiting from today's good economy.
C) In the past we were more aware of the poor than we are today.
D) In the past there were many more poor people than there are today.

4. What is the organizational pattern of the first two paragraphs of this passage?
A) order of importance
B) comparison/contrast
C) chronological order
D) example/illustration

5. Which statement from the passage is an opinion?


A) Another force has made this a rich era with barely visible poor people.
B) Clinton's tour was called a "new markets initiative."
C) Poverty really is lower than it has been in decades.
D) Eight years ago, the Dow Jones average was under 3,500, and yearly federal budget
deficits were projected at hundreds of billions of dollars.

6. After reading this passage, you can conclude that


A) the relationship between the rich and the poor has changed
B) the good economy will soon end
C) poverty will be obliterated as a result of increased wealth
D) all people benefit from good economic conditions

7. The author states that one important reason that we do not talk much about poverty is that
A) no one knows what to do about it
B) poverty really is lower than in the past
C) no one has been left out of the current boom
D) the president is not concerned about the poor
8. What is the author's purpose in writing this passage?
A) to entertain
B) to tell a story
C) to describe
D) to persuade
Reading Comprehension 7 Reading Time: 5 Min

The Rockies, the Alps, the Himalayas--something about these peaks draws us to them. Many
of us are simply inspired by their beauty, and some even risk life and limb to climb them. What
enormous forces could have shaped common rocks into such massive mountain ranges?
Elsewhere, we can see rocks that look twisted and bent. What powerful forces could so
dramatically distort such seemingly resistant material, shaking our common belief in the
hardness of rock?

Around North America, we can visit young mountains, such as the Cascades of the Pacific
Northwest, that continue to rise higher even as you read this book. We can also visit older
mountains, such as the Appalachians of eastern North America, that may have been much
loftier once but have long since reached their maximum heights and begun eroding away. We
can even visit the eroded cores of ancient, now-departed mountains, like those widely
exposed in Minnesota, Wisconsin, and northern Michigan and throughout the provinces
of eastern Canada.

We can even see future mountains growing. In the rocks along Sagami Bay near Yokohama,
Japan, lives a colony of clams called Lithophaga, or "rock eaters." These creatures scoop out
small shelters for themselves from the soft rocks at sea level, and wait there for high tide to
flood their homes and bring their meals of marine algae. At the moment of Japan's great
earthquake of 1923, the land at Sagami Bay shifted upward, leaving rows of Lithophaga to
starve 5 meters (16 feet) above sea level. Even higher rows of abandoned Lithophaga
dwellings populate the cliffs at Sagami Bay, including one that correlates with the area's 1703
tremor and another that correlates with its earthquake of 818. The rocks adjacent to the bay
have risen roughly 15 meters (50 feet) during the past 2000 years .
Answering Time: 5, Min
1. What is the main idea of the second paragraph of this passage?

A) Mountain building is still going on today.


B) People are mysteriously attracted to mountains.
C) Old, new, and ancient mountains exist.
D) Some mountains rise even higher as you read this passage.

2. In this passage, loftier means

A) higher
B) older
C) steeper
D) faster

3. In this passage, the word tremor means

A) earthquake
B) landslide
C) eruption
D) distortion

4. Reread the sentence printed in bold type in the second paragraph. What relationship
connects the parts of this sentence?

A) chronological order
B) main idea/summary
C) description
D) main idea/details

5. The author's tone in this passage can best be described as

A) objective
B) awed
C) cold
D) disapproving

6. After reading this passage, you can conclude that


A) mountain building is a rare occurrence
B) many people die in earthquakes every year
C) mountain building is going on all the time
D) more people die in climbing accidents every year

7. According to the passage, where can you see the remains of ancient mountains?

A) Japan
B) Wisconsin
C) Pacific Northwest
D) Eastern North America
Reading Comprehension 8 Reading Time: 5 Min

In Bangor, Maine, where snow and ice have paralyzed the community, Angie's headache gets
worse as her four-year-old daughter and six-year-old son start bickering again. The day-care
center and elementary school are closed, so Angie must stay home from her job at the grocery
store. She probably couldn't have gotten there anyway, since the buses have stopped running.
During the latest storm, the power went out and the house is now almost unbearably cold; the
can of spaghetti Angie opens is nearly frozen. Worry begins to creep into her head: "If I can't
work, how will I pay the rent and the cost of day care?" Her parents have money problems too,
so they can't offer financial help, and her ex- husband rarely makes his child-support
payments. On top of everything, Angie is coming down with the flu.

How do people manage such adversity, and what are its consequences for the individual?
Psychologists who study questions like these have established a specialty known as health
psychology.
Answering Time: 5, Min

1. How are the two parts of the following sentence from the passage related to each other?
"The day-care center and elementary school are closed, so Angie must stay home from her job
at the grocery store."

A) cause/effect
B) definition
C) order of importance
D) comparison/contrast

2. In this passage, adversity means

A) sickness
B) weather
C) hunger
D) trouble

3. The purpose of the first paragraph of this passage is to

A) tell a story
B) inform
C) create a scene
D) persuade

4. You can tell from the passage that health psychology is the study of

A) how people cope with the illness of their children


B) how psychological factors affect peoples' health
C) how healthy people cope with stress
D) why some people cope better than others

5. What is the tone of the first paragraph of the passage?

A) Objective
B) Awed
C) Cold
D) Disappointing
Reading Comprehension 9 Reading Time: 5 Min

Curious about which digital camera is best for you? Where you can hear reggae music in
Toronto? Or what the distance is to Mars? Question-and-answer sites offer an alternative to
the ubiquitous search engine: a place to post a question--and then wait for another Internet
user to respond.

The idea isn't new; Usenet newsgroups have let us do this for years. But Q&A sites offer
features that enhance and simplify the process. All offer eBay-like feedback, where users rate
the helpfulness of experts; over time, these scores become a benchmark score for determining
how trustworthy someone's answers are. Question-and-answer sites also e-mail you when an
expert has responded, and they offer a personalized page where you can see the questions
you've asked, the people who have responded, and what they've had to say.
Answering Time: 5, Min
1. The main idea of this passage is that

A) Question-and-answer sites are alternatives to search engines.


B) Usenet newsgroups have been around for years.
C) The idea of question-and-answer sites is not new.
D) People have a lot of questions to be answered.

2. In this passage, ubiquitous means

A) difficult
B) new
C) common
D) expert

3. The organizational pattern of the second paragraph of the passage may best be described
as
A) cause/effect
B) example/illustrations
C) description
D) classification

4. The author's purpose in writing this passage is to

A) tell a story
B) give information
C) persuade
D) entertain

5. The tone of this passage can best be described as

A) negative
B) hysterical
C) opinionated
D) objective
Reading Comprehension 10 Reading Time: 5 Min

To the Editor:

I have been reading your newspaper, the Hometown Gazette, for the past two years, ever
since I moved to Smithville. We moved here from New York City, so I am accustomed to
reading excellent newspapers such as The New York Times. In fact, we still have the Times
delivered on Sundays. The entire family enjoys reading the recipes in the magazine, as well as
the Styles section.

The Times is great, but the Gazette is another story. I've never read an article that doesn't
contain at least three or four spelling or grammatical errors. For instance, in last week's issue,
you misspelled the word "secretary," used a singular verb with a plural noun, and used "it's" as
a possessive. And that was just in the lead story! In case you never went to elementary school,
"it's" means "it is." It's not a possessive adjective!

It's a pity that this tiny little hick town has only one newspaper, because I'd like to have an
alternative to the rag you publish. I find it hard to believe your news stories. If you can't spell
correctly, how can you get your facts right?

I've been meaning to get this off my chest for some time. Please cancel my subscription. And
buy yourself a dictionary.

Sincerely,
Jane Z. Jones
Answering Time: 5, Min
1. Which phrase from the passage shows the writer's bias?

A) get this off my chest


B) three or four spelling or grammatical errors
C) tiny little hick town
D) reading the recipes in the magazine

2. The author's tone in this passage can best be described as

A) angry
B) sympathetic
C) happy
D) objective

3. Which statement from the passage is a fact?

A) We still have the Times delivered on Sundays.


B) The Times is great, but the Gazette is another story.
C) It's a pity this town has only one newspaper.
D) I am accustomed to reading excellent newspapers.

4. Which statement is an example of an invalid argument?

A) I have been reading your newspaper, the Hometown Gazette, for the past two years,
ever since I moved to Smithville.
B) In fact, we still have the Times delivered on Sundays.
C) I've never read an article that doesn't contain at least three or four spelling or
grammatical errors.
D) If you can't spell correctly, how can you get your facts right?

5. Which statement in the passage is unrelated to the writer's argument?

A) We moved here from New York City, so I am accustomed to reading excellent


newspapers.
B) The entire family enjoys reading the recipes in the magazine, as well as the Styles
section.
C) In case you never went to elementary school, "it's" means "it is." It's not a possessive
adjective!
D) Please cancel my subscription. And buy yourself a dictionary.
Reading Comprehension 11 Reading Time: 5 Min

The Iroquois were unique among Native Americans, not only because of the strength and
persistence of their alliance but also because of the role-played by their clan matrons. The
older women of each village chose its chief and could either start wars (by calling for the
capture of prisoners to replace dead relatives) or stop them (by refusing to supply warriors with
necessary foodstuffs).

Before the arrival of Europeans, the Iroquois waged wars primarily to acquire captives to
replenish their population. Contact with foreign traders brought ravaging disease as early as
1633 and thus intensified the need for captives. Simultaneously, the Europeans' coming
created an economic motive for warfare: the desire to control the fur trade and gain unimpeded
access to European goods. The war with the Hurons in the 1640s was but the first of a series
of conflicts with other tribes known as the Beaver Wars, in which the Iroquois fought
desperately to maintain a dominant position in the trade.

In the mid-1670s, just when they were achieving their goal, the French stepped in; an Iroquois
triumph would have destroyed France's plans to trade directly with the Indians of the Great
Lakes and Mississippi valley. Over the next twenty years the French launched repeated
attacks on Iroquois villages. The English offered little assistance other than weapons to their
trading partners and nominal allies. Their people and resources depleted by constant warfare,
the Iroquois in 1701 negotiated neutrality treaties with France, England, and their Indian
neighbors. For the next half-century they maintained their power through trade and skilled
diplomacy rather than warfare.
Answering Time: 5, Min

1. According to the passage, the Iroquois fought against


A. the English only
B. other Indian tribes only
C. other Indian tribes and the French
D. the French, the English, and other Indian tribes

2. The most important organization pattern in this passage is


A. definition/description
B. comparison/contrast
C. classification
D. chronological order

3. After reading this passage, you could conclude that in most Native American tribes, the
women
A. exercised economic power
B. were the principal warriors
C. did not engage in agriculture
D. did not have political influence

4. In this passage, to replenish means to


A. restore
B. resuscitate
C. convey
D. capture

5. The author's tone in this passage can best be described as


A. disapproving
B. objective
C. hostile
D. emotional

6. The passage states that after the Europeans arrived, the Iroquois fought primarily to
A. control the fur trade
B. prove their strength and courage
C. acquire land
D. fight off the Europeans

7. What organizational pattern links the parts of this sentence together?


"In the mid-1670s, just when they were achieving their goal, the French stepped in; an Iroquois
triumph would have destroyed France's plans."
A. cause/effect B. chronological order
C. simple listing D. compare/contrast

8. According to the passage, after 1701 the Iroquois remained powerful primarily because of
their
A. skill at warfare B. land
C. political alliances D. diplomatic skills
Reading Comprehension 12 Reading Time: 5 Min

So many feelings in each awkward kiss; such endless shades of response in a quick clasping
of hands.

Every time Cherry Jones and Gabriel Byrne make physical contact in the heart-stopping new
production of "A Moon for the Misbegotten," the floodgates open to a tide of clashing emotions.

Exaltation, hunger and resignation and, most acutely, a sorrow that is all the more profound for
the faint sparks of hope it lets shine through: you see flickers of all these elements from the
moment Ms. Jones, the Tony-winning star of "The Heiress," and Mr. Byrne, the accomplished
Irish film actor, first brush against each other in the revival of Eugene O'Neill's great, elegiac
love story that opened last night at the Walter Kerr Theater.

The characters they portray--Josie Hogan, a Junoesque farm woman, and James Tyrone, a
self-consuming alcoholic actor--are, on one level, as opposite as the quick and the dead, as
the flesh and the spirit. Ms. Jones is the image of earthy robustness, with her wide-legged
stance and wrestler's arms; Mr. Byrne has the wan, mechanical air of a man who has already
abdicated his spot among the living. But it is clear here that Josie and James are each wearing
a mask that only the other can remove.

What makes this "Moon," which has been lovingly directed by Daniel Sullivan, so illuminating is
the extraordinary transparency of its stars: their ability to play, and display, all those layers at
once. To watch Ms. Jones, Mr. Byrne, and Roy Dotrice, who completes the triangle of principal
performers, react to one another is to realize the degree to which O'Neill's last completed play
is about how everyone is an actor, a deceiver by necessity.
Answering Time: 5, Min
1. In this passage, the word robustness means

A. winning
B. excellence
C. mask
D. strength

2. The author's tone in this passage can best be described as

A. enthusiastic
B. objective
C. frustrated
D. disapproving

3. The organizational pattern of the fourth paragraph of this passage can best be described
as

A. chronological order
B. comparison/contrast
C. spatial order
D. cause/effect

4. What is the purpose of the third paragraph of the passage?

A. to entertain
B. to describe
C. to narrate
D. to explain

5. Which statement from the passage is a fact?

A. Ms. Jones won a Tony award for "The Heiress."


B. The play is a great, elegiac love story.
C. Josie and James are each wearing a mask that only the other can remove.
D. The actors' ability to play many emotions at once makes the performance moving.

6. In this passage, abdicated means

A. enjoyed
B. wrestled with
C. covered
D. given up

7. Describe the kind of bias that the author of this passage displays.
A. Emotional bias
B. Racial prejudice
C. Ethnic difference
D. Class distinction
Reading Comprehension 13 Reading Time: 5 Min

After the initial surge of interest in the education of children who were deaf and blind and had
mental retardation in the United States, school services for children with disabilities plateaued
for many years. It was not until the 1960s that two events converged to reignite national
interest in the needs of children with disabilities. The first of these was the election of John F.
Kennedy as president in 1960. Kennedy had a sister, Rosemary, with mental retardation, and
he was openly committed to improving the quality of life for people with mental retardation. He
did two concrete things to accomplish this goal: He set up the President's Commission on
Mental Retardation, a group of expert researchers and practitioners who identified the issues
and priorities in the field, and he supported the use of federal funds to educate teachers of
children with disabilities.

The other event of the 1960s that influences families and other advocates of children with
disabilities was the civil rights movement. The political and social demands of African
Americans for equal rights and access to opportunities at all levels of society provided an
example to families and groups working with children with disabilities of what could be
accomplished on behalf of disenfranchised groups.

In 1972, Wolf Wolfensberger articulated the principle of normalization--that people with


disabilities should have the opportunity to live as close to a normal life as possible.
Answering Time: 5, Min

1. The main organizational pattern of this passage is

A. chronological order
B. simple list
C. comparison/contrast
D. clarification

2. The author's main purpose in writing this passage is to

A. entertain
B. tell an interesting story
C. give information
D. persuade

3. According to the author, the second event that reignited interest in education of children with
disabilities was the

A. establishment of the President's Commission on Mental Retardation


B. election of John F. Kennedy as president
C. use of federal funds to educate teachers
D. beginning of the civil rights movement

4. After reading this passage, you can conclude that

A. children with disabilities were not educated before the 1960s


B. education for children with disabilities has improved since the 1960s
C. President Kennedy's successors disapproved of education for children with disabilities
D. Wolf Wolfensberger was active in the civil rights movement

5. The tone of this passage can best be described as

A. impassioned
B. objective
C. disapproving
D. hopeless
Reading Comprehension 14 Reading Time: 5 Min

The most precious fluid on earth is not oil, but water. There are few challenges as important as
conserving the world's usable water and supplying clean drinking water and water for irrigation
to those who need it. Yet this work is not getting done. Humans are depleting the earth's store
of usable water at a rate that will soon threaten our food supply. Poor water management
already kills millions of people a year and condemns hundreds of millions to hunger.

The technology exists to solve these problems. Providing slums with drinking water and
sanitation, for example, is easy to do and a cost-effective way to prevent deaths and disease.
But because those who suffer are poor, their access to water is rarely a political priority. There
is now an opportunity for progress. More than 100 water ministers from around the world,
along with thousands of water experts from villagers to scientists, have gathered in the
Netherlands. The conference will be useful if it can persuade governments and international
banks to pay more attention to conservation and fair distribution of water.

In 1980, the U.N. set a goal of safe drinking water for all by 1990. Because of international
prodding, about two billion new people received clean drinking water over the subsequent 14
years. A new commitment is urgently needed to spread these health gains, help poor farmers
and conserve the world's precious supply of usable water.
Answering Time: 5, Min
1. The author's main purpose in writing this passage is to

A. entertain
B. persuade
C. inform
D. tell a story

2. The organizational pattern used in this passage is best described as

A. chronological order
B. simple listing of ideas
C. division/classification
D. spatial order

3. What is the relationship between the parts of the following sentence from the passage?

"But because those who suffer are poor, their access to water is rarely a political priority."

A. cause/effect
B. listing of ideas
C. comparison/contrast
D. place order

4. Which statement from the passage is a fact?

A. The most precious fluid on earth is not oil, but water.


B. Few challenges are as important as conserving usable water.
C. A new commitment is urgently needed.
D. About two billion people received clean drinking water.

5. The author's tone in this passage can best be described as

A. neutral
B. impassioned
C. opinionated
D. hostile
Reading Comprehension 15 Reading Time: 5 Min

I love music, but we hear way too much of it in today's society. I mean, have you tried calling
an insurance company lately? If you're put on hold (and let's be real here--you're sure to be put
on hold), you'll be forced to listen to some country or easy-listening type drivel. Call your
doctor's office, and the same thing happens, only the forced entertainment will probably be
classical music. I'm all for culture, but how many times do you want to hear Eine Kleine
Nachtmusic in the daytime or "Winter" from Vivaldi's Four Seasons in the spring? And
personally, I've always thought that Vivaldi's music was boring and repetitive next to Handel's.

I don't know how all this force-fed music got started. I suppose it was Muzak in elevators.
Certainly, that's been around for a long time. A few years ago, the governor of Georgia got the
idea that classical music was good for a baby's brain development. He actually got the
legislature to appropriate enough money to give every new mother a classical-music tape to
take home with her. And you know what was probably on it? Eine Kleine Nachtmusic and
"Winter." The mothers might as well just call their doctors' offices. I'll bet that those tapes have
done more to make babies hate classical music than all the music appreciation courses ever
taught.

And how about when you make a business call from your office and are forced to hear music
while you're kept on hold? You can't put the call on speakerphone, because the noise would
disturb the people around you, so you're just left there holding the phone. You can't work,
because the music is blasting in your ear. I love music--but enough is enough.
Answering Time: 5, Min

1. Which sentence in the passage is logically unrelated to the paragraph it is in?

A. Call your doctor's office, and the same thing happens, only the forced entertainment will
probably be classical music.
B. How many times do you want to hear Eine Kleine Nachtmusic in the daytime or "Winter"
from Vivaldi's Four Seasons in the spring?
C. And personally, I've always thought that Vivaldi's music was boring and repetitive next to
Handel's.
D. And how about when you make a business call from your office and are forced to hear
music while you're kept on hold?

2. The tone of this passage can best be described as

A. neutral
B. annoyed
C. objective
D. approving

3. What is the relationship between the parts of this sentence?

"You can't put the call on speakerphone, because the noise would disturb the people around
you, so you're just left there holding the phone."

A. example/illustration
B. cause/effect
C. spatial order
D. chronological order

4. What is the organizational pattern of this passage as a whole?

A. example/illustration
B. chronological order
C. clarification
D. simple listing

5. Describe the type of bias that the author demonstrates in this passage.

A. Emotional bias
B. Racial prejudice
C. Ethnic difference
D. Class distinction
Reading Comprehension 16 Reading Time: 5 Min

The radical transformation of Soviet society had a profound impact on women's lives. Marxists
had traditionally believed that both capitalism and the middle-class husband exploited women.
The Russian Revolution of 1917 immediately proclaimed complete equality of rights for
women. In the 1920s divorce and abortion were made easily available, and women were urged
to work outside the home and liberate themselves sexually. After Stalin came to power, sexual
and familial liberation was played down, and the most lasting changes for women involved
work and education.

These changes were truly revolutionary. Young women were constantly told that they had to
be equal to men, that they could and should do everything men could do. Peasant women in
Russia had long experienced the equality of backbreaking physical labor in the countryside,
and they continued to enjoy that equality on collective farms. With the advent of the five-year-
plans, millions of women also began to toil in factories and in heavy construction, building
dams, roads and steel mills in summer heat and winter frost. Most of the opportunities open to
men through education were also open to women. Determined women pursued their studies
and entered the ranks of the better-paid specialists in industry and science. Medicine
practically became a woman's profession. By 1950, 75 percent of doctors in the Soviet Union
were women.

Thus Stalinist society gave women great opportunities but demanded great sacrifices as well.
The vast majority of women simply had to work outside the home. Wages were so low that it
was almost impossible for a family or couple to live only on the husband's earnings. Moreover,
the full-time working woman had a heavy burden of household tasks in her off hours, for most
Soviet men in the 1930s still considered the home and the children the woman's responsibility.
Men continued to monopolize the best jobs. Finally, rapid change and economic hardship led
to many broken families, creating further physical, emotional, and mental strains for women. In
any event, the often-neglected human resource of women was ruthlessly mobilized in Stalinist
society. This, too, was an aspect of the Soviet totalitarian state.
Answering Time: 5, Min
1. The main idea of this passage is that women in Stalinist society
A. had economic opportunities that had never been available before
B. had difficulty balancing their work and family responsibilities
C. had new opportunities but also many hardships
D. moved quickly into the highest levels of government

2. In this passage, to monopolize means to


A. hold
B. earn
C. leave
D. pay

3. In this passage, mobilized means


A. punished
B. paid
C. fought
D. used

4. The author's main purpose in writing this passage is to


A. compare different systems of government
B. tell an interesting story
C. amuse the reader
D. provide information

5. The overall organizational pattern of this passage is


A. definition
B. example/illustration
C. chronological order
D. comparison/contrast

6. How are the ideas within the following sentence related?

"Moreover, the full-time working woman had a heavy burden of household tasks in her off
hours, for most Soviet men in the 1930s still considered the home and the children the
woman's responsibility."

A. cause/effect
B. division/classification
C. summary
D. sequence/process

7. The author's tone in this passage can best be described as


A. disapproving
B. emotional
C. objective
D. sympathetic
Reading Comprehension 17

COLUMBUS, Ohio--The heart operation taking place in the pale-green tiled operating room at
the Ohio State University Medical Center was almost eerie. The patient, a 62-year-old man,
was anesthetized, swathed with blue drapes and lying face up on a narrow table. But no one
was touching him.

Instead, the operation was being performed by a robot, whose three metal arms protruded
through pencil-sized holes in the man's chest. At the ends of the robot's arms were tiny metal
fingers, with rotating wrists, that held a miniscule instrument, a light and a camera. The robot's
arms and fingers were controlled by Dr. Randall K. Wolf, sitting at a computer console in a
corner of the operating room about 20 feet away.

This sort of operation, heart surgeons say, is the start of what may be the biggest change in
their profession since heart bypass surgery began nearly 30 years ago. "The reason we make
incisions is that we have big hands," said Dr. Wolf, the director of minimally invasive surgery at
Ohio State. The robot's dainty fingers, no longer than a nail on a pinkie, at the end of the long
sticks could eliminate that constraint.

Eventually, surgeons believe, most heart surgery will be done by robots whose arms are
inserted through pencil-sized holes punched in patients' chests. Instead of directly peering into
a patient's body, surgeons will view magnified images of the operation on computer screens. In
theory, the doctor would not have to be in the same room, or even the same country, as the
patient.
Answering Time: 5, Min
1. In this passage, protruded means
A. sliced
B. cut
C. moved away
D. stuck out

2. In this passage, dainty means


A. small
B. weak
C. fat
D. quick

3. According to the passage, the reason that most operations require large incisions is that
A. patients have large organs
B. surgeons have large hands
C. large incisions take less time
D. large incisions cost less money

4. The purpose of the first paragraph of this passage is to


A. tell a story
B. describe
C. persuade
D. entertain

5. Which statement from the passage is a fact?


A. This sort of operation is the biggest change since heart bypass surgery began.
B. Eventually most heart surgery will be done by robots.
C. The robot's arms and fingers are controlled by a doctor.
D. The doctor will not have to be in the same room as the patient.

6. The main idea of this passage is that heart surgery by robots


A. is quicker than surgery done by doctors
B. was developed at Ohio State University
C. is a new and risky procedure
D. may replace surgery done by doctors

7. Based on the information in this passage, all of the following conclusions are valid except

A. All doctors at Ohio State develop new surgical techniques.


B. Robot surgery is being developed at Ohio State.
C. Robot surgery will be used on many patients in the near future.
D. Many hospitals will eventually offer robot surgery to patients.
ANSWERS OF READING COMPREHENSION 1 TO 17

S.No. R.C. 1 R.C. 2 R.C. 3 R.C. 4 R.C. 5 R.C. 6 R.C. 7


1. B C C D D A C
2. A A B A B C A
3. A B B A B C A
4. C C A C A D D
5. D D D A C A B
6. D A A C
7. A B B B
8. B D
S.No. R.C. 8 R.C. 9 R.C. 10 R.C. 11 R.C. 12 R.C. 13 R.C. 14
1. A A C C D A B
2. D C A D A C B
3. C C A D B D A
4. B B D A B B D
5. B D B B A B C
6. A D
7. A A
8. D
S.No. R.C. 15 R.C. 16 R.C. 17
1. C C D
2. B A A
3. B D B
4. A D B
5. C B C
6. A D
7. C A
8.

You might also like